Você está na página 1de 78

Answers to

even-numbered problems in
Mathematics for
Economic Analysis
Knut Sydster
Peter Hammond
Preface
Mathematics for Economic Analysis, Prentice Hall, 1995 has been out for a long time, and over the years we
have had many request for supplying solutions to the even-numbered problems. (Answers to the odd-numbered
problems are given in the main text.)
This manual provides answers to all the even-numbered problems in the text. These answers are taken
from the old Instructors Manual which is no longer available. (In fact, some of the old answers have been
extended.) For many of the more interesting and/or difcult problems, detailed solutions are provided. For
some of the simpler problems, only the nal answer is presented.
Appendix A in the main text reviews elementary algebra. This manual includes a Test I, designed for the
students themselves to see if they need to review particular sections of Appendix A.
Many students using our text will probably have some background in calculus. The accompanying Test II
is designed to give information to the students about what they actually know about single variable calculus,
and about what needs to be studied more closely, perhaps in Chapters 6 to 9 of the text.
Oslo and Coventry, November 2010
Knut Sydster and Peter Hammond
Contact addresses:
knut.sydsater@econ.uio.no
P.J.Hammond@warwick.ac.uk
Version 1.0
07122010 1113
Knut Sydster and Peter Hammond 2010
CHAPT ER 1 I NTRODUCTI ON 1
Chapter 1 Introduction
1.3
2. (a) Put p/100 = x. Then the given expression becomes a +ax (a +ax)x = a(1 x
2
), as required.
(b) $2000 1.05 0.95 = $1995. (c) The result is precisely the formula in (a). (d) With the notation
used in the answer to (a), a ax +(a ax)x = a(1 x
2
), which is the same expression as in (a).
4. (a) F = 32 yields C = 0; C = 100 yields F = 212. (b) F =
9
5
C +32
(c) F = 40 for C 4.4, F = 80 for C 26.7. The assertion is meaningless.
6. x =
R(1 p) S(1 q)
(p q)(1 p q)
, y =
pS qR
(p q)(1 p q)
if (p q)(1 p q) = 0.
(Use, for example, (A.39) in Section A.9.)
1.4
2. (a) Correct. (b) Incorrect. (c) Incorrect. (d) Correct. (e) Incorrect.
(f) Incorrect. (Usually the sum of two irrationals is irrational, but not always. For example, and
are both irrationals, but +() = 0, which is rational.)
4. (a) y 3
3
4
x (b) y >
3
2
z (c) y (mpx)/q
6. |2 0 3| = | 3| = 3, |2
1
2
3| = | 2| = 2, |2
7
2
3| = |4| = 4
8. (a) 3 2x = 5 or 3 2x = 5, so 2x = 2 or 8. Hence x = 1 or x = 4.
(b) 2 x 2 (c) 1 x 3 (d) 1/4 x 1 (e) x >

2 or x <

2
(f) 1 x
2
3, and so 1 x

3 or

3 x 1
1.5
2. (a) right, wrong (b) wrong, right (c) right, wrong
(d) and both right (e) wrong (0 5 = 0 4, but 5 = 4), right (f) right, wrong
4. x = 2. (x = 1, 0 or 1 make the equation meaningless. Multiplying each term by the common de-
nominator x(x 1)(x +1) yields (x +1)
3
+(x 1)
3
2x(3x +1) = 0. Expanding and simplifying,
2x
3
6x
2
+ 4x = 0, or 2x(x
2
3x + 2) = 0, or 2x(x 1)(x 2) = 0. Hence, x = 2 is the only
solution.)
6. (a) No solutions. (Squaring each side yields x4 = x+518

x +5+81, which reduces to

x +5 = 5,
with solution x = 20. But x = 20 does not satisfy the given equation.) (b) Just as in part (a) we nd
that x must be a solution of

x +5 = 5, which has the solution x = 20. Inserting this value of x into


the given equation we nd that x = 20 is the solution.
8. (a) x+

x +4 = 2

x +4 = 2x x+4 = 44x+x
2
x
2
5x = 0
(1)
x5 = 0
(2)
x = 5.
Here implication (1) is incorrect (x
2
5x = 0 x 5 = 0 or x = 0.) Implication (2) is correct, but it
breaks the chain of implications. (b) x = 0. (By modifying the argument we see that the given equation
implies x = 5 or x = 0. But only x = 0 is a solution.)
10. No. (To study law it seems that you need a course in logic.)
Knut Sydster and Peter Hammond 2010
2 CHAPTER 1 I NTRODUCTI ON
1.6
2. Logically the two statements are equivalent, but the second statement is still an expressive poetic rein-
forcement.
1.7
2. (a) No, not necessarily. (b) Yes (if anybody is both a painter and a poet).
4. F B C is the set of all female biology students in the university choir; MF the female mathematics
students;
_
(MB) \C
_
\T the students who study both mathematics and biology but neither play tennis
nor belong to the university choir.
6. (b) and (c) are true, the others are wrong. (Counter example for (a), (d), and (f): A = {1, 2}, B = {1},
C = {1, 3}. As for (e), note in particular that A B = A C = A whenever B and C are subsets of A,
even if B = C.)
8. 50 35 = 15 liked only coffee, 40 35 = 5 liked only tea, 35 liked both, and 10 did not like either. In
all there were 15 +5 +35 +10 = 65 who responded.
10. (a) Consider Fig. 1.7.10(a). Both sets consist of the elements in (1) and in (3).
(b) Consider Fig. 1.7.10(b): A B consists of (3), (4), (2), and (5). Then (A B) C consists of (4),
(5), (6), and (7), which is described verbally in the problem.
(1) (2) (3) B A
(4)
(1)
(7)
(3)
(2)
(5)
(6)
(8)
A
B
C
Figure 1.7.10(a) Figure 1.7.10(b)
12. (a) Consider Fig. 1.7.10(b), and let n
i
denote the number of people in the set marked (i), for i = 1, 2, . . . , 8.
The responses reported imply that: n
1
+n
3
+n
4
+n
7
= 420; n
1
+n
2
+n
5
+n
7
= 316; n
2
+n
3
+n
6
+n
7
=
160; n
1
+ n
7
= 116; n
3
+ n
7
= 100; n
2
+ n
7
= 30; n
7
= 16. From these equations we easily nd
n
1
= 100, n
2
= 14, n
3
= 84, n
4
= 220, n
5
= 186, n
6
= 46, n
7
= 16. (i) n
3
+ n
4
= 304 had read A
but not B; (ii) n
6
= 46; (iii) 1000 (n
1
+n
2
+n
3
+n
4
+n
5
+n
6
+n
7
) = 334.
(b) (i) n(A \ B) (ii) n(C \ (A B)) (iii) n(\ (A B C)) = n() n(A B C))
(c) The equality is n
1
+n
2
+n
3
+n
4
+n
5
+n
6
+n
7
= (n
1
+n
3
+n
4
+n
7
) +(n
1
+n
2
+n
5
+n
7
) +
(n
2
+n
3
+n
6
+n
7
) (n
1
+n
7
) (n
3
+n
7
) (n
2
+n
7
) +n
7
, which is easily veried. The last equality
is a special case of n(\ D) = n() n(D): The number of persons who are in , but not in D, is the
number of persons in all of minus the number of those who are in D.
Knut Sydster and Peter Hammond 2010
CHAPTE R 2 F UNCTI ONS OF ONE VARI ABL E : I NT RODUCTI ON 3
Chapter 2 Functions of One Variable: Introduction
2.2
2. F(0) = F(3) = 10, F(a +h) F(a) = 10 10 = 0
4. (a) f (1/10) = 10/101, f (0) = 0, f (1/

2) =

2/3, f (

) =

/(1 + ),
f (2) = 2/5 (b) f (x) = x/(1 + (x)
2
) = x/(1 + x
2
) = f (x) and moreover, f (1/x) =
(1/x)/[1 +(1/x)
2
] = (1/x) x
2
/[1 +(1/x)
2
] x
2
= x/(1 +x
2
) = f (x)
6. F(0) = 2, F(3) =

19, F(t +1) =

t
2
+3
8. (a) b(0) = 0, b(50) = 100/11, b(100) = 200
(b) b(50 +h) b(50) is the additional cost of removing h% more than 50% of the impurities.
10. (a) No. f (2 +1) = f (3) = 2 3
2
= 18, whereas f (2) +f (1) = 2 2
2
+2 1
2
= 8 +2 = 10.
(b)Yes. f (2+1) = f (2)+f (1) = 9 (c) No. f (2+1) = f (3) =

3 1.73, whereas f (2)+f (1) =

2 +1 2.41.
12. See Figs. 2.2.12(a) and 2.2.12(b).
x
2
x 1 1 1
1 x
x
1
x
1
x
1
x
1
Figure 2.2.12(a). Area = (x +1)
2
= x
2
+2x +1 Figure 2.2.12(b) Area = x
2
+1
14. (a) (, 2) (2, ) (b) f (8) = 5
(c) f (x) =
3x +6
x 2
= 3 3x +6 = 3(x 2) 6 = 6, which is absurd.
16. (a) is invalid: |2 + (2)| = 0, whereas |2| + | 2| = 2 + 2 = 4. (b), called the triangle inequality,
|x +y| |x| +|y|, is valid. In fact, if x and y have the same sign, then |x +y| = |x| +|y|. If x and y
have opposite signs, then we see that |x +y| < |x| +|y|. (See Problem 12.4.8 for a generalization.) (c)
is valid: |xy| = |x| |y|. (Look at the 4 different sign combinations of x and y: If x is positive and y is
negative, then xy is negative, |xy| = xy and |x||y| = (x)y = xy, and so on.) (d) is easily seen to
be valid. (e) is not valid (put x = 1). (f) is easily seen to be valid. (Note that
_
4)
2
= (2) = 2.)
(g) is valid: | 2x| = 2|x|. (h) is valid because

|x| |y|

|x y| x
2
2|x||y| + y
2

x
2
2xy +y
2
2|x||y| 2xy |x||y| x y, which is true because (c) is valid.
Knut Sydster and Peter Hammond 2010
4 CHAPTER 2 F UNCTI ONS OF ONE VARI ABL E: I NTRODUCTI ON
2.3
2. See Figs. 2.3.2(a)2.3.2(f).
1
1
y
x
1
1
y
x
1
2
3
4
1 2 3
y
x
Figure 2.3.2(a) Figure 2.3.2(b) Figure 2.3.2(c)
1
1
1 1 2 2
y
x
2
1
1
y
x
1
1
y
x
Figure 2.3.2(d) Figure 2.3.2(e) Figure 2.3.2(f)
4. See Figs. 2.3.4(a)(c).
y
2
1
1
2
x 3 2 1 1 2 3
y
2
1
1
2
x 1 2 3 4 5
y
2
1
1
2
x 3 2 1 1 2 3
Figure 2.3.4(a) Figure 2.3.4(b) Figure 2.3.4(c)
y
x
5
(2, 4)
2
2
y
x
A = (3, 2)
B = (5, 4)
2
2
P
y
1
x 1
Figure 2.3.6 Figure 2.3.10 Figure 2.3.12
Knut Sydster and Peter Hammond 2010
CHAPTE R 2 F UNCTI ONS OF ONE VARI ABL E : I NT RODUCTI ON 5
6. (5 2)
2
+(y 4)
2
= 13, or y
2
8y +12 = 0, with solutions y = 2 and y = 6. Geometric explanation:
The circle with center at (2, 4) and radius

13 intersects the line x = 5 at two points. If the radius were


2, the circle would not intersect the line x = 5. See Fig. 2.3.6.
8. (a) (x 2)
2
+(y 3)
2
= 16 (b) (x 2)
2
+(y 5)
2
= 13
10. (x 3)
2
+(y 2)
2
= (x 5)
2
+(y +4)
2
, which reduces to x 3y = 7. See Fig. 2.3.10
12. (x, y) = (1, 0), (0, 1), (1/4, 1/4) are all solutions. See Fig. 2.3.12.
14. The condition is: p +r
_
x
2
+y
2
= p +s
_
(x a)
2
+y
2
. Cancelling p, then squaring each side and
rearranging yields (r
2
s
2
)(x
2
+ y
2
) = s
2
(a
2
2ax). If r = s, then the markets are separated by the
straight line x =
1
2
a. Otherwise, completing squares yields the equation [x + as
2
/(r
2
s
2
)]
2
+ y
2
=
[ars/(r
2
s
2
)]
2
. This is a circle with center at
_
as
2
/(r
2
s
2
), 0
_
and radius |ars/(r
2
s
2
)|.
2.4
2. (a) f (5) = 0, f (3) = 3, f (2) = 0, f (0) = 2, f (3) = 4, f (4) = 0
(b) D
f
= [5, 4], V
f
= [3, 4]
4. See Figs. 2.4.4(a)(d).
1 1
x
1
y
3 1
x
1
y
1
x
1
y
1
x
1
y
Figure 2.4.4(a) Figure 2.4.4(b) Figure 2.4.4(c) Figure 2.4.4.4(d)
2.5
2. 0.78 4. See Figs. 2.5.4(a)(c).
y
1
2
3
4
x
1 2 3 4
y
5
4
3
2
1
1
x
1 2 3 4 5 6 7 8 10
y
1
1
2
3
4
x
1 2 3 4 5
Figure 2.5.4(a) Figure 2.5.4(b) Figure 2.5.4(c)
6. (a) Assume F = aC + b. Then 32 = a 0 + b and 212 = a 100 + b. Therefore a = 180/100 = 9/5
and b = 32, so F = (9/5)C +32. (b) If X = (9/5)X +32, then X = 40.
Knut Sydster and Peter Hammond 2010
6 CHAPTER 3 POL YNOMI AL S, POWERS, AND E XPONE NT I AL S
8. L: y = 3x 2; M: y =
3
4
x +
5
4
. P: (13/15, 3/5). N: y =
3
4
x
7
4
. See Fig. 2.5.8.
10. (a) 75 3P
e
= 20 +2P
e
, and so P
e
= 11. (b) P
e
= 90
12. C =
4
5
y +100. (The general equation is C = ay +b. Here 900 = a 1000 +b, and a = 80/100 = 4/5,
so b = 100.)
14. The slope is
1
x
0
+h

1
x
0
x
0
+h x
0
. Multiplying both numerator and denominator by the product (x
0
+h)x
0
, then
simplifying, yields the given expression.
16. See Figs. 2.5.16(a)(c).
x
L
M
N
y
1
1
x
y
1
1
x
y
1
1
x
y
1
1
Figure 2.5.8 Figure 2.5.16(a) Figure 2.5.16(b) Figure 2.5.16(c)
Chapter 3 Polynomials, Powers, and Exponentials
3.1
2. (a)
x 4 3 2 1 0 1 2
f (x) 2.5 0 1.5 2 1.5 0 2.5
(b) See Fig. 3.1.2(b). (c) f (x) = (1/2)(x +1)
2
+2. Maximum point (1, 2).
(d) x = 3 and x = 1 (e) f (x) > 0 in the interval (3, 1), while f (x) < 0 for x < 3 and for x > 1.
y
4
3
2
1
1
2
3
x
4 3 21 1 2 3
f (x) =
1
2
x
2
x +
3
2
Figure 3.1.2(b)
Knut Sydster and Peter Hammond 2010
CHAPT ER 3 POL YNOMI AL S, POWERS, AND EXPONENTI AL S 7
4. (a) x(x +4). Zeros 0 and 4. (b) Factorization not possible. No zeros.
(c) 3(x x
1
)(x x
2
), where the zeros are x
1
= 5 +

15 and x
2
= 5

15.
(d) 9(x x
1
)(x x
2
), where the zeros are x
1
= 1/3 +

5 and x
2
= 1/3

5.
(e) (x +300)(x 100), where the zeros are 300 and 100.
(f) (x +200)(x 100), where the zeros are 200 and 100.
6. (a) If the other side is y, then 2x +2y = L, and so y = L/2 x. The area is then A(x) = x(L/2 x) =
Lx/2 x
2
. The square with side L/4 gives the largest area.
(b) Yes; the radius is L/2 so the area is L
2
/4 > L
2
/16.
8. (a) x = 1, x = 2 (b) (i) x
2
= 9 (x
2
< 0 is impossible), so x = 3
(ii) x
3
= 1 or x
3
= 8, so x = 1 or x = 2
10. y = 2x
2
+ x 6. ((1, 3) belongs to the graph if 3 = a + b + c, (0, 6) belongs to the graph if
6 = c, and (3, 15) belongs to the graph if 15 = 9a +3b +c. It follows that a = 2, b = 1, and c = 6.)
12. (a) (i) 289 290 (ii) 361 377 (b) If B
2
4AC were > 0, then according to formula [3.2] the
equation f (x) = 0 would have two distinct solutions, which is impossible when f (x) 0 for all x. We
nd that A = a
2
1
+ a
2
2
+ + a
2
n
, B = 2(a
1
b
1
+ a
2
b
2
+ + a
n
b
n
), and C = b
2
1
+ b
2
2
+ + b
2
n
, so
the conclusion follows.
3.3
2. (a) Integer roots must divide 6. Thus 1, 2, 3, and 6 are the only possible integer solutions. We
nd that 2, 1, 1, 3 all are roots, and since there can be no more than 4 roots in a polynomial equation
of degree 4, we have found them all.
(b) The same possible integer solutions. Only 6 and 1 are integer solutions. (The third root is 1/2.)
(c) Neither 1 nor 1 satises the equation, so there are no integer roots.
(d) First multiply the equation by 4 to have integer coefcients. Then 1, 2, and 4 are seen to be the
only possible integer solutions. In fact, 1, 2, 2 are all solutions.
4. (a) The answer is 2x
2
+2x +4 +3/(x 1), because
(2x
3
+2x 1) (x 1) = 2x
2
+2x +4
2x
3
2x
2
2x
2
+2x 1
2x
2
2x
4x 1
4x 4
3 remainder
(b) The answer is x
2
+1, because
(x
4
+x
3
+x
2
+x) (x
2
+x) = x
2
+1
x
4
+x
3
x
2
+x
x
2
+x
0 no remainder
Knut Sydster and Peter Hammond 2010
8 CHAPTER 3 POL YNOMI AL S, POWERS, AND E XPONE NT I AL S
(c) The answer is 3x
5
+6x
3
3x
2
+12x 12 +(28x
2
36x +13)/(x
3
2x +1):
(3x
8
x
2
+ 1) (x
3
2x +1) = 3x
5
+6x
3
3x
2
+12x 12
3x
8
6x
6
+3x
5
6x
6
3x
5
+ x
2
+ 1
6x
6
12x
4
+ 6x
3
3x
5
+12x
4
6x
3
+ x
2
+ 1
3x
5
+ 6x
3
3x
2
12x
4
12x
3
+ 4x
2
+ 1
12x
4
24x
2
+12x
12x
3
+28x
2
12x + 1
12x
3
+24x 12
28x
2
36x +13 remainder
(d) The answer is x
3
4x
2
+3x +1 4x/(x
2
+x +1), because
(x
5
3x
4
+1) (x
2
+x +1) = x
3
4x
2
+3x +1
x
5
+ x
4
+ x
3
4x
4
x
3
+1
4x
4
4x
3
4x
2
3x
3
+4x
2
+1
3x
3
+3x
2
+3x
x
2
3x +1
x
2
+ x +1
4x remainder
6. (a) p(x) = x(x
2
+x 12) = x(x 3)(x +4), because x
2
+x 12 = 0 for x = 3 and x = 4. (b)
1, 2, 4, 8 are the only possible integer zeros. By trial and error we nd that q(2) = q(4) =
0, so 2(x 2)(x + 4) = 2x
2
+ 4x 16 is a factor for q(x). By polynomial division we nd that
q(x) (2x
2
+4x 16) = x 1/2, so q(x) = 2(x 2)(x +4)(x 1/2).
3.4
2. (a) 2.511886 (b) 0.000098 (c) 0.530094
4. 50
0.16
=
_
100

50
_
16
(Find rst the number that, when raised to the power of 100, yields 50. Then this
number is raised to the power of 16.)
6. (a) 2
3
= 8, so x = 3/2 (b)
1
81
= 3
4
, so 3x +1 = 4, and then x = 5/3.
(c) x
2
2x +2 = 2, so x
2
2x = 0, implying that x = 0 or x = 2.
8. (a) Multiply by 5K
1/2
to obtain K
1/2
= 15L
1/3
. Squaring each side, K = 225L
2/3
. (b) abx
b1
0
= p,
so x
b1
0
= p/ab. Now raise each side to the power 1/(b 1).
(c) x =
b
2a
. (Multiply each side by (ax +b)
2/3
.) (d) b =
1/
_
c

(1 )a

_
1/
10. K = 57 315.86 for Y = 100, L = 6, t = 10.
Knut Sydster and Peter Hammond 2010
CHAPT ER 3 POL YNOMI AL S, POWERS, AND EXPONENTI AL S 9
3.5
2. (a) P = 1.22 1.034
t
million (b) The doubling time t

is given by the equation (1.034)


t

= 2, and we
nd t

20.7 (years).
4. (a) 2
10
= 1024 (b) f (t ) = 2
t
(c) f (20) = (2
10
)
2
= (1024)
2
> 1000
2
= 1 million.
6. The graph is shown in Fig. 3.5.6.
x 2 1 0 1 2
2
x
2
16 2 1 2 16
y
x
1
1
y
4
3
2
1
1
2
3
4
5
6
x
2 1 1 2 3
y
2
4
6
x
5 4 3 2 1 1 2
y = x
2
2
x
Figure 3.5.6 Figure 3.5.10 Figure 3.5.12
8. If the initial time is t , the doubling time t

is given by the equation Aa


t +t

= 2Aa
t
, which implies
Aa
t
a
t

= 2Aa
t
, so a
t

= 2, independent of t .
10. (a) The graph is shown in Fig. 3.5.10.
x 3 2 1 0 1 2 3 4
1 2
x
7 3 1 0 1/2 3/4 7/8 15/16
(b) 1 2
x
gets close to 1 as x becomes very large, and becomes very large negative as x becomes large
negative. (Formally, lim
x
f (x) = 1, lim
x
f (x) = . See Section 6.1.)
12. The graph is drawn in Fig. 3.5.12.
x 10 5 4 3 2 1 0 1 2
x
2
2
x
0.1 0.8 1.0 1.1 1.0 0.5 0 2.0 16
14. I (t ) = I
0
(1/2)
t /8
remains after t days.
Knut Sydster and Peter Hammond 2010
10 CHAPTER 4 S I NGL E VARI ABL E DI F F ERENTI ATI ON
3.6
2. The function in (b) is one-to-one and has an inverse: the rule mapping each youngest child alive today to
his/her mother. (Though the youngest child of a mother with several children will have been different at
different dates.) The function in (d) is one-to-one and has an inverse: the rule mapping the surface area to
the volume. The function in (e) is one-to-one and has an inverse: the rule that maps (u, v) to (u 3, v).
The other functions are many-to-one, in general, and so have no inverses.
Chapter 4 Single Variable Differentiation
4.2
2. f

(x) = 6x +2, f

(0) = 2, f

(2) = 10, f

(3) = 20. The tangent equation is y = 2x 1.


4.
f (x +h) f (x)
h
=
1/(x +h) 1/x
h
=
x (x +h)
hx(x +h)
=
h
hx(x +h)
=
1
x(x +h)

1
x
2
as h 0,
which proves the implication.
6. (a) f (x + h) f (x) = a(x + h)
2
+ b(x + h) + c (ax
2
+ bx + c) = 2ahx + bh + ah
2
, so
[f (x +h) f (x)]/h = 2ax +b +ah 2ax +b as h 0. Thus f

(x) = 2ax +b. (b) f

(x) = 0
for x = b/2a. The tangent is parallel to the x-axis at the minimum/maximum point.
8. (a) Use [A.12] in SectionA.3. (b) and (c) [f (x +h)f (x)]/h =
_
x +h

x)/h. Multiplying both


numerator and denominator by

x +h +

x , then using the identity in (a), yields the result. Letting


h 0, the formula follows. (d)

x = x
1/2
and 1/

x = x
1/2
.
10. (a) [f (x +h) f (x)]/h = [(x +h)
1/3
x
1/3
]/h. Then use the hint.
(b) Follows from (a) by letting h 0.
4.3
2. I is the xed cost, whereas k is the marginal cost, and also the incremental cost of producing one additional
unit.
4. T

(y) = t , so the marginal tax rate is constant.


4.4
2. (a) The following table seems to indicate that the limit is 9.
x 0.9 0.99 0.999 1 1.001 1.01 1.1
x
2
+7x8
x1
8.9 8.99 8.999 9.001 9.01 9.1
*not dened
(b) x
2
+7x 8 = (x 1)(x +8), so for x = 1,
x
2
+7x 8
x 1
= x +8 9 as x 1.
4. (a) 2
2
+3 2 5 = 5 (b) 1/5 (c) 1 (d) 2 (e) 3x
2
(f) h
2
6. (a) 4 (b) 5 (c) 6 (d) 2a +2 (e) 2a +2 (f) 4a +4 8. (a) 1/6 (b) 1/27 (c) n
Knut Sydster and Peter Hammond 2010
CHAPTER 4 SI NGL E VARI ABL E DI F F ERENTI ATI ON 11
4.5
2. (a) 2g

(x) (b) (1/6)g

(x) (c) (1/3)g

(x) 4. (a) 8r (b) A(b +1)y


b
(c) (5/2)A
7/2
6. Add an arbitrary constant to these answers: (a) (1/3)x
3
(b) x
2
+3x (c)
x
a+1
a +1
4.6
2. (a) (6/5)x 14x
6
(1/2)x
1/2
(b) 4x(3x
4
x
2
1) (c) 10x
9
+5x
4
+4x
3
x
2
4. (a)
3
2

x(

x +1)
2
(b)
x
4
+5x
2
+18x +2
(x
2
+2)
2
(x +3)
2
(c) 2(1+2x)x
3
(d)
4x
(x
2
+1)
2
(e)
2x
2
+2
(x
2
x +1)
2
(f)
2(x
3
x
2
+x +1)
3x
3
(x +1)
2
6. (a) x = 2 (b) x =

3, x = 0, x =

3 (c) x =

2, x =

2
(d) x = 0, x = 1/2

5/2, x = 1/2 +

5/2
8. (a)
ad bc
(ct +d)
2
(b) a(n +1/2)t
n1/2
+nbt
n1
(c)
(2at +b)
(at
2
+bt +c)
2
10. Differentiating f (x) f (x) = x gives f

(x) f (x) + f (x) f

(x) = 1, so 2f

(x) f (x) = 1. Hence,


f

(x) =
1
2f (x)
=
1
2

x
.
12. The Newton quotient of F is
F(x +h) F(x)
h
=
f (x +h)/g(x +h) f (x)/g(x)
h
Multiplying the numerator and denominator by g(x)g(x+h) yields the expression for the Newton quotient
given in the hint. Letting h 0 yields the desired result.
4.7
2. dy/dx = ax
a1
ax
a1
and d
2
y/dx
2
= a(a 1)x
a2
+a(a +1)x
a2
4. g

(t ) = (t
2
2t )/(t 1)
2
, so g

(t ) = [(2t 2)(t 1)
2
(t
2
2t )2(t 1)]/(t 1)
4
= 2 for t = 2.
6. For n = 1, the formula is true. Suppose it is true for n = k, so that y = x
k
y
(k)
= k! . Then
(d
k+1
/dx
k+1
)x
k+1
= (d
k
/dx
k
)(d/dx)(x
k+1
) = (d
k
/dx
k
)(k+1)x
k
= (k+1)(d
k
/dx
k
)x
k
= (k+1) k!,
by the induction hypothesis. This is equal to (k +1)!, so the given formula is also true for n = k +1. By
induction, the formula is true for all n.
Knut Sydster and Peter Hammond 2010
12 CHAPTER 5 MORE ON DI F F ERENTI ATI ON
Chapter 5 More on Differentiation
5.1
2. (a) 3(2x +1)
2
2 = 6(2x +1)
2
(b) 5(1 x)
4
(c) 2(x
2
2x +2)(2x 2)
(d) (x +1)
4
(4x 1)/x
2
(e) 21(3x 4)
8
(f) 2(4x +3)(2x
2
+3x 4)
3
4. (a) 6at (at
2
+1)
4
(b) na(at +b)
n1
(c)
b(a +1)
nt
2
_
at +b
nt
_
a
6. dx/dt = Ar(Ap +B)
r1
(2at +b)
8. Puttingy = 1/(g(x))
n
implies that y

= [0(g(x))
n
1n(g(x))
n1
g

(x)]/(g(x))
2n
= n(g(x))
n1
g

(x).
10. For a = 1, the formula is true. Suppose the formula is true for a = k. Consider y =
_
g(x)
_
k+1
=
_
g(x)
_
k
g(x). The product rule for differentiation and the induction hypothesis yield
y

= k
_
g(x)
_
k1
g

(x) g(x) +
_
g(x)
_
k
g

(x) = (k +1)
_
g(x)
_
k
g

(x), which is the given formula for


a = k +1. By induction, the formula is true for all natural numbers a.
5.2
2. (a) dY/dt = (dY/dV)(dV/dt ) = (3)5(V +1)
4
t
2
= 15(t
3
/3 +1)
4
t
2
(b) dK/dt = (dK/dL)(dL/dt ) = AaL
a1
b = Aab(bt +c)
a1
4. dY/dt = (dY/dK) (dK/dt ) = Y

(K(t
0
))K

(t
0
)
6. x = b

ap c = b

u, with u = ap c. Then
dx
dp
=
1
2

u
u

=
a
2

ap c
. (The restriction
should be p > c/a, for x to be differentiable.)
8. (a) b(t ) is the total fuel consumption after t hours. (b) b

(t ) = B

_
s(t )
_
s

(t ). So the rate of fuel


consumption per hour is equal to the rate per kilometer multiplied by the speed in kph.
10. f (f (x)) = f (3x +7) = 3(3x +7) +7 = 9x +28. Then 9x +28 = 100 for x = 8.
12. dC/dx = q(25
1
2
x)
1/2
14. p

(x) = 2(x a)q(x) +(x a)


2
q

(x), so p

(a) = 0.
16. F

(x) = f

_
x
n
g(x)
__
nx
n1
g(x) +x
n
g

(x)
_
5.3
2. By implicit differentiation, 4x + 6y + 6xy

+ 2yy

= 0. So y

=
2x +3y
3x +y
. In particular, y

= 8/5
at the point (1, 2).
4. (a) 2x +2yy

= 0, and solve for y

to get y

= x/y.
(b) 1/2

x +y

/2

y = 0, and solve for y

to get y

y/x.
(c) 4x
3
4y
3
y

= 2xy
3
+x
2
3y
2
y

, and so y

= 2x(2x
2
y
3
)/y
2
(3x
2
+4y).
6. (a) No relationship. (b) f

(x
0
) = g

(x
0
)
8. Differentiatingy
n
= x
m
w.r.t. x yields ny
n1
y

= mx
m1
, soy

= mx
m1
/ny
n1
= mx
m1
/n(x
m/n
)
n1
=
(m/n)x
m1(m/n)(n1)
= (m/n)x
(m/n)1
.
Knut Sydster and Peter Hammond 2010
CHAPTE R 5 MORE ON DI F F ERENTI ATI ON 13
5.4
2. f (x) f (0) +f

(0)x =
1
9

10
27
x
4. F(1) = A, F

(K) = AK
1
, so F

(1) = A. Then F(K) F(1) +F

(1)(K 1)=
A +A(K 1) = A(1 +A(K 1)).
6. (a) (i) y = 0.61, dy = 0.6 (ii) y = 0.0601, dy = 0.06
(b) (i) y = 0.011494, dy = 0.011111 (ii) y = 0.001124, dy = 0.001111
(c) (i) y = 0.012461, dy = 0.0125 (ii) y = 0.002498, dy = 0.0025
8. g(0) = A 1 and g

() =
_
Aa/(1 + b)
_
(1 + )
[a/(1+b)]1
, so g

(0) = Aa/(1 + b). Hence, g()


g(0) +g

(0) = A 1 +
aA
1 +b

5.5
2. Follows from formula [5.9] with f = U, a = y, x = y +M s.
4. We nd x(0) = 2[x(0)]
2
= 2 1 = 2. Differentiating the expression for x(t ) yields x(t ) = x(t ) +t x(t ) +
4[x(t )] x(t ), and so x(0) = x(0)+4[x(0)] x(0) = 1+412 = 9. Hence, x(t ) x(0)+ x(0)t +
1
2
x(0)t
2
=
1 +2t +
9
2
t
2
.
6. h

(x) =
(px
p1
qx
q1
)(x
p
+x
q
) (x
p
x
q
)(px
p1
+qx
q1
)
(x
p
+x
q
)
2
=
2(p q)x
p+q1
(x
p
+x
q
)
2
, so h

(1) =
1
2
(p q). Since h(1) = 0, h(x) h(1) +h

(1)(x 1) =
1
2
(p q)(x 1).
5.6
2. El
K
T = 1.06. A 1% increase in expenditure on road building leads to an increase in the trafc volume
of approx. 1.06 %.
4. (a) El
x
f (x) = 0 (b) El
x
f (x) = x/(x +1) (c) El
x
f (x) = 20x
2
/(1 x
2
)
6. El
x
Af (x) = El
x
f (x), El
x
_
A +f (x)
_
=
f (x)El
x
f (x)
A +f (x)
8. (a) 3 (b)
1 +2x
1 +x
(c)
30x
3
x
3
+1
(d) El
x
5x
2
= 2, so El
x
El
x
5x
2
= 0 (e)
2x
2
1 +x
2
(f) El
x
_
x 1
x
5
+1
_
= El
x
(x 1) El
x
(x
5
+1) =
x El
x
x
x 1

x
5
El
x
x
5
x
5
+1
=
x
x 1

5x
5
x
5
+1
(Some answers
are easier if the results of Problem 6 are used as well.)
10. (a) 6El
x
y = 5, so El
x
y = 5/6 (b) Using rules (c) and (b) in Problem 7, we obtain El
x
y El
x
x =
El
x
(x +1)
a
+El
x
(y 1)
b
. Here El
x
x = 1, whereas
El
x
(x +1)
a
= [x/(x +1)
a
]a(x +1)
a1
= ax/(x +1)
El
x
(y 1)
b
= [x/(y 1)
b
]b(y 1)
b1
y

= [by/(y 1)]El
x
y
Thus, El
x
y 1 = ax/(x +1) +[by/(y 1)]El
x
y. Solving for El
x
y yields
El
x
y =
(ax +x +1)(y 1)
(y by 1)(x +1)
Knut Sydster and Peter Hammond 2010
14 CHAPTER 6 L I MI TS , CONTI NUI T Y, AND SERI ES
Chapter 6 Limits, Continuity, and Series
6.1
2. (a)
x 3
x
2
+1
=
(1/x) (3/x
2
)
1 +(1/x
2
)

x
(b)
_
2 +3x
x 1
=
_
3 +2/x
1 1/x

3 (c) a
2
4. lim
x
f
i
(x) = for i = 1, 2, 3; lim
x
f
4
(x) = 0. Then:
(a) (b) 0 (c) (d) 1 (e) 0 (f) (g) 1 (h)
6. y = Ax +A(b c) +d is an asymptote as x .
6.2
2. (a) None of the six functions is continuous at a. (b) Only in (i) does f have a limit: lim
xa
f (x) = A
(c) (i) lim
xa
f (x) = lim
xa
+ f (x) = lim
xa
f (x) = A,
(ii) lim
xa
f (x) = f (a), lim
xa
+ f (x) = A; (iii) lim
xa
f (x) = lim
xa
+ f (x) = ;
(iv) lim
xa
f (x) = , lim
xa
+ f (x) = f (a); (v) lim
xa
f (x) = , lim
xa
+ f (x) = f (a);
(vi) lim
xa
f (x) = A, lim
xa
+ f (x) = B. (d) Only the function in (ii) is left-continuous at x = a.
The functions in (iv), (v), and (vi) are right-continuous at x = a. (e) (v) lim
x
f (x) = A;
(vi) lim
x
f (x) does not exist.
4. (a) Continuous for all x. (b) Continuous for all x = 1. (c) Continuous for all x < 2. (d) Continuous
for all x. (e) Continuous for all x where x =

3 1 and x =

3 1. (f) Continuous for all x


where x 1 or x > 1. (g) Continuous for all x > 0. (h) Continuous for all x = 0. (i) Continuous
for all x > 0.
6. See Fig. 6.2.6; y is discontinuous at x = a. In the case where y is the nearest point on the ground, it
would be a continuous function.
y
x
a
Figure 6.2.6
8. Because lim
x0
f (x) = 2 and lim
x2
+ f (x) = 3, we need f (0) = 2 and f (2) = 3. Because f is
linear on [0, 2], put f (x) = (5/2)x 2 for x [0, 2].
6.3
2. f

(0
+
) = 1, f

(0

) = 1. At x = 0, f is continuous, but not differentiable.


Knut Sydster and Peter Hammond 2010
CHAPT ER 6 L I MI TS, CONTI NUI TY, AND SERI ES 15
4. The tax function is
t (x) =

0.15x for x [0, 20 250]


0.28x 2 632.50 for x (20 250, 49 300]
0.31x 4 111.50 for x (49 300, )
In particular, t (22 000) = 3527.50 and t (50 000) = 11388.50.
6.4
2. (a) Converges to 5. (b) Diverges (to ). (c)
3n

2n
2
1
=
3
_
2 1/n
2

n
3

2
=
3

2
2
6.5
2. (a) Geometric series with quotient 1/8. Its sum is 8/(1 1/8) = 64/7. (b) Geometric with quotient
3. It diverges. (c) Geometric, with sum 2
1/3
/(1 2
1/3
). (d) Not geometric. (One can show that
the series is convergent with sum ln 2.)
4. Geometric series with quotient (1 +p/100)
1
. Its sum is b/[1 (1 +p/100)
1
] = b(1 +100/p).
6. The general term does not approach 0 as n in any of these three cases, so each of the series is
divergent.
8. s
n
= n/(n +1) = 1/(1 +1/n) 1 as n . The innite series converges to 1.
6.6
2. Offer (a) is best. The second offer has present value 4600
1 (1.06)
5
1 (1.06)
1
20 540.
4. Schedule (b) has present value
12 000 1.115
0.115
[1 (1.115)
8
] 67 644.42.
The present value of the contract in (c) is 22, 000 +7000
(1.115)
12
1
0.115(1.115)
12
= 66, 384.08. Thus the contract
in (c) is the best in any case. When the interest rate becomes 12.5 %, contracts (b) and (c) have present
values equal to 65907.61 and 64374.33, respectively.
6. This is a geometric series with rst term a = D/(1 +r) and quotient k = (1 +g)/(1 +r). It converges
iff k < 1, i.e. iff 1 +g < 1 +r, or g < r. The sum is
a
1 k
=
D/(1 +r)
1 (1 +g)/(1 +r)
=
D
r g
.
6.7
2. |(x +1)
3
1| = |x
3
+3x
2
+3x +11| |x|
3
+3|x|
2
+3|x| |x| +3+3|x| +3|x| = 7|x|. Let > 0.
Choose = /7. Then according to [6.24], lim
x0
(x +1)
3
= 1. Because f (0) = 1, f is continuous at
x = 0.
4. (a) 4x
2
100 = 4(x
2
25) = 4(x +5)(x 5), so for x = 5,
4x
2
100
x 5
= 4(x +5). Hence, for x = 5,

4x
2
100
x 5
40

= |4(x +5) 40| = 4|x 5|. Let > 0. Choose = /4. According to [6.24], the
conclusion follows. (b) |x
2

2
| = |(x +)(x )|, so for x = , |(x
2

2
)/(x +) +2| =
|x +2| = |x +|. Let > 0. Choose = . According to [6.24], the conclusion follows.
Knut Sydster and Peter Hammond 2010
16 CHAPTER 7 I MPL I CAT I ONS OF CONTI NUI TY AND DI F F ERENT I ABI L I TY
Chapter 7 Implications of Continuity and Differentiability
7.1
2. For x = 0, we have f (x) = x
3
+ax
2
+bx +c = x
3
(1 +a/x +b/x
2
+c/x
3
), which shows that f (x)
is (large) positive if x is sufciently large, and that f (x) is (large) negative if x is sufciently large and
negative. Hence Theorem 7.2 implies that f (x) = 0 has a solution. A similar argument applies to the
general case, when the order n of the polynomial is odd. If n is even, there may be no real roots. For
instance, x
2
+1 = 0 has no real roots.
4. Recall that, arbitrarily close to any given real number, there are rational as well as irrational numbers.
The function f is continuous at a = 0, because |f (x) f (0)| = |f (x) 0| = |f (x)| |x| for any
x, so f (x) f (0) as x 0. If a = 0 is rational, then |f (x) f (a)| = |f (x)|, which is equal to |x|
when x is irrational. But if a = 0 is irrational, then |f (x) f (a)| = |f (a)| whenever x is rational. In
either case, f (x) does not approach 0 as x approaches a. It follows that f is discontinuous for all x = 0.
We prove that g is discontinuous by using the denition of continuity. Let a be an arbitrary number
and choose =
1
2
. For each positive number , there is a rational number x
1
and an irrational number x
2
in
the interval (a, a+). Theng(x
1
) = 1andg(x
2
) = 0. If a is rational, then|g(x
2
)g(a)| = |01| = 1,
and if a is irrational, then |g(x
1
) g(a)| = |1 0| = 1. In both cases there is a number x arbitrarily
close to a for which |g(x) g(a)| = 1 > . Hence g is discontinuous at a.
7.2
2. (a) No, because f (x) can get arbitrarily close to 1, by letting x be sufciently close to 1. But there is no
value of x for which f (x) = 1. Similarly, there is no minimum, because f (x) can get arbitrarily close
to 1 by letting x be sufciently close to 1, yet there is no value of x for which f (x) = 1.
(b) No, not at x = 1 and x = 1.
1
x
1
y
1
y x
23
y x 3
x
1 2 3 4 5
y
3
2
1
1
x 1
y
1 2
y
x
1
1
Figure 7.3.2(a) Figure 7.3.2(b) Figure 7.3.2(c)
7.3
2. In (a), f is not differentiable at x = 0; in (b), f is not differentiable at x = 3; in (c), f is not even dened
at x = 1. See Fig. 7.3.2(a)(c).
4. There is at least one point where you must be heading in the direction of the straight line joining A to B
(even if that straight line hits the shore).
7.4
2. (a)
3

25 = 3(1 2/27)
1/3
3(1
1
3
2
27

1
9
4
27
2
) 2.924
(b)
5

33 = 2(1 +1/32)
1/5
2(1 +
1
532

2
25
1
32
2
) 2.0125
Knut Sydster and Peter Hammond 2010
CHAP TE R 7 I MPL I CAT I ONS OF CONTI NUI TY AND DI F F E RENTI ABI L I T Y 17
4. g(0) = g(x) = 0 follows immediately from [1] and [2]. Because f is thrice continuously differentiable,
S(x) and g(x) are differentiable in (0, x). The formula for g

(t ) follows easily, and the condition g

(c) = 0
yields S(x) = f

(c). The conclusion follows.


7.5
2. (a) lim
x2
x
4
4x
3
+6x
2
8x +8
x
3
3x
2
+4
=
0
0

= lim
x2
4x
3
12x
2
+12x 8
3x
2
6x
=
0
0

=
lim
x2
12x
2
24x +12
6x 6
=
12
6
= 2
(b) lim
x0
2(1 +x)
1/2
2 x
2(1 +x +x
2
)
1/2
2 x
=
0
0

= lim
x0
(1 +x)
1/2
1
(1 +2x)(1 +x +x
2
)
1/2
1
=
0
0

=
lim
x0

1
2
(1 +x)
3/2
2(1 +x +x
2
)
1/2
+(1 +2x)
2
(
1
2
)(1 +x +x
2
)
3/2
=
1
3
4. G = lim
v0
+
1 (1 +v

v
=
0
0

= lim
v0
+
(1 +v

)
1
v
1
1
. If = 1, then
G = . If > 1, then G = 0, and if < 1, then G = .
6. lim
x
f (x)
g(x)
= lim
t 0
+
f (1/t )
g(1/t )
=
0
0

= lim
t 0
+
f

(1/t )(1/t
2
)
g

(1/t )(1/t
2
)
= lim
t 0
+
f

(1/t )
g

(1/t )
= lim
x
f

(x)
g

(x)
8.
L = lim
xa
f (x)
g(x)
= lim
xa
1/g(x)
1/f (x)
=
0
0

= lim
xa
1/(g(x))
2
1/(f (x))
2

g

(x)
f

(x)
= lim
xa
(f (x))
2
(g(x))
2

g

(x)
f

(x)
= L
2
lim
xa
g

(x)
f

(x)
= L
2
lim
xa
1
f

(x)/g

(x)
The conclusion follows. (Here, we have ignored problems with division by 0, when either f

(x) or g

(x)
tends to 0 as x tends to a.)
7.6
2. p = (157.8/D)
10/3
4. f (x) = x
2
= (x)
2
is not one-to-one on (, ), and therefore has no inverse. On [0, ), f is
strictly increasing and has the inverse g(x) =

x.
6. f

(x) = 7x
6
+25x
4
+2 > 0 for all x, so f has an inverse g. g

(2) = 1/f

(0) = 1/2.
8. (a) See Fig. 7.6.8(a). (b) Triangles OBA and OBC in Fig. 7.6.8(b) are congruent.
10. See Fig. 7.6.10. (This example shows that the commonlyseenstatement: if the inverse functionexists, the
original and the inverse function must both be monotonic is wrong. This claim is correct for continuous
functions, however.)
12. Differentiating f (g(x)) = x yields () f

_
(g(x)
_
g

(x) = 1, so g

(x) = 1/f

_
(g(x)
_
. Differentiating
() yields f

_
g(x)
_
g

(x)g

(x) +f

_
g(x)
_
g

(x) = 0. Solving for g

(x) gives
g

(x) = f

_
g(x)
__
g

(x)
_
2
/f

_
(g(x)
_
= f

_
g(x)
_
/
_
f

_
g(x)
__
3
. The conclusions follow.
Knut Sydster and Peter Hammond 2010
18 CHAPTER 8 E XPONENTI AL AND L OGARI T HMI C F UNCT I ONS
y
x
(3, 1)
(5, 3)
(1, 3)
(3, 5)
y = x
y
x
C = (b, a)
A = (a, b)
B
y = x
D
E
O
y
x
Figure 7.6.8(a) Figure 7.6.8(b) Figure 7.6.10
14. f

(x) =
1
2
(x+1)
1/2
+
1
2
(x1)
1/2
> 0for all x > 1. Hence, f is strictlyincreasingin[1, ), andhas an
inverse g. Because f (1) =

2 and f (x) as x , the range of f is [

2, ). To nd a formula
for g, let (1) y =

x +1 +

x 1. Interchanging x and y, we have (2) x =



y +1 +

y 1. The
next step is to solve (2) for y, with x [

2, ) and y [1, ). We obtain x =

y +1+

y 1
x

y +1 =

y 1 (x

y +1 )
2
= y 1 x
2
2x

y +1 + y + 1 = y 1
2x

y +1 = x
2
+2 4x
2
(y +1) = x
4
+4x
2
+4 y =
1
4
x
2
+1/x
2
.
Thus, x =

y +1 +

y 1 with y 1 implies y = x
2
/4 +1/x
2
, x

2. We prove the converse by


substitution:

y +1 +

y 1 =
_
x
2
4
+
1
x
2
+1 +
_
x
2
4
+
1
x
2
1 =
_
x
4
+4x
2
+4
4x
2
+
_
x
4
4x
2
+4
4x
2
=
x
2
+2
2x
+
x
2
2
2x
= x.
We conclude that g(x) =
1
4
x
2
+1/x
2
, x [

2, ) is the required inverse function.


Chapter 8 Exponential and Logarithmic Functions
8.1
2. (a) e
e
x
e
x
= e
e
x
+x
(b)
1
2
(e
t /2
e
t /2
) (c)
e
t
e
t
(e
t
+e
t
)
2
(d) z
2
e
z
3
(e
z
3
1)
2/3
4. p

(x) = kce
cx
, p

(x) = kc
2
e
cx
. The graph is shown in Fig. 8.1.4.
p
x
a +k
p(x) = a +k(1 e
cx
)
a
y
-3
-2
-1
1
2
3
x
-3 -2 -1 1 2 3
y =
1
2
(e
x
e
x
)
y =
1
2
(e
x
+e
x
)
Figure 8.1.4 Figure 8.1.6
Knut Sydster and Peter Hammond 2010
CHAPT E R 8 E XPONE NTI AL AND L OGARI T HMI C F UNCT I ONS 19
6. The graphs are in Fig. 8.1.6. Verifying the identities is straightforward. We prove only equality (a):
cosh x cosh y +sinh x sinh y =
1
2
(e
x
+e
x
)
1
2
(e
y
+e
y
) +
1
2
(e
x
e
x
)
1
2
(e
y
e
y
) =
1
4
(e
x+y
+e
xy
+e
x+y
+e
xy
+e
x+y
e
xy
e
x+y
+e
xy
) =
1
2
(e
x+y
+e
xy
) = cosh(x +y)
8. f (z + x) = a
z+x
= a
z
a
x
= f (z)f (x). Differentiating w.r.t. z while x is xed gives f

(z + x) =
f

(z)f (x). Putting z = 0 yields f

(x) = f

(0)f (x), which is [8.2].


8.2
2. (a) ln 3
x
= x ln 3 = ln 8, so x = ln 8/ ln 3. (b) x = e
3
(c) x
2
4x + 5 = 1, or x
2
4x + 4 = 0,
so (x 2)
2
= 0. Thus x = 2. (d) x(x 2) = 1, so x = 1

2 or x = 1 +

2. (e) x = 0 or
ln(x +3) = 0, so x = 0 or x = 2. (f)

x 5 = 1, so x = 36.
4. (a) t = (ln x b)/a (b) t = (ln 2)/a (c) t =
_
ln(8/

2 ) =
_
5
2
ln 2
1
2
ln
6. (a) True (
e
22.5, e

23.1) (b) True because (a) gives (


e
)
1/e
< (e

)
1/e
.
8. (a) Wrong. (Put A = B = C = 1.) (b) Correct by rule [8.7] (b). (c) Correct. (Use [8.7] (b) twice.)
(d) Wrong. (If A = e and p = 2, then the equality becomes 0 = ln 2.) (e) Correct by [8.7](c).
(f) Wrong. (Put A = 2, B = C = 1.)
10. (a) 1/(x +1) (b) 1/x (c) ln x +1 (d)
ln x 1
(ln x)
2
12. (a) (i) y = x 1 (ii) y = 2x 1 ln 2 (iii) y = x/e
(b) (i) y = x (ii) y = 2ex e (iii) y = e
2
x 4e
2
14. (a) Let f (x) = e
x
(1 + x + x
2
/2). Then f (0) = 0 and f

(x) = e
x
(1 + x), which is positive for
all x > 0, as shown in the problem. Hence f (x) > 0 for all x > 0, and the inequality follows. (b)
Let f (x) = ln(1 + x)
1
2
x. Then f (0) = 0 and f

(x) = 1/(x + 1)
1
2
= (1 x)/2(x + 1) > 0
in (0, 1), so ln(1 + x) >
1
2
x in (0, 1). To prove the other inequality, let g(x) = x ln(1 + x). Then
g(0) = 0 and g

(x) = 1 1/(x + 1) = x/(1 + x) > 0 in (0, 1), so x > ln(1 + x) in (0, 1). (c) Let
f (t ) = ln[(1 +t )/(1 t )] 2t . Then f (0) = 0 and f

(t ) = 2/[(1 +t )(1 t )] 2 = 2t
2
/(1 t
2
) > 0
in (0, 1), so the required inequality follows.
16. (a) x x = 0 (b) 4 ln x x (c) x
2
/y
2
18. (a) x (b) 1/ ln x (c) x ln a
20. (a) ln y =

x ln x implies y

/y =
1
2
x
1/2
ln x +x
1/2
so y

= x

x
_
ln x +2
2

x
_
(b)
1
2
(

x )
x
(ln x +1) (c) Differentiate ln ln y = x ln x +ln ln x to get
y

/(y ln y) = ln x +1 +1/(x ln x) and so y

= x
x
x
+x
_
(ln x)
2
+ln x +
1
x
_
22. First take logarithms to obtain the expression:
ln a +(v/)[ ln N + ln K ln(N

+bK

) = B (v/) ln(N

+bK

)
for ln F(), where B does not depend on . Differentiation yields:
F

() = v
2
F()
_
ln(N

+bK

)
N

ln N +bK

ln K
N

+bK

_
8.3
2. (a) x = 4 (b) x = e (c) x = 1/27 (d) x = 10
50
and x = 10
50
Knut Sydster and Peter Hammond 2010
20 CHAPTER 8 E XPONENTI AL AND L OGARI T HMI C F UNCT I ONS
4. (a) e
x+14/x
= e
1
, so x +14/x = 1, hence x = 2. (b) Putting u = ln(x +e), the equation becomes
u
3
4u
2
u + 4 = 0, so (u 4)(u
2
1) = 0, implying that u = 1 or 4. Because x = e
u
e, one
has x = 0, x = e +1/e, or x = e
4
e.
6. (a) 1 (b) Does not exist. (c) 4/5 (d) 1 (x
1/x
= (e
ln x
)
1/x
= e
ln x/x
, and [8.19].) (e) 0. (x ln x =
ln x/(1/x), and then lHpitals rule.) (f) 1. (x
x
= (e
ln x
)
x
= e
x ln x
, and then use (e).)
8. (a) xe
x
x + x
2
+
1
2
x
3
(b) e
2x
1 + 2x + 2x
2
+
4
3
x
3
(c) x
2
+ e
1
2
x

1 +
1
2
x +
9
8
x
2
+
1
48
x
3
(d)

e
x
+1

2 +
1
4

2x +
3
32

2x
2
+
1
384
(7

2)x
3
10. For x = 0, one has f

(x) = x
3
2e
1/x
2
, f

(x) = x
6
(6x
2
+ 4)e
1/x
2
, and therefore f

(x) =
x
9
(24x
4
36x
2
+ 8)e
1/x
2
. Hence, f
(k)
= x
3k
p
k
(x)e
1/x
2
is correct for k = 1, 2, 3. The general
formula is provedbyinduction. (Actually, all we needinthe followingis the fact that p
k
(x) is a polynomial.
Its degree is of no importance.) Now, f

(0) = lim
x0
[f (x) f (0)]/x = lim
x0
e
1/x
2
/x = 0
because, in general, () e
1/x
2
/x
p
0 as x 0 for any natural number p. (Substitute x = 1/

t
and use [8.18].) The general case is proved by induction: Suppose that f
(k)
(0) = 0 for an arbitrary
natural number k. Then, by the denition of the derivative, f
(k+1)
(0) = lim
x0
[f
(k)
(x) f
(k)
(0)]/x =
lim
x0
x
3k
p
k
(x)e
1/x
2
/x = lim
x0
x
3k1
p
k
(x)e
1/x
2
= 0, using () again. (Note that p
k
(x), as a
polynomial, approaches a constant as x 0.)
8.4
2. (a) S(t ) = S
0
e
at
(b) t = (ln 2)/a is the time it takes for sales to halve.
4. k = 0.1 ln(705/641) 0.0095. P(15) 739, P(40) 938.
6. (a) In 1950 there were about 276 thousand. In the next 10 years the number increased by 155 thousand.
(b) y 479.36 as t . The graph is sketched in Fig. 8.4.6.
20 15 10 5 0
(1970) (1960) (1950)
t
300
400
200
100
0
500
Tractors (in 1000)
8 6 4 2 lnx
8
4
2
6
lny
Figure 8.4.6 Figure 8.4.10
8. h

= f

K/f
2
= rf (1 f/K)K/f
2
= r(K/f 1) = rh. Hence h = Ae
rt
for some constant
A. But then 1 +K/f = Ae
rt
, and solving for f yields [8.24].
Knut Sydster and Peter Hammond 2010
CHAPTER 9 SI NGL E - VARI ABL E OPTI MI Z ATI ON 21
10. (a) See Fig. 8.4.10 and the following table:
ln x 3.00 3.69 4.09 4.38 4.61 4.79 4.94 5.08 5.19 5.30
ln y 7.44 6.31 5.44 4.79 4.32 4.09 3.81 3.56 3.22 3.00
(b) ln y = ln A+a ln x. We see fromthe graph in Fig. 8.4.10 that a 2. Assuming that the graph passes
through (say) (ln x, ln y) = (5.19, 3.22), we obtain 3.22 = ln A+(2)5.19, implying that ln A = 13.6,
so A = e
13.6
806 129. The required formula is y = 806 129x
2
.
12. (a) y = 2.81x
0.83
(b) a =
ln y
2
ln y
1
ln x
2
ln x
1
, A = y
1
x
a
1
14. Take ln of each side and solve for t
0
.
8.5
2. (a) A = 4, k = 0.25. The price after 5 years is f (5) = 4e
1.25
14. (b) Price controls are needed after
6 years. The doubling time before price controls is 2.8 years, after price controls it is 7.3 years.
Chapter 9 Single-Variable Optimization
9.2
2. h

(x) =
8(2

3x)(2 +

3x)
(3x
2
+4)
2
. Note that h(x) 0 as x . The function has a maximum at
x = 2

3/3 and a minimum at x = 2

3/3, with h(2

3/3) = 2

3/3 and h(2

3/3) = 2

3/3.
4. (a) f

(x) = [4x(x
4
+1) 2x
2
4x
3
]/(x
4
+1)
2
, then simplify and factor.
(b) f has maximum 1 at x = 1, because f (x) increases in [0, 1] and decreases in [1, ). We see that
f (x) = f (x) for all x, so f (x) is symmetric about the y-axis. On (, ), f has maximum 1 at
x = 1 and at x = 1.
6. (a) f

(x) = 48x(3x
2
+ 4)
2
, so f

(x) > 0 for x < 0 and f

(x) < 0 for x > 0. Hence, f has a


maximum at x = 0. (b) g

(x) = 2(x 2), so g

(x) > 0 for x < 2 and g

(x) < 0 for x > 2. Hence,


g has a maximum at x = 2. (c) h

(x) = 20(x + 2)
3
, so h

(x) < 0 for x < 2 and h

(x) > 0 for


x > 2. Hence, h has a minimum at x = 2. (d) F

(x) = 4x/(2 +x
2
)
2
, so F

(x) < 0 for x < 0 and


F

(x) > 0 for x > 0. Hence, F has a minimum at x = 0. (e) G

(x) = 1/2

1 x > 0 for x < 1, so G


has a maximum at x = 1. (f) H

(x) = 4x
3
/(1 +x
4
)
2
, so H

(x) > 0 for x [1, 0), and H

(x) < 0
for x (0, 1]. Hence, H has a maximum at x = 0, and minima at x = 1.
8. Here d

(x) = 2(x a
1
) +2(x a
2
) + +2(x a
n
) = 2[nx (a
1
+a
2
+ +a
n
)]. So d

(x) = 0
for x = x, where x =
1
n
(a
1
+ a
2
+ + a
n
), the arithmetic mean of a
1
, a
2
, . . . , a
n
. If x > x, then
d

(x) > 0, and if x < x, then d

(x) < 0. We conclude that x minimizes d(x).


9.3
2. In all cases the maximum and minimum exist by the extreme value theorem. Follow the recipe in [9.5].
(a) f (x) is strictly decreasing. Maximum 1 at x = 0, minimum 7 at x = 3.
Knut Sydster and Peter Hammond 2010
22 CHAPTER 9 S I NGL E- VARI ABL E OP TI MI ZATI ON
(b) Maximum 10 at x = 1 and x = 2. Minimum 6 at x = 1. (f

(x) = 3x
2
3 = 0 at x = 1.)
(c) Maximum2.5at x = 1/2 andx = 2. Minimum2at x = 1. (f (x) = x+1/x andf

(x) = 11/x
2
= 0
at x = 1.) (d) Maximum 4 at x = 1. Minimum 6

3 at

3. (f

(x) = 5x
2
(x
2
3).)
(e) Maximum 4.5 10
9
at x = 3000. Minimum 0 at x = 0.
(f

(x) = 3(x
2
3000x +2 10
6
= 3(x 500)(x 2000).)
4. (a) Total costs when there are 61, 70, and 80 passengers are: $ 48,190; $ 49,000; $ 48,000.
(b) C(x) = (60 +x)(800 10x) = 48, 000 +200x 10x
2
, x [0, 20] (c) Maximum cost is with 70
travelers (x = 10).
6. h

(t ) = 1/2

t
1
2
= (1

t )/2

t . We see that h

(t ) 0 in [0, 1] and h

(t ) 0 in [1, ), so t = 1
maximizes h(t ). The plant is highest after 1 month, when it is 1/2 meter tall.
8. Because A

(Q) = [C

(Q)QC(Q)]/Q
2
, the result follows from Example 4.20 in Section 4.6.
10. A

(Q) = a(b 1)Q


b2
c/Q
2
, so there is a minimum at Q =
_
c/a(b 1)
_
1/b
.
9.4
2. (a) No local extreme points.
(b) Local maximum 10 at x = 1. Local minimum 6 at x = 1.
(c) Local maximum 2 at x = 1. Local minimum 2 at x = 1.
(d) Local maximum 6

3 at x =

3. Local minimum 6

3 at x =

3.
(e) No local maximum point. Local minimum 0.5 at x = 3.
(f) Local maximum 2 at x = 2. Local minimum 2 at x = 0.
4. (a) (a) We nd f

(t ) = 0.05(t + 5)(35 t )e
t
. Obviously, f

(t ) > 0 for t < 35 and f

(t ) < 0 for
t > 35, so t = 35 maximizes f (with f (35) 278). (b) f (t ) 0 as t . The graph is shown in
Fig. 9.4.4.
y
t
30
100
200
60 90
y
x 1 2 3 3
5
2
2
1
5
10
2
Figure 9.4.4 Figure 9.4.8
6. (a) a > 0, b = 0 (f

(0) = 0 b = 0. f

(0) 0 a 0. If a = b = 0, then f (x) = x


3
+ c
has no loc. min. at x = 0.) (b) a = 6, b = 9. (f

(1) = 0 and f

(3) = 0 give 3 + 2a + b = 0 and


27 +6a +b = 0.)
8. (a) f

(x) =
2 x
2
(x
2
+3x +2)
2
; f

(x) =
2x
3
12x 12
(x
2
+3x +2)
3
; x =

2 is a local maximum point with
f (

2) 0.17; x =

2 is a local minimum point with f (

2) 5.83. (b) There are no global


extreme points. The graph is shown in Fig. 9.4.8.
(c) e
x
=

2 and so x =
1
2
ln 2 gives a global maximum, but g has no global minimum.
Knut Sydster and Peter Hammond 2010
CHAPTE R 10 I NTE GRATI ON 23
10. f (x) = x
3
+ax +b as x , and f (x) as x . Thus f (x) has at least one real
root. We have f

(x) = 3x
2
+a. Thus, for a 0, f

(x) > 0 for all x = 0, so f is strictly increasing, and


there is only one real root. Note that for a 0, 4a
3
+27b
2
0. Assume next that a < 0. Then f

(x) = 0
for x =

a/3 =

p, where p = a/3 > 0. Then f has a local maximum at (

p, b +2p

p)
and a local minimum at (

p, b 2p

p). If one of the local extreme values is 0, the equation has a


double root, and this is the case iff 4p
3
= b
2
, that is, iff 4a
3
+ 27b
2
= 0. The equation has three real
roots iff the local maximum value is positive and the local minimum value is negative. This occurs iff
|b| < 2p

p or iff b
2
< 4p
3
or iff 4a
3
+27b
2
< 0.
9.5
2. (a) f

(x) = 3(x 1)(x +2), f

(x) = 6x +3 (b) Stationary points: x = 2 and x = 1. f increases


in (, 2) and in (1, ). (c) x = 1/2 is the only inection point.
4. x = 2 and x = 4 are minimum points, whereas x = 2 is a maximum point. Moreover, x = 0, x = 1,
x = 3, and x = 5 are inection points.
6. a = 2/5, b = 3/5 (f (1) = 1 gives a + b = 1. Moreover, f

(x) = 3ax
2
+ 2bx and f

(x) =
6ax +2b, so f

(1/2) = 0 yields 3a +2b = 0.)


8. (Q) is stationary at Q = (P/ab)
1/(b1)
. Moreover,

(Q) = ab(b 1)Q


b2
< 0 for all Q > 0, so
this is a maximum point.
9.6
2. a 0, b arbitrary.
4. (a) Since u(c) is concave,
1
T

T
t =1
u(c
t
) u
_
1
T

T
t =1
c
t
_
u
_
1
T

T
t =1
y
t
_
= u( y), where y =
1
T

T
t =1
y
t
. Thus y is optimal. (For c
t
= y, t = 1, 2, . . . , T , this holds with equality.)
(b) Put
t
= (1 +r)
t
/

T
t =1
(1 +r)
t
. Then

T
t =1

t
= 1 and
max

T
t =1
(1 +r)
t
u(c
t
) =

T
t =1
(1 +r)
t
max

T
t =1

t
u(c
t
). Also,

T
t =1

t
u(c
t
) u
_

T
t =1

t
c
t
_

u
_

T
t =1

t
y
t
_
. So c
t
= c =

T
t =1

t
y
t
(t = 1, 2, . . . , T ) solves the problem.
Chapter 10 Integration
10.1
2. See Figs. 10.1.2(a) to 10.1.2(d). (a)
_
2
0
3x
2
dx =
2
0
x
3
= 8 (b) 1/7 (c) e 1/e (d) 9/10
x
2 1
y
2
1
x
1
y
1
x
1 1
y
2
10 1
x
y
1
Figure 10.1.2(a) Figure 10.1.2(b) Figure 10.1.2(c) Figure 10.1.2(d)
Knut Sydster and Peter Hammond 2010
24 CHAPTER 10 I NTE GRAT I ON
4. A =
1
2
_
1
1
(e
x
+e
x
) dx =
1
2
1
1
(e
x
e
x
) = e e
1
10.2
2. (a)
_
(t
3
+2t 3) dt =
_
t
3
dt +
_
2t dt
_
3 dt =
1
4
t
4
+t
2
3t +C
(b)
_
(x1)
2
dx =
_
(x
2
2x+1) dx =
1
3
x
3
x
2
+x+C. Alternative: Since
d
dx
(x1)
3
= 3(x1)
2
,
we have
_
(x 1)
2
dx =
1
3
(x 1)
3
+C
1
. This agrees with the rst answer, with C
1
= C +1/3.
(c)
_
(x 1)(x +2) dx =
_
(x
2
+x 2) dx =
1
3
x
3
+
1
2
x
2
2x +C
(d) Either rst evaluate (x +2)
3
= x
3
+6x
2
+12x +8, to get
_
(x +2)
3
dx =
1
4
x
4
+2x
3
+6x
2
+8x +C, or:
_
(x +2)
3
=
1
4
(x +2)
4
+C
1
.
(e)
_
(e
3x
e
2x
+e
x
) dx =
1
3
e
3x

1
2
e
2x
+e
x
+C
(f)
_
x
3
3x +4
x
dx =
_
_
x
2
3 +
4
x
_
dx =
1
3
x
3
3x +4 ln |x| +C
4. (a) Differentiate the right-hand side.
(b) (i)
1
10
(2x +1)
5
+C (ii)
2
3
(x +2)
3/2
+C (iii) 2

4 x +C
6. (a) F(x) =
_
(
1
2
2x) dx =
1
2
x x
2
+C. F(0) =
1
2
implies C =
1
2
.
(b) F(x) =
_
(x x
3
) dx =
1
2
x
2

1
4
x
4
+C. F(1) =
5
12
implies C =
1
6
.
8. The general form for f

is f

(x) =
1
3
x
3
+A, so that for f is f (x) =
1
12
x
4
+Ax +B. If we require that
f (0) = 1 and f

(0) = 1, then B = 1 and A = 1, so f (x) =


1
12
x
4
x +1.
10.3
2. (a) 12/5 (b) 41/2 (c)
_
3
2
_
1
t 1
+t
_
dt =
3
2
_
ln(t 1) +
1
2
t
2
_
= ln 2 +
5
2
4. (a) 6/5 (b) 26/3 (c) (e

1)/ (d) ln 2 6. (a) 32/15 2.133. See Fig. 10.3.6.


y
x
1
2
2 1 2
y
x
1
4/3
2
1 3 4 x

y =

x
Figure 10.3.6 Figure 10.3.10
8. (a) 59/30 (b) A
_
b 1 +(b c) ln

b +c
1 +c

_
+d ln |b| (c) 2

2 3/2
10. (a) Given F(x) =
_
x
a
f (t ) dt , according to the mean-value theorem, there exists a number x

(a, b)
such that [F(b) F(a)]/(b a) = F

(x

). Here F(b) =
_
b
a
f (t ) dt , F(a) = 0, and F

(x

) = f (x

),
so the conclusion follows. (b) f (x

) = 4/3 at x

= 16/9. See Fig. 10.3.10, in which the two shaded


areas are equal.
Knut Sydster and Peter Hammond 2010
CHAP TER 11 F URTHER TOPI CS I N I NTEGRAT I ON 25
10.4
2. (a) Let n be the total number of individuals. The number of individuals with income in the interval [b, 2b]
is then N = n
_
2b
b
Br
2
dr = n
2b
b
Br
1
=
nB
2b
. Their total income is M = n
_
2b
b
Br
2
r dr =
n
_
2b
b
Br
1
dr = n
2b
b
B ln r = nB ln 2. Hence the mean income is m = M/N = 2b ln 2.
(b) Total demand is x(p) =
_
2b
b
nD(p, r)f (r) dr =
_
2b
b
nAp

Br
2
dr = nABp

_
2b
b
r
2
dr =
nABp

2b
b
r
1
1
= nABp

b
1
2
1
1
1
.
4. PDV =
_
15
0
500e
0.06t
dt = 500

15
0
1
0.06
e
0.06t
=
500
0.06
_
1 e
0.9
_
4945.25.
FDV = e
0.0615
PDV = e
0.9
PDV 2.4596 4945.25 12163.3.
Chapter 11 Further Topics in Integration
11.1
2. (a)
_
1
1
x ln(x +2) dx =
1
1
1
2
x
2
ln(x +2)
_
1
1
1
2
x
2 1
x+2
dx
=
1
2
ln 3
1
2
_
1
1
_
x 2 +
4
x+2
_
dx = 2
3
2
ln 3
(b) Recall that
d
dx
2
x
= 2
x
ln 2, and therefore 2
x
/ ln 2 is the indenite integral of 2
x
. If follows that
_
2
0
x2
x
dx =
2
0
x
2
x
ln 2

_
2
0
2
x
ln 2
dx =
8
ln 2

2
0
2
x
(ln 2)
2
=
8
ln 2

3
(ln 2)
2
(c) First use integrationbyparts onthe indenite integral: withf (x) = x
2
andg(x) = e
x
, ()
_
x
2
e
x
dx =
x
2
e
x

_
2xe
x
dx. Toevaluate the last integral we must use integrationbyparts once more: withf (x) = 2x
andg(x) = e
x
,
_
2xe
x
dx = 2xe
x

_
2e
x
dx = 2xe
x
(2e
x
+C). Insertedinto() this gives
_
x
2
e
x
dx =
x
2
e
x
2xe
x
+2e
x
+C, andhence,
_
1
0
x
2
e
x
dx =
1
0
(x
2
e
x
2xe
x
+2e
x
) = (e2e+2e)(00+2) = e2.
Alternatively, more compactly using formula [11.2]:
_
1
0
x
2
e
x
dx =
1
0
x
2
e
x
2
_
1
0
xe
x
dx = e 2
_ 1
0
xe
x

_
1
0
e
x
dx
_
= e 2[e

1
0
e
x
] = e 2
4. Using integration by parts, we have
_
t
1
t
0
F(t ) (t ) dt =

t
1
t
0
F(t )(t )
_
t
1
t
0

F(t )(t ) dt
= F(t
1
)(t
1
) F(t
0
)(t
0
)
_
t
1
t
0

F(t )(t ) dt =
_
t
1
t
0

F(t )(t ) dt
6. Using [11.2], we have
_
T
0
U(C(t ))e
rt
dt =

T
0
U(C(t ))(1/r)e
rt
+
_
T
0
(d/dt )U(C(t ))(1/r)e
rt
dt
which becomes the required expression when U(C(0)) = 0.
Knut Sydster and Peter Hammond 2010
26 CHAPTER 11 F URTHER T OPI CS I N I NT EGRATI ON
11.2
2. (a)
1
9
(x
2
+ 1)
9
+ C. (Substitute u = x
2
+ 1, du = 2x dx.) (b) With u = x
3
+ 2, du = 3x
2
dx and
_
x
2
e
x
3
+2
dx =
_
1
3
e
u
du =
1
3
e
u
+ C =
1
3
e
x
3
+2
+ C. (c) First attempt: u = x + 2, which gives
du = dx and
_
ln(x +2)
2x +4
dx =
_
ln u
2u
du. Not promising. A better idea: Substitute u = ln(x + 2).
Then du =
dx
x +2
and
_
ln(x +2)
2x +4
dx =
_
1
2
u du =
1
4
(u)
2
+C =
1
4
(ln(x +2))
2
+C.
(d) Attempt: u = 1 +x. Then, du = dx, and
_
x

1 +x dx =
_
(u 1)

u du =
_
(u
3/2
u
1/2
) du =
2
5
u
5/2

2
3
u
3/2
+C =
2
5
(1 +x)
5/2

2
3
(1 +x)
3/2
+C. Second attempt: u =

1 +x. Then u
2
= 1 +x
and 2udu = dx. Then the integral is
_
x

1 +x dx =
_
(u
2
1)u2u du =
_
(2u
4
2u
3
) du e.t.c.
Check that you get the same answer. Actually, even integration by parts works in this case. Put f (x) = x
and g

(x) =

1 +x, and choose g(x) =


2
3
(1 +x)
3/2
. (The answer looks different, but is not.)
(e) With u = 1 + x
2
, x
2
= u 1, and du = 2xdx, so
_
x
3
(1 +x
2
)
3
dx =
_
x
2
x
(1 +x
2
)
3
dx =
1
2
_
u 1
u
3
du =
1
2
_
(u
2
u
3
) du =
1
2
u
1
+
1
4
u
2
+C =
1
2(1 +x
2
)
+
1
4(1 +x
2
)
2
+C.
(f) With u =

4 x
3
, u
2
= 4x
3
, and 2udu = 3x
2
dx, so
_
x
5
_
4 x
3
dx =
_
x
3
_
4 x
3
x
2
dx =
_
(4u
2
) u (
2
3
)u du =
_
(
8
3
u
2
+
2
3
u
4
) du =
8
9
u
3
+
2
15
u
5
+C =
8
9
(4x
3
)
3/2
+
2
15
(4x
3
)
5/2
+C
4. Substituting u = t
2
2t and assuming x > 2 gives
_
x
3
2t 2
t
2
2t
dt =
x
2
2x
3
ln u = ln
1
3
(x
2
2x). Thus,
the equation implies that
1
3
(x
2
2x) =
2
3
x 1. Hence, x
2
4x +3 = 0, with solutions x = 1 and x = 3.
Here x = 1 is impossible, because the integral is not dened when x = 1. So the solution is x = 3.
6. Substitute z = x(t ). Then dz = x

(t )dt , and the result follows.


8. (a) Introduce z = x as a new variable in the right-hand side integral. Then dz = dx, and
_
b+
a+
f (x
) dx =
_
b
a
f (z) dz. Then replace the dummy variable z by x in the last integral. (b) Introduce z = x/
as a new variable in the right-hand side integral.
10. Substitute u = x
1/6
. Then I = 6
_
u
8
1 u
2
du. Here u
8
: (u
2
+1) = u
6
u
4
u
2
1+1/(u
2
+1).
It follows that
I =
6
7
x
7/6

6
5
x
5/6
2x
1/2
6x
1/6
3 ln |1 x
1/6
| +3 ln |1 +x
1/6
| +C
11.3
2. (a)
_
+

f (x) dx =
_
b
a
1
b a
dx =
1
b a
b
a
x =
1
b a
(b a) = 1
(b)
_
+

xf (x) dx =
1
b a
_
b
a
x dx =
1
2(b a)
b
a
x
2
=
1
2(b a)
(b
2
a
2
) =
1
2
(a +b)
(c)
1
3(b a)
b
a
x
3
=
1
3
b
3
a
3
b a
=
1
3
(a
2
+ab +b
2
)
4. Divergence because
_
b
0
x/(1 + x
2
) dx =
b
0
1
2
ln(1 + x
2
) =
1
2
ln(1 + b
2
) as b . But
_
b
b
x/(1 +x
2
) dx =
b
b
1
2
ln(1 +x
2
) = 0 for all b, so the limit is 0.
Knut Sydster and Peter Hammond 2010
CHAPT ER 12 L I NE AR AL GE BRA: VECTORS AND MAT RI CE S 27
6.
1
1+x
2

1
x
2
for x 1, and
_
b
1
1
x
2
dx =
b
1
(1/x) = 1 1/b 1 as b , so by
Theorem 11.1 the given integral converges.
8. (a) z =
1
r
(1 e
r
) (b) z =
2
r
_
1
1
r
(1 e
r
)
_
10. Integrating by parts,
_
ln x/

x dx = 2

x ln x 4

x +C. Hence,
_
1
h
ln x/

x dx =
1
h
(2

x ln x 4

x) = 4 (2

hln h 4

h) 4 as h 0
+
, so the given
integral converges to 4. (

hln h 0 by lHpitals rule.)


12. See Fig. 11.3.12. If p > 1, then

n=1
(1/n
p
) = 1 +

n=2
(1/n
p
) is nite because

n=2
(1/n
p
) is the
sum of the shaded rectangles, and this sum is certainly less than the area under the curve y = 1/x
p
over
[1, ), which is equal to 1/(p1). If p 1, the sum

n=1
(1/n
p
) is the sum of the larger rectangles in
the gure, and this sumis larger than the area under the curve y = 1/x
p
over [1, ), which is unbounded
when p 1. Hence,

n=1
(1/n
p
) diverges in this case.
y
1
x
1 2 3 4
y = 1/x
p
Figure 11.3.12
11.4
2. The Gini coefcients are approximately: 0.34 (US 1980), 0.37 (US 1990), 0.35 (Netherlands 1959), 0.27
(Netherlands 1985), 0.69 (World 1989). (These numbers are obtained by approximating the Lorenz curve
with a broken line through the points corresponding to the observations, then computing the area between
this broken line and the diagonal of the square. Note that this underestimates the Gini coefcients.)
Chapter 12 Linear Algebra: Vectors and Matrices
12.1
2. (a) No sector delivers to itself. (b) The total amount of good i needed to produce one unit of each good.
(c) This collection gives the number of units of each good which are needed to produce one unit of good j.
(d) No economic interpretation. (The goods are usually measured in different units, so it is meaningless
to add them together.)
4. The Leontief model for this three sector model is as follows:
0.9x
1
0.2x
2
0.1x
3
= 85
0.3x
1
+0.8x
2
0.2x
3
= 95
0.2x
1
0.2x
2
+0.9x
3
= 20
which does have the claimed solution.
Knut Sydster and Peter Hammond 2010
28 CHAPTER 12 L I NE AR AL GE BRA: VE CTORS AND MATRI CES
12.2
2. a + b + c = (1, 6, 4), a 2b + 2c = (3, 10, 2), 3a + 2b 3c = (9, 6, 9), a b c =
(a +b +c) = (1, 6, 4)
4. 3(x, y, z) +5(1, 2, 3) = (3x 5, 3y +10, 3z +15) = (4, 1, 3) provided that 3x 5 = 4, 3y +10 = 1,
and 3z +15 = 3. So x = 3, y = 3, z = 4.
6. (a)
3x 2y = 1
4x +3y = 2
, x = 1, y = 2 (b)
2x +4y = 1
3x 6y = 0
, which have no solution.
8. We need to nd numbers t and s such that t (2, 1)+s(1, 4) = (4, 11). This vector equation is equivalent
to (2t + s, t + 4s) = (4, 11), which in turn is equivalent to the equation system (i) 2t + s = 4
(ii) t +4s = 11. This system has the solution t = 3, s = 2, so (4, 11) = 3(2, 1) 2(1, 4).
12.3
2. (a) = 0 gives x = (3, 1), = 1/4 gives x = (2, 5/4), = 1/2 gives x = (1, 3/2),
= 3/4 gives x = (0, 7/4), = 1 gives x = (1, 2). See Fig. 12.3.2. (b) When runs through
[0, 1], then x will run through all points on the line segment between a and b.
4. See Fig. 12.3.4. (The point R should be one unit further down.)
a
b
1/2
1/4
0
3/4
1
x
y
1 1
1
2 3
S(3, 2, 4)
x
y
Q
R
P
z
Figure 12.3.2 Figure 12.3.4
12.4
2. (a) a b = b a = 6, (a +b) c = a c +b c = 9, a (3b) = 3a b = 18.
(b) a = 3, b = 3, c =

29 , (c) |ab| = 6 ab = 9
4. The scalar product of the two vectors is x
2
+(x1)x+33x = x
2
+x
2
x+9x = 2x
2
+8x = 2x(x+4),
which is 0 for x = 0 and x = 4.
6. x = (5, 7, 12), u = (20, 18, 25), x u = 526
8. ||a + b||
2
= (a + b) (a + b) = a a + 2a b + b b ||a||
2
+ 2||a||||b|| + ||b||
2
= (||a|| + ||b||)
2
.
The conclusion follows because ||a +b|| and ||a|| +||b|| are both nonnegative.
10. (a) Total output is

x
1
x
2
x
3

8
4
4

+(1 )

2
6
10

6 +2
2 +6
6 +10

.
Knut Sydster and Peter Hammond 2010
CHAPT ER 12 L I NE AR AL GE BRA: VECTORS AND MAT RI CE S 29
To produce the output in (i) we can put = 1/2. To produce the output in (ii) would require a value of
such that 6+2 = 7, 2+6 = 5, and 6+10 = 5, and the second of these equations gives a different
value for from the other two. (b) In (a) we saw that (i) can be produced even without throwing away
outputs. For (ii) to be possible if we were allowed to throw away output, there must exist a [0, 1]
such that 6 +2 7, 2 +6 5, and 6 +10 5. These inequalities reduce to 5/6, 1/2,
5/6, which are incompatible.
(c) Revenue = R() = p
1
x
1
+ p
2
x
2
+ p
3
x
3
= p
1
(6 + 2) + p
2
(2 + 6) + p
3
(6 + 10) =
(6p
1
2p
2
6p
3
) +2p
1
+6p
2
+10p
3
. If the constant slope 6p
1
2p
2
6p
3
is > 0, then R() is
maximized at = 1; if 6p
1
2p
2
6p
3
is < 0, then R() is maximized at = 0. Only in the special
case where 6p
1
2p
2
6p
3
= 0 can the two plants both remain in use.
12.5
2. (a) Direct verication. (To showthat a lies on L, put t = 0.) (b) The direction of Lis given by (1, 2, 1).
(c) The equation of plane is (1)(x
1
2) +2(x
2
(1)) +1 (x
3
3) = 0, or x
1
+2x
2
+x
3
= 1.
(d) We must have 3(t +2) +5(2t 1) (t +3) = 6, and so t = 4/3. Thus P = (2/3, 5/3, 13/3).
4. (a) Direct verication. (b) x
1
= 2 t , x
2
= 1 +2t , x
3
= 1 +3t .
12.6
2. (a) A =
_
2 3 4
3 4 5
_
(b) A =
_
1 1 1
1 1 1
_
4. A +B =
_
1 0
7 5
_
, 3A =
_
0 3
6 9
_
12.7
2. A +B =

4 1 1
9 2 7
3 1 4

, A B =

2 3 5
1 2 3
1 1 2

, AB =

5 3 3
19 5 16
1 3 0

,
BA =

0 4 9
19 3 3
5 1 3

, (AB)C = A(BC) =

23 8 25
92 28 76
4 8 4

4. T(Ts) =

0.2875
0.2250
0.4875

12.8
2. We start by performing the multiplication

a d e
d b f
e f c

x
y
z

ax +dy +ez
dx +by +f z
ex +fy +cz

. Next,
(x, y, z)

ax +dy +ez
dx +by +f z
ex +fy +cz

= (ax
2
+by
2
+cz
2
+2dxy +2exz +2fyz)
which is a 1 1 matrix.
4. Equality in both [1] and [2] AB = BA. 6. (a) x
0
= (1/

3)

1
1
1

(b) A
n
x
0
= x
0
for all n.
Knut Sydster and Peter Hammond 2010
30 CHAPTER 13 DET ERMI NANT S AND MATRI X I NVERSI ON
8. (a) Direct verication yields (1) A
2
= (a +d)A (ad bc)I
2
=
_
a
2
+bc ab +bd
ac +cd bc +d
2
_
(b) Multiplying (1) by A and using A
3
= 0 yields (2) (a + d)A
2
= (ad bc)A, and further,
(3) 0 = (a + d)A
3
= (ad bc)A
2
. If ad bc = 0, (3) yields A
2
= 0. If ad bc = 0,
(2) yields (a + d)A
2
= 0, and if a + d = 0, again A
2
= 0. Finally, if ad bc = a + d = 0,
then (1) implies A
2
= 0. (c) Let A =
_
1 1
1 1
_
. Then A
2
= 0, and also (of course) A
3
= 0.
12.9
2. A

=
_
3 1
2 5
_
, B

=
_
0 2
2 2
_
, (A +B)

=
_
3 1
4 7
_
, (A)

=
_
6 2
4 10
_
,
AB =
_
4 10
10 8
_
, (AB)

=
_
4 10
10 8
_
= B

, A

=
_
2 4
10 14
_
4. Symmetry if a
2
1 = a +1 and a
2
+4 = 4a, which reduces to a = 2.
6. (a) (A
1
A
2
A
3
)

= (A
1
(A
2
A
3
))

= (A
2
A
3
)

1
= (A

3
A

2
)A

1
= A

3
A

2
A

1
. (b) Easy induction proof.
8. (a) tr(A +B) =

n
i=1
(a
ii
+b
ii
) =

n
i=1
a
ii
+

n
i=1
b
ii
= tr(A) +tr(B)
(b) tr(cA) =

n
i=1
ca
ii
= c

n
i=1
a
ii
= c tr(A) (c) tr(AB) =

n
i=1

n
j=1
a
ij
b
ji
, whereas tr(BA) =

n
i=1

n
j=1
b
ij
a
ji
=

n
j=1

n
i=1
b
ji
a
ij
, because the indices i and j can be interchanged. The two are
equal because the order of multiplication and summation in each double sum is irrelevant. (If this is hard
to understand, write out the sums in full for the cases when n = 2 and/or n = 3.)
(d) tr(A

) =

n
i=1
a
ii
= tr(A)
Chapter 13 Determinants and Matrix Inversion
13.1
2. See Fig. 13.1.2. The shaded area is 18 =

3 0
2 6

.
(3, 0)
(2, 6)
Figure 13.1.2
4. The matrix product is AB =
_
a
11
b
11
+a
12
b
21
a
11
b
12
+a
12
b
22
a
21
b
11
+a
22
b
21
a
21
b
12
+a
22
b
22
_
, implying that
|AB| = (a
11
b
11
+ a
12
b
21
)(a
21
b
12
+ a
22
b
22
) (a
11
b
12
+ a
12
b
22
)(a
21
b
11
+ a
22
b
21
). On the other hand,
|A||B| = (a
11
a
22
a
12
a
21
)(b
11
b
22
b
12
b
21
). A tedious computation shows that the two expressions
are equal.
Knut Sydster and Peter Hammond 2010
CHAPTER 13 DETE RMI NANTS AND MATRI X I NVERSI ON 31
6. We write the system as
_
Y C = I
0
+G
0
bY +C = a
. Then Cramers rule yields
Y =

I
0
+G
0
1
a 1

1 1
b 1

=
a +I
0
+G
0
1 b
, C =

1 I
0
+G
0
b a

1 1
b 1

=
a +b(I
0
+G
0
)
1 b
The expression for Y is most easily found by substituting the second equation into the rst, and then
solving for Y. Then use C = a +bY to nd C.
8. |A(t )| = 2t (1 t ), which is 0 for t = 0 and t = 1.
13.2
2. AB =

1 1 1
7 13 13
5 9 10

, |A| = 2, |B| = 3, |AB| = |A| |B| = 6


4. By Sarrus rule the determinant is (1 +a)(1 +b)(1 +c) +1 +1 (1 +b) (1 +a) (1 +c), which
reduces to the given expression.
6. Direct verication using [13.7] or Sarrus rule.
13.3
2. +a
12
a
23
a
35
a
41
a
54
(Four lines between pairs of elements rise as one goes to the right.)
13.4
2. A

2 1 1
1 0 2
3 1 5

, |A| = |A

| = 2
4. (a) The rst and the second columns are proportional. (b) Add the second column to the third. Then the
rst and the third columns are proportional. (c) The term x y is a common factor for each entry in
the rst row. If x = y, all elements in the rst row are 0. If x y = 0, we divide this row by x y, and
the resulting determinant has rows 1 and 2 identical, so it is 0.
6. |AB| = |A||B| = 12, 3|A| = 9, | 2B| = (2)
3
(4) = 32, |A| +|B| = 1, whereas |A +B| is not
determined.
8. (a) Because A
2
= I
n
it follows from part 8 of Theorem 13.1 that |A|
2
= 1, and so |A| = 1.
(b) Direct verication. (c) (I
n
A)(I
n
+A) = I
2
n
+I
n
A AI
n
A
2
= I
n
A
2
= 0 A
2
= I
n
Knut Sydster and Peter Hammond 2010
32 CHAPTER 13 DET ERMI NANT S AND MATRI X I NVERSI ON
10. Start by adding the sum of the last n 1 rows to the rst row. Then na + b is a common factor in the
rst row, which we take out. Next, add the rst row multiplied by a to all the other n 1 rows. The
upper triangular matrix resulting from these operations obviously has the determinant b
n1
:
D
n
=

a +b a a
a a +b a
.
.
.
.
.
.
.
.
.
.
.
.
a a a


1
.
.
.
1
=

na +b na +b na +b
a a +b a
.
.
.
.
.
.
.
.
.
.
.
.
a a a +b

= (na +b)

1 1 1
a a +b a
.
.
.
.
.
.
.
.
.
.
.
.
a a a +b

a a

.
.
.

= (na +b)

1 1 1
0 b 0
.
.
.
.
.
.
.
.
.
.
.
.
0 0 b

According to [13.14], the last determinant is b


n1
. Thus D
n
= (na +b)b
n1
.
13.5
2. In each of these cases we keep expanding by the last (remaining) column. The answers are: (a) abc
(b) abcd (c) 6 4 3 5 1 = 360
13.6
2. Multiply the two matrices to get I
3
.
4. Using
_
2 3
3 4
_
1
=
_
4 3
3 2
_
, we get: (a) x = 3, y = 1 (b) x = 1, y = 2 (c) x = y = 0
6. (a) |A| = 1, A
2
=

0 1 1
1 1 2
1 1 1

, A
3
=

1 1 2
2 2 3
1 2 2

, then direct verication of the equality.


(b) By (a), (A I)
2
A = I. (c) P = (A I)
1
=

0 0 1
1 0 1
0 1 0

does the job. So does P.


8. (a) A
2
= (PDP
1
)(PDP
1
) = PD(P
1
P)DP
1
= PDIDP
1
= P(D)
2
P
1
.
(b) Suppose the formula is valid for m = k. Then A
k+1
= AA
k
= PDP
1
(PD
k
P
1
) = PD(P
1
P)D
k
P
1
= PDID
k
P
1
= PDD
k
P
1
= PD
k+1
P
1
.
10. (a) If C
2
+ C = I, then C(C + I) = I, and so C
1
= C + I = I + C. (b) Because C
2
= I C, so
C
3
= C
2
C = (I C)C = CC
2
= C(I C) = I +2C. Moreover, C
4
= C
3
C = (I +2C)C =
C +2C
2
= C +2(I C) = 2I 3C.
12. (a) AT =
1
12

s +17 4t 16 0
2s +10 5t 8 0
3s +15 4t 16 12

. We see that AT = I
3
and so T = A
1
for s = 5, t = 4.
(b) BX = 2X +C BX 2X = C (B 2I)X = C. But it is easy to see that B 2I = A,
so BX = 2X +C AX = C X = A
1
C = TC. Hence, X =

1/2 0 0 1/6
1/2 3 3 1/6
1/2 1 2 1/6

.
Knut Sydster and Peter Hammond 2010
CHAPTE R 14 F URTHE R TOPI CS I N L I NEAR AL GEBRA 33
13.7
2.
1
|A|

C
11
C
21
C
31
C
12
C
22
C
32
C
13
C
23
C
33

=
1
72

3 5 9
18 6 18
6 14 18

. 3. (I A)
1
=
5
62

18 16 10
2 19 8
4 7 16

4. Let B denote the n p matrix whose ith column has the elements b
i1
, b
i2
, . . . , b
in
. The p systems of n
equations in n unknowns can be expressed as AX = B, where A is n n and X is n p. Following the
method in Example 13.20, exactly the same row operations that transforms the n2n matrix (A : I) into
(I : A
1
) will also transform the n (n + p) matrix (A : B) into (I : B

), where B

is the matrix with


elements b

ij
. Because these row operations are together equivalent to premultiplcation by A
1
, it must
be true that B

= A
1
B. When i = k, the solution to the system is x
1
= b

k1
, x
2
= b

k2
, . . . , x
n
= b

kn
.
13.8
2. The determinant of the system is equal to 10, so the solution is unique. The determinants in (2) are
D
1
=

b
1
1 0
b
2
1 2
b
3
3 1

, D
2
=

3 b
1
0
1 b
2
2
2 b
3
1

, D
3
=

3 1 b
1
1 1 b
2
2 3 b
3

Expanding each of these determinants by the column (b


1
, b
2
, b
3
), we nd that D
1
= 5b
1
+b
2
+2b
3
,
D
2
= 5b
1
3b
2
6b
3
, D
3
= 5b
1
7b
2
4b
3
. Hence, x
1
=
1
2
b
1

1
10
b
2

1
5
b
3
, x
2
=
1
2
b
1
+
3
10
b
2
+
3
5
b
3
,
x
3
=
1
2
b
1
+
7
10
b
2
+
2
5
b
3
.
Chapter 14 Further Topics in Linear Algebra
14.1
2. Only the vectors in (b) are linearly independent.
4. If x(1, 1, 1) +y(2, 1, 0) +z(3, 1, 4) +w(1, 2, 2) = (0, 0, 0), then x, y, z, and w satisfy the equations
x +2y +3z +w = 0, x +y +z +2w = 0, x +4z 2w = 0. One solution is x = 2, y = 1, z = 1,
w = 1, so the vectors are linearly dependent.
6. (a) Suppose a+b+ c = 0. Taking the scalar product of each side with a yields a (a+b+ c) =
a 0 = 0. But a b = a c = 0, so (a a) = 0. Because a = 0, = 0. In a similar way we can prove
that = = 0, so a, b, and c are linearly independent. (b) Generalize the proof in (a).
14.2
2. (a) By cofactor expansion along the rst row, the determinant of the matrix A is |A| = x (1) 0 1 +
(x
2
2) 1 = x
2
x 2 = (x + 1)(x 2). If x = 1 and x = 2, then |A| = 0, so the rank of A
equals 3. If x = 1 or x = 2, then |A| = 0 and r(A) 2. On the other hand, the minor we get if we
strike out the rst row and the third column in A is

0 1
1 x

= 1 = 0 for all x, so r(A) can never be


less than 2. Thus, r(A) = 2 if x = 1 or x = 2, r(A) = 3 otherwise.
(b) A little calculation shows that the determinant is t
3
+4t
2
4t 16, and if we note that this expression
has t + 4 as a factor, it follows that the determinant is t
3
+ 4t
2
4t 16 = t
2
(t + 4) 4(t + 4) =
Knut Sydster and Peter Hammond 2010
34 CHAPTER 14 F URTHER T OPI CS I N L I NEAR AL GEBRA
(t
2
4)(t + 4) = (t + 2)(t 2)(t + 4). Thus, if t is not any of the numbers 2, 2, and 4, the rank
of the matrix is 3. If we strike out the second row and the rst column of the matrix, we get the minor

5 6
1 t +4

= 5t +14, which is different from 0 for all the three special values of t that we found above,
and thus the rank of the matrix is 2 if t = 4, 2, or 2, and 3 otherwise.
(c) The rst and third rows are identical, as are the second and fourth. But the rst two rows are always
linearly independent. So the rank is 2 for all values of x, y, z, and w.
14.3
2. Unique solution for c = 3/2: x = (3 kc)/(3 2c), y = (k 2)/(3 2c). For c = 3/2, k = 2 the
solutions are x = (3/2)a +1, y = a, a arbitrary. For c = 3/2, k = 2 there are no solutions.
4. A((1 )x
1
+ x
2
) = (1 )Ax
1
+ Ax
2
= (1 )b + b = b. This shows that if x
1
and x
2
are
different solutions, then so are all points on the straight line through x
1
and x
2
.
6. (a) |A
t
| = (t 2)(t +3), so A
t
has an inverse for all t except t = 2 and t = 3.
(b) r(A
t
) = 3for t = 2 andfor t = 3. Because
_
1 3
2 5
_
is always nonsingular, r(A
2
) = 2, r(A
3
) = 2.
(c) x
1
= 46 +19t , x
2
= 19 7t , x
3
= t , with t arbitrary.
(d) Any z = 0 satisfying zA
2
= 0 has this property. In particular, z = (10a, 7a, a) for a = 0.
14.4
2. |A I| = 0 iff |(A I)

| = 0 iff |A

I| = 0 using [12.44] in Section 12.9 and Thm. 13.1 [2] in


Section 13.4. The conclusion follows.
4. (a) X

AX = (ax
2
+ay
2
+bz
2
+2axy), A
2
=

2a
2
2a
2
0
2a
2
2a
2
0
0 0 b
2

, A
3
=

4a
3
4a
3
0
4a
3
4a
3
0
0 0 b
3

(b) = 0, = 2a, = b (c) p() = (b )(2a ) =


3
+(2a +b)
2
2ab. Using A
2
and
A
3
from part (a), it is easy to show that p(A) = A
3
+(2a +b)A
2
2abA = 0.
6. Since A
1
exists, |A| = 0. So is an eigenvalue for AB |ABI| = 0 |A(BA
1
)| =
0 |BA
1
| = 0 |(BA
1
)A| = 0 |BAI| = 0 is an eigenvalue for
BA. (Using [14.11] in the next section, here is a simpler argument: AB and A
1
(AB)A = BA have the
same eigenvalues.)
14.5
2. (a) No, because the eigenvectors are not linearly independent. (b) Yes, because the eigenvectors are
linearly independent.
14.6
2. (a) |AI| =

1 2
2 2

=
2
+6 = 0, with the real eigenvalues
1
= 3 and
2
= 2. One
set of linearly independent eigenvectors is
_
1
2
_
and
_
2
1
_
. These two eigenvectors are orthogonal.
Knut Sydster and Peter Hammond 2010
CHAP TER 15 F UNCTI ONS OF SE VERAL VARI ABL ES 35
Thus, Theorem 14.7 (not 14.6!) is conrmed. Further, let
P =
_
1 2
2 1
_
for which P
1
=
_
1/5 2/5
2/5 1/5
_
It is easy to verify that P
1
AP is the diagonal matrix diag (3, 2), which conrms Theorem 14.8 (not
14.6!).
(b) Here |AI| =

0 1
0 1
1 1 1

= (1)(+2) = 0, with the real eigenvalues


1
= 2,

2
= 0 and
3
= 1. One set of linearly independent eigenvectors is

1
1
2

1
1
0

, and

1
1
1

.
Obviously, these eigenvectors are mutually orthogonal. Thus Theorem 14.7 is conrmed. Further, let
P =

1 1 1
1 1 1
2 0 1

for which P
1
=

1/6 1/6 1/3


1/2 1/2 0
1/3 1/3 1/3

It is easy to verify that P


1
AP is the diagonal matrix diag (2, 0, 2), which conrms Theorem 14.8.
Chapter 15 Functions of Several Variables
15.1
2. f (1, 1) = 2, f (2, 3) = 51, f (1/x, 1/y) = 3/x
2
2/xy + 1/y
3
, [f (x + h, y) f (x, y)]/h =
6x 2y +3h, [f (x, y +k) f (x, y)]/k = 2x +3y
2
+3yk +k
2
4. (a) F(1, 1) = 10, F(4, 27) = 60, F(9, 1/27) = 10, F(3,

2 ) = 10

3 2
1/6
,
F(100, 1000) = 1000, F(2K, 2L) = 2
5/6
F(K, L) (b) a = 5/6
6. (a) y gets 2
1.053
2.07 times larger.
(b) ln y = ln 2.9 +0.015 ln x
1
+0.25 ln x
2
+0.35 ln x
3
+0.408 ln x
4
+0.03 ln x
5
8. (a) x +y > 0 (b) Must require 2 (x
2
+y
2
) 0, i.e. x
2
+y
2
2.
(c) Put a = x
2
+y
2
. We must require (4 a)(a 1) 0, i.e. 1 a 4. (Use a sign diagram.)
10. Total output is

n
i=1
(T/t
i
) = T

n
i=1
(1/t
i
). If all n of the machines were equally efcient, and their
production per unit were

t
H
, then each machine would produce T/

t
H
units. Total output is the same as
before if T

n
i=1
(1/t
i
) = nT/

t
H
. So

t
H
= n/

n
i=1
(1/t
i
), the harmonic mean of t
1
, . . . , t
n
.
15.2
2. x
2
+y
2
= 6 is a level curve of f at height c =

64, because when x


2
+y
2
= 6, then f (x, y) =

6
6+2 =

64. In the general case, a level curve is dened by f (x, y) =


_
x
2
+y
2
(x
2
+y
2
) +2 = c
for some constant c. Let x
2
+ y
2
= u
2
, where u > 0. Then u u
2
+ 2 = c. Solving for u yields
u =
1
2

1
2

9 4c (provided that c 9/4) and thus x


2
+ y
2
is a constant, which is the equation for a
circle centered at (0, 0). If 9 4c > 0 and 1

9 4c > 0, i.e. 2 < c < 9/4, there are two circles. If


c 2 or c = 9/4, there is only one positive value of u, so the level curve is one circle.
Knut Sydster and Peter Hammond 2010
36 CHAPTER 15 F UNCTI ONS OF SE VERAL VARI ABL E S
4. Generally, the graph of g(x, y) = f (x) in 3-space consists of a surface traced out by moving the graph
of z = f (x) parallel to the y-axis in both directions. The graph of g(x, y) = x is the plane through the
y-axis at a 45

angle with the xy-plane. The graph of g(x, y) = x


3
is shown in Fig. 15.2.4. (Only a
portion of the unbounded graph is indicated, of course.)
z
y
x
Figure 15.2.4
15.3
2. (a) f

1
(x, y) = 7x
6
, f

2
(x, y) = 7y
6
, and f

12
(x, y) = 0
(b) f

1
(x, y) = 5x
4
ln y, f

2
(x, y) = x
5
/y, and f

12
(x, y) = 5x
4
/y
(c) f

1
(x, y) = 5(x
2
2y
2
)
4
2x = 10x(x
2
2y
2
)
4
, f

2
(x, y) = 5(x
2
2y
2
)
4
(4y) = 20y(x
2
2y
2
)
4
,
and f

12
(x, y) = 40x(x
2
2y
2
)
3
(4y) = 160xy(x
2
2y
2
)
3
4. (a) F

S
= 2.260.44S
0.56
E
0.48
= 0.9944S
0.56
E
0.48
, F

E
= 2.260.48S
0.44
E
0.52
= 1.0848S
0.44
E
0.52
(b) SF

S
+EF

E
= S 2.26 0.44S
0.56
E
0.48
+E 2.26 0.48S
0.44
E
0.52
= 0.44 F +0.48 F = 0.92 F,
so k = 0.92.
6. (a) z

x
= 2x, z

y
= 2e
2y
, z

xx
= 2, z

yy
= 4e
2y
, z

xy
= z

yx
= 0 (b) z

x
= y/x, z

y
= ln x, z

xx
= y/x
2
,
z

yy
= 0, z

xy
= z

yx
= 1/x (c) z

x
= y
2
ye
xy
, z

y
= 2xyxe
xy
, z

xx
= y
2
e
xy
, z

yy
= 2xx
2
e
xy
,z

xy
=
z

yx
= 2y xye
xy
e
xy
8. Here z/x = x/(x
2
+ y
2
), z/y = y/(x
2
+ y
2
),
2
z/x
2
= (y
2
x
2
)/(x
2
+ y
2
)
2
, and
2
z/y
2
=
(x
2
y
2
)/(x
2
+y
2
)
2
. Thus,
2
z/x
2
+
2
z/y
2
= 0.
10. u

x
= au/x and u

y
= bu/y, so u

xy
= au

y
/x = abu/xy. Hence, u

xy
/u

x
u

y
= 1/u (u = 0). Then,
1
u

x
_
u

xy
u

x
u

y
_
=
1
u

x
u
2
=
1
u
2
=
1
u

y
_
u

xy
u

x
u

y
_
15.4
2. F(1, 0) = F(0, 0) +
_
1
0
F

1
(x, 0) dx 2, F(2, 0) = F(1, 0) +
_
2
1
F

1
(x, 0) dx F(1, 0) +2
F(0, 1) = F(0, 0) +
_
1
0
F

2
(0, y) dy 1, F(1, 1) = F(0, 1) +
_
1
0
F

1
(x, 1) dx F(0, 1) +2
F(1, 1) = F(1, 0) +
_
1
0
F

2
(1, y) dy F(1, 0) +1
4. With u = y/x, z

x
= f (u) uf

(u) and z

y
= f

(u). The equation for the tangent plane at (x, y, z)


is Z xf (u) = [f (u) uf

(u)](X x) + f

(u)(Y y). This goes through the origin (0, 0, 0) iff


Knut Sydster and Peter Hammond 2010
CHAP TER 15 F UNCTI ONS OF SE VERAL VARI ABL ES 37
xf (u) = [f (u) uf

(u)](x) +f

(u)(y) iff 0 = xuf

(u) +f

(u)(y), which is satised because


xu = y.
15.5
2. F

1
(x, y, z) = 2xe
xz
+ x
2
ze
xz
+ y
4
e
xy
, so F

1
(1, 1, 1) = 4e; F

2
(x, y, z) = 3y
2
e
xy
+ xy
3
e
xy
, so
F

2
(1, 1, 1) = 4e; F

3
(x, y, z) = x
3
e
xz
, so F

3
(1, 1, 1) = e.
4. First-order partials: w

1
= 3yz + 2xy z
3
, w

2
= 3xz + x
2
, w

3
= 3xy 3xz
2
. Second-order partials:
w

11
= 2y, w

12
= w

21
= 3z +2x, w

13
= w

31
= 3y 3z
2
, w

22
= 0, w

23
= w

32
= 3x, w

33
= 6xz.
6. For (x, y) = (0, 0), f

1
= y(x
4
+4x
2
y
2
y
4
)(x
2
+y
2
)
2
. Thus, for y = 0, f

1
(0, y) = y. This is also
correct for y = 0, because f

1
(0, 0) = lim
h0
[f (h, 0) f (0, 0)]/h = 0. Similarly, for (x, y) = (0, 0),
f

2
= x(x
4
4x
2
y
2
y
4
)(x
2
+ y
2
)
2
, and f

2
(x, 0) = x for all x. It follows that f

12
(0, 0) = 1 and
f

21
(0, 0) = 1. Also, for (x, y) = (0, 0), f

12
= f

21
= (x
6
+ 9x
4
y
2
9x
2
y
4
y
6
)(x
2
+ y
2
)
3
. Thus,
f

12
(0, y) = 1, f

12
(x, 0) = 1, and f

12
(x, y) has no limit as (x, y) (0, 0). Similarly for f

21
.
15.6
2. (a) Y

K
= aAK
a1
and Y

K
= aBL
a1
, so KY

K
+ LY

L
= aAK
a
+ aBL
a
= a(AK
a
+ BL
a
) = aY
(b) KY

K
+LY

L
= KaAK
a1
L
b
+LAK
a
bL
b1
= aAK
a
L
b
+bAK
a
L
a
= (a +b)AK
a
L
b
= (a +b)Y
(c) Y

K
=
2aKL
5
bK
4
L
2
(aL
3
+bK
3
)
2
and Y

L
=
2bK
5
L aK
2
L
4
(aL
3
+bK
3
)
2
, so KY

K
+LY

L
=
2aK
2
L
5
bK
5
L
2
+2bK
5
L
2
aK
2
L
5
(aL
3
+bK
3
)
2
=
K
2
L
2
(aL
3
+bK
3
)
(aL
3
+bK
3
)
2
=
K
2
L
2
aL
3
+bK
3
= Y. (According to
Section 16.5 these functions are homogeneous of degrees a, a +b, and 1, respectively, so the results are
immediate consequences of Eulers Theorem.)
4. D/p and E/q are normally negative, because the demand for a commodity goes down when the
price of that commodity increases. D/q and E/p are (usually) positive, because the demand for a
commodity increases when the price of a substitute increases.
6. Y

K
= maK
1
Ae
t
_
aK

+bL

_
(m/)1
, Y

L
= mbL
1
Ae
t
_
aK

+bL

_
(m/)1
.
Thus, KY

K
+LY

L
= m(aK

+bL

)Ae
t
_
aK

+bL

_
(m/)1
= mY. (This function is homo-
geneous of degree m, so the result is an immediate consequences of Eulers Theorem.)
15.7
2. If the monopolist is not allowed to discriminate, he must charge the same price P in both markets. Thus,
from [1], P = a
1
b
1
Q
1
= a
2
b
2
Q
2
. Substituting Q
1
= (a
1
P)/b
1
and Q
2
= (a
2
P)/b
2
in the
expression for prot gives the quadratic function =
_
(a
1
b
2
+a
2
b
1
)[a
1
b
2
+a
2
b
1
+(b
1
+b
2
)]P +
(b
1
+ b
2
)P
2
_
/b
1
b
2
. This is maximized when P =
1
2
+ [a
1
b
2
+ a
2
b
1
)/2(b
1
+ b
2
)], giving maximum
prot of [a
1
b
2
+a
2
b
1
(b
1
+b
2
)]
2
/4b
1
b
2
(b
1
+b
2
). If this expression is subtracted from the (higher)
maximum prot in the example, we eventually nd that the loss of prot is (a
1
a
2
)
2
/4(b
1
+b
2
). Note
that the loss is 0 if a
1
= a
2
.
4.

L =
yy

2
xy
/
xx
Knut Sydster and Peter Hammond 2010
38 CHAPTER 16 T OOL S F OR COMP ARATI VE S TATI CS
15.8
2. (a) = 5/4, = 1/2, d = 7/8 (b) = 3/5, = 1/5, d = 26/5
4.
_
a b
b c
__
x
y
_
=
_
ax +by
bx +cy
_
, so (x, y)
_
a b
b c
__
x
y
_
= x(ax + by) + y(bx + cy), which reduces
to ax
2
+2bxy +cy
2
.
15.9
2. (a) (x, y)
_
1 1
1 1
__
x
y
_
(b) (x
1
, x
2
, x
3
)

3 1 3/2
1 1 2
3/2 2 3

x
1
x
2
x
3

4. Let
1
, . . . ,
n
be the eigenvalues of A. If A is positive semidenite and symmetric, then according to
Theorem 15.2(b), all the eigenvalues are nonnegative. Because |A| is not 0, 0 is not an eigenvalue, so all
the eigenvalues must be positive. Then according to Theorem 15.2(a), A is positive denite.
Chapter 16 Tools for Comparative Statics
16.1
2.
dz
dt
= F

1
(t, y) +F

2
(t, y)
dy
dt
=
z
t
+
z
y
dy
dt
. If z = t
2
+ye
y
and y = t
2
, then
z
t
= 2t ,
z
y
= e
y
+ye
y
,
and
dy
dt
= 2t . Hence we get
dz
dt
= 2t (1 +e
t
2
+t
2
e
t
2
).
4. The usual rules for differentiating (a) a sum; (b) a difference; (c) a product; (d) a quotient; (e) a composite
function of one variable.
6. (a) F is dened for x > 0, y > 0. F(x, y) = 0 y = 1/x
2
. See Fig. 16.1.6.
(b)
dz
dt
=
2 +e
3t
1 +e
3t
2 as t .
y
1
2
x
1 2
F(x, y) > 0
y = 1/x
2
Figure 16.1.6
8. From [16.1], dz/dt = F

1
(x, y)dx/dt + F

2
(x, y)dy/dt . Differentiating w.r.t. t yields d
2
z/dt
2
=
(d/dt )[F

1
(x, y)dx/dt ] + (d/dt )[F

2
(x, y)dy/dt ]. Here (d/dt )[F

1
(x, y)dx/dt ] = [F

11
(x, y)dx/dt +
F

12
(x, y)dy/dt ]dx/dt +F

1
(x, y)d
2
x/dt
2
. Also, (d/dt )[F

2
(x, y)dy/dt ] = [F

21
(x, y)dx/dt +F

22
(x, y)dy/dt ]dy/dt +
F

2
(x, y)d
2
y/dt
2
. Assuming F

12
= F

21
, the conclusion follows.
Knut Sydster and Peter Hammond 2010
CHAPTE R 16 TOOL S F OR COMPARATI VE S TAT I CS 39
16.2
2. z/t
1
= F

(x)x/t
1
, z/t
2
= F

(x)x/t
2
4. (a) Let u = ln v, where v = x
3
+ y
3
+ z
3
3xyz. Then u/x = (1/v)(v/x) = (3x
2
3yz)/v.
Similarly, u/y = (3y
2
3xz)/v, and u/z = (3z
2
3xy)/v. Hence,
x
u
x
+y
u
y
+z
u
z
=
x
v
(3x
2
3yz) +
y
v
(3y
2
3xz) +
z
v
(3z
2
3xy) =
3v
v
= 3
which proves (i). Equation (ii) is then proved by elementary algebra. (b) Here z/x = 2xyf

(x
2
y)
and z/y = x
2
f

(x
2
y), so xz/x = 2x
2
yf

(x
2
y) = 2yz/y.
6. F

() =
_
1
0
(/)(xe
x
2
) dx =
_
1
0
x
3
e
x
2
dx. Introduce u = x
2
as a new variable. Then the integral
is (1/2
2
)
_

0
ue
u
du = (1/2
2
)

0
(ue
u
e
u
) = (e

+ 1)/2
2
. Also, F() = (1/2)

1
0
e
x
2
=
(e

1)/2, so F

() = (e

+1)/2
2
is correct.
8. With T = T (t ),

N(t ) = n(t )e
(t T )
n(t T )e
(t T )
(1

T ) +
_
t
t T
n()()(1

T )e
(t T )
d =
[n(t ) (1

T )n(t T )]e
(t T )
(1

T )N(t ).
10. We have z(t ) =
_
2t
t
F(, t ) d, where F(, t ) = x()e

t
r(s) ds
. Leibnizs formula gives z(t ) =
2F(2t, t ) F(t, t ) +
_
2t
t
(F(, t )/t ) d = 2x(2t )e

_
2t
t
r(s) ds
x(t ) +
_
2t
t
x()e

t
r(s) ds
r(t ) d =
2x(2t )p(t ) x(t ) +
_
2t
t
F(, t )r(t ) d = 2p(t )x(2t ) x(t ) +r(t )z(t ), and therefore z(t ) r(t )z(t ) =
2p(t )x(2t ) x(t ).
12. (a) g

(Q) = c + h
_
Q
0
f (D) dD p
_
a
Q
f (D) dD and g

(Q) = (h + p)f (Q) 0 for all Q, so g


is convex. (b) g

(Q

) = 0 yields c + h
_
Q

0
f (D) dD p
_
a
Q

f (D) dD = 0. Here
_
a
Q

f (D) dD =
_
a
0
f (D) dD
_
Q

0
f (D) dD = 1 F(Q

). It follows that F(Q

) = (p c)/(h +p).
16.3
2. (a) With F(x, y) = xy, F

1
= y, F

2
= x, F

11
= F

22
= 0, and F

12
= 1. Hence, [16.9] and [16.12] yield
y

=
y
x
, y

=
1
x
3
(2 1 yx) =
2yx
x
3
=
2
x
3
(b) y

= (1 +3y)/(3x 1), y

= 6(1 +3y)/(3x 1)
2
(c) y

= 5x
4
/6y
5
, y

= (10/3)x
3
y
5
(125/36)x
8
y
11
4. (a) With F(x, y) = 2x
2
+ xy + y
2
, y

= F

1
/F

2
= (4x + y)/(x + 2y) = 4 at (2, 0). Moreover,
y

= (28x
2
+14y
2
+14xy)/(x +2y)
3
= 14 at (2, 0). The tangent has the equation y = 4x +8.
(b) y

= 0 requires y = 4x. Inserting this into the original equation gives the two points.
6. For x = 1 and y = 2 both LHS and RHS are 1, so the curve passes through (1, 2). Implicit
differentiation yields 2yy

+5 = e
x(y2)
+xe
x(y2)
(y 2 +xy

). At (1, 2) we nd y

= 4/3, so the
equation for the tangent is y =
1
3
(4x +2).
16.4
2. Let z = u
g
with u = a
1
x
d
1
+a
2
x
d
2
+a
3
x
d
3
. Then, for i = 1, 2, 3, El
i
z = El
u
u
g
El
i
u = g(x
i
/u)a
i
dx
d1
i
=
dga
i
x
d
i
/u, so El
1
z + El
2
z + El
3
z = dga
1
x
d
1
/u + dga
2
x
d
2
/u + dga
3
x
d
3
/u = dg. (This result follows
Knut Sydster and Peter Hammond 2010
40 CHAPTER 16 T OOL S F OR COMP ARATI VE S TATI CS
easily from the fact that the function is homogeneous of degree dg (see Problem 2(b) in Section 16.6)
and from the elasticity form [16.28] of the Euler equation.)
4. (a) El
x
z = 20, El
y
z = 30, El
t
x = t /(t + 1), El
t
y = 2t /(t + 1), so, according to [16.15], El
t
z =
El
x
z El
t
x +El
y
z El
t
y = 20t /(t +1) +30 2t /(t +1) = 80t /(t +1).
(b) El
t
z =
2x
2
x
2
+y
2

1
ln t
+
2y
2
x
2
+y
2
(t +2) =
2[ln t +t
4
e
2t
(2 t )]
(ln t )
2
+t
4
e
2t
6. Taking logarithms yields ln z = ax +by and a ln x +b ln y +c ln z = 0. Differentiating w.r.t. x yields
z

x
/z = a+by

x
and a/x+by

x
/y+cz

x
/z = 0. Solving for y

x
and z

x
, and then multiplying the expressions
obtained by x/y and x/z respectively, yields nally El
x
y =
a(1 +cx)
b(1 +cy)
, El
x
z =
a(x y)
1 +cy
.
8. The easiest way to solve this problem is to note that
KL
(aL
2/3
+bK
2/3
)
3/2
= KL(aL
2/3
+bK
2/3
)
3/2
= (aK
2/3
+bL
2/3
)
3/2
Comparing this with Example 16.20, we see that the production function is actually a CES-function with
A = A
0
e
0.021t
and = 2/3. According to [2] in that example,
KL
= 1/(2/3 +1) = 3/5.
10. With F(K, L) = AK
a
L
b
, F

K
= aF/K, F

L
= bF/L, F

KK
= a(a 1)F/K
2
, F

KL
= abF/KL, and
F

LL
= b(b1)F/L
2
. Also, F

K
F

L
(KF

K
+LF

L
) = (aF/K)(bF/L)(a+b)F = ab(a+b)F
3
/KL.
Moreover, KL
_
(F

L
)
2
F

KK
2F

K
F

2
F

KL
+(F

K
)
2
F

LL
_
= ab(a +b)F
3
/KL, so
KL
= 1.
16.5
2. x(tp, t r) = A(tp)
1.5
(t r)
2.08
= At
1.5
p
1.5
t
2.08
r
2.08
= t
1.5
t
2.08
Ap
1.5
r
2.08
= t
0.58
x(p, r), so the
function is homogeneous of degree 0.58 .
4. By using [16.18] we nd that f is homogeneous of degree 0, and using the results in Example 15.13(b)
in Section 15.3 we conrm that xf

1
(x, y) +yf

2
(x, y) = 0.
6. f (t x, ty) = a ln[g(t x, ty)/t x] = a ln[tg(x, y)/t x] = a ln[g(x, y)/x] = f (x, y), so f is homogeneous
of degree 0.
8. Suppose F is homogeneous of degree k = 0. Then F
1
is homogeneous of degree 1/k iff F
1
(t x) =
t
1/k
F
1
(x) for all t > 0. But this equation is equivalent to F(F
1
(t x)) = F(t
1/k
F
1
(x)). Here the
LHS is t x, and because F is homogeneous of degree 1/k, the RHS is (t
1/k
)
k
F(F
1
(x)) = t x. The
function F(x) =

x dened for x 0 with range [0, ) has an inverse F
1
= x
2
, dened for x 0.
F is homogeneous of degree 1/2, while F
1
is homogeneous of degree 2.
16.6
2. (a)
F(t x
1
, t x
2
, t x
3
) =
(t x
1
t x
2
t x
3
)
2
(t x
1
)
4
+(t x
2
)
4
+(t x
3
)
4
_
1
t x
1
+
1
t x
2
+
1
t x
3
_
=
t
6
(x
1
x
2
x
3
)
2
t
4
(x
4
1
+x
4
2
+x
4
3
)
1
t
_
1
x
1
+
1
x
2
+
1
x
3
_
= t F(x
1
, x
2
, x
3
)
The function is homogeneous of degree 1.
(b) G(t x
1
, t x
2
, t x
3
) =
_
a(t x
1
)
d
+b(t x
2
)
d
+c(t x
3
)
d
_
g
=
_
t
d
(ax
d
1
+bx
d
2
+cx
d
3
)
_
g
=
(t
d
)
g
G(x
1
, x
2
, x
3
) = t
dg
G(x
1
, x
2
, x
3
), so G is homogeneous of degree dg.
Knut Sydster and Peter Hammond 2010
CHAPTE R 16 TOOL S F OR COMPARATI VE S TAT I CS 41
4. All are homogeneous of degree 1, as is easily checked by using [16.23].
6. When F(x, y) = a ln x +b ln y, xF

1
+yF

2
= x(a/x) +y(b/y) = a +b, which is not equal to kF(x, y)
for any choice of k, so F is not homogeneous of any degree.
8. (a) f ((t x
1
)
m
, . . . , (t x
n
)
m
) = f (t
m
x
m
1
, . . . , t
m
x
m
n
) = (t
m
)
r
f (x
m
1
, . . . , x
m
n
) = t
mr
h(x
1
, . . . , x
n
), so h is
homo. of degree mr. (b) Homo. of degree sp. (c) Homo. of degree r for r = s, not homo. for r = s.
(d) Homo of degree r +s. (e) Homo. of degree r s.
16.7
2. Differentiating partially w.r.t. x yields () 3x
2
+3z
2
z

x
3z

x
= 0, so z

x
= x
2
/(1 z
2
). Similarly, or by
symmetry, z

y
= y
2
/(1 z
2
). To nd z

xy
, differentiate () partially w.r.t. y to obtain 6zz

y
z

x
+3z
2
z

xy

3z

xy
= 0, so z

xy
= 2zx
2
y
2
/(1 z
2
)
3
.
4. Implicit differentiation gives f

P
(r, P)P

w
= g

w
(w, P) +g

P
(w, P)P

w
. Hence
P

w
=
g

w
(w, P)
g

P
(w, P) f

P
(r, P)
< 0, because g

w
> 0, g

P
> 0, and f

P
< 0.
6. Differentiating the equation w.r.t. x gives 1 az

x
= f

(y bz)(bz

x
). Differentiating w.r.t. y gives
az

y
= f

(y bz)(1 bz

y
). Simple algebra yields az

x
+bz

y
= 1.
16.8
2. Write the function on the form g

(, ) = (1 +)
a
(1 +)
a
1, where a = 1/(1 +). Then
g

(, )/ = a(1 +)
a1
(1 +)
a
, g

(, )/ = (1 +)
a
a(1 +)
a1
Hence, g

(0, 0) = 0, g

(0, 0)/ = a, g

(0, 0)/ = a, and g

(, )a +a.
4. f (0.98, 1.01) 4.97. The exact value is 4.970614, so the error is 0.000614.
6. (a) and (b): dz = (y
2
+3x
2
)dx +2xy dy
8. (a) dU = 2a
1
u
1
du
1
+ +2a
n
u
n
du
n
(b) dU = A(
1
u

1
+ +
n
u

n
)
11/
(
1
u
1
1
du
1
+ +
n
u
1
n
du
n
)
10. Let T (x, y, z) = [x
2
+ y
2
+ z
2
]
1/2
= u
1/2
, where u = x
2
+ y
2
+ z
2
. Then dT =
1
2
u
1/2
du =
1
2
u
1/2
(2xdx + 2ydy + 2zdz). For x = 2, y = 3, and z = 6, we have u = 49, T = 7 and dT =
1
7
(xdx+ydy+zdz) =
1
7
(2dx+3dy+6dz). Accordingly, T (2+0.01, 30.01, 6+0.02) T (2, 3, 6)+
1
7
[2 0.01 +3(0.01) +6 0.02] = 7 +
1
7
0.11 7.016. (The exact value is

49.2206 7.015739.)
12. (a) dX = AN
1
e
t
dN +AN

e
t
dt
(b) dX
1
= BEX
E1
N
1E
dX +B(1 E)X
E
N
E
dN
14. g(0) = f (x
0
1
, . . . , x
0
n
). Using the result in part (a) of Problem 11 in Section 16.2 gives g

(0) =
f

1
(x
0
1
, . . . , x
0
n
) dx
1
+ + f

n
(x
0
1
, . . . , x
0
n
) dx
n
. The approximation g(1) g(0) + g

(0) now yields,


using vector notation, f (x
0
+dx) f (x
0
) +f

1
(x
0
) dx
1
+ +f

n
(x
0
) dx
n
.
16.9
2. By differentiation, 3 dx
1
+2x
2
dx
2
dy
1
9y
2
2
dy
2
= 0, 3x
2
1
dx
1
2dx
2
+6y
2
1
dy
1
dy
2
= 0. Letting
dx
2
= 0 and solving for dy
1
and dy
2
leads to dy
1
= Adx
1
and dy
2
= Bdx
1
, where A = y
1
/x
1
=
J
1
(3 27x
2
1
y
2
2
) and B = y
2
/x
1
= J
1
(3x
2
1
+18y
2
1
) with J = 1 +54y
2
1
y
2
2
.
Knut Sydster and Peter Hammond 2010
42 CHAPTER 17 MUL TI VARI ABL E OPTI MI ZATI ON
4. Differentiation w.r.t. x yields y + u

x
v + uv

x
= 0, u + xu

x
+ yv

x
= 0. Solving this system for u

x
and
v

x
yields
u

x
=
u
2
y
2
yv xu
=
u
2
y
2
2yv
, v

x
=
xy uv
yv xu
=
2xy 1
2yv
where we substituted xu = yv and uv = 1 xy. Now,

2
u
x
2
=

x
u

x
=
2uu

x
2yv (u
2
y
2
)2yv

x
4y
2
v
2
=
2u(u
2
y
2
) (u
2
y
2
)(2xy 1)/v
4y
2
v
2
=
(u
2
y
2
)(4uv 1)
4y
2
v
3
(The answer to this problem can be expressed in many different ways.)
6. From w = G(x, y, z) we have () (
w
x
)
y
= G

x
+G

z
(
z
x
)
y
. Here, (
w
x
)
y,z
= G

x
and (
w
z
)
x,y
= G

z
.
Substituting these expressions into () yields the result.
16.10
2. Denote the left hand sides of the three equations by f
1
, f
2
, and f
3
, respectively. Then the Jacobian
determinant of these functions w.r.t. u, v, and w is

f
1
/u f
1
/v f
1
/w
f
2
/u f
2
/v f
2
/w
f
3
/u f
3
/v f
3
/w

1 1 3w
2
1 3v
2
1
1 1 3w
2

= 2(3w
2
1)
At P the value of this determinant is 4 = 0, and the functions f
1
, f
2
, and f
3
are C
1
functions, so according
to the implicit function theorem the system denes u, v, and w as differentiable functions of x, y, and z
in a neighborhood of P. Differentiating the system w.r.t. x (holding y and z constant) yields
u

x
v

x
3w
2
w

x
= 0
u

x
+3v
2
v

x
w

x
= 2
u

x
v

x
+3w
2
w

x
= 2x
Substituting (x, y, z, u, v, w) = (1, 1, 0, 1, 0, 1) and solving the system yields u

x
= 5/2, v

x
= 1, and
w

x
= 1/2.
Chapter 17 Multivariable Optimization
17.1
2. First-order conditions: f

1
(x, y) = 4x 2y +36 = 0, f

2
(x, y) = 2x 4y +42 = 0, with the solution
x = 5, y = 8.
4. (a) P(10, 8) = P(12, 10) = 98 (b) First-order conditions: P

x
= 2x +22 = 0, P

y
= 2y +18 = 0.
It follows that x = 11 and y = 9, with P(11, 9) = 100.
6. Solving the constraint for z yields z = 4x + 2y 5, so we have to minimize P(x, y) = x
2
+ y
2
+
(4x +2y 5)
2
w.r.t. x and y. We nd P(x, y) = 17x
2
+5y
2
+16xy 40x 20y +25. The rst-order
conditions are P

1
= 34x + 16y 40 = 0, P

2
= 16x + 10y 20 = 0, with the solution x = 20/21,
y = 10/21. The minimum value is 525/441.
Knut Sydster and Peter Hammond 2010
CHAPTER 17 MUL TI VARI ABL E OP TI MI ZATI ON 43
8. F

K
= 2(K3)(L6) and F

L
= 4(L6)(K3), so here [17.3] yields 2(K3)(L6) =
0.65, 4(L 6) (K 3) = 1.2. The solution is K = 2.8, L = 5.75.
10. x = am/(a + b + c)p, y = bm/(a + b + c)q, z = cm/(a + b + c)r. (The constraint yields
z = (m px qy)/r, so we maximize P = Ax
a
y
b
[(mpx qy)/r]
c
w.r.t. x and y. Keeping in
mind that z depends on x and y, the rst-order conditions are P

x
= Ax
a1
y
b
z
c1
[az c(px/r)] = 0
and P

y
= Ax
a
y
b1
z
c1
[bz c(qy/r)] = 0. These give px = (a/c)rz and qy = (b/c)rz. Inserted into
the constraint and solved for z, these yield the given value for z, and the other two follow easily.)
17.2
2. Use Theorem 9.2 with F(u) = ln u.
4. By the chain rule, g

1
(x, y) = F

(f (x, y))f

1
(x, y) and g

2
(x, y) = F

(f (x, y))f

2
(x, y). Because F

> 0,
the conclusion follows.
6. (a) S = {(x, y) : g(x, y) = c}, where g(x, y) = x
2
+xy +y
2
, c = 3, with g continuous.
(b) The given equation is symmetric in x and y, so 2 x 2. (c) The set is closed because
g(x, y) = x
2
+ xy y
2
is continuous. The pair given in the hint does satisfy the equation, so the
conclusion follows.
17.3
2. (a) f

1
(x, y) = 3x
2
9y and f

2
(x, y) = 3y
2
9x. The only stationary point in the interior of S is (3, 3).
((0, 0) is a stationary point on the boundary of S.) Along the four parts of the boundary we nd the
following candidates for extreme points: (0, 0), (4, 0), (4,

12), (4, 4), (

12, 4), and (0, 4). Comparing


the values of f at all these points, we nd that f has mimimum 91 at (0, 4) and at (4, 0), minimum 0 at
(3, 3). (A maximum and a minimum exist by the extreme-value theorem.)
(b) Maximum 9/4 at (1/2,

3/2) and at (1/2,

3/2). Minimum 1/4 at (1/2, 0).


(c) Maximum 3 at (0, 0) and at (1, 0). Minimum 2 at (0, 1) and at (0, 1).
(d) Maximum 2e
4
at (0, 0). Minimum 0 for all (x, 1/2) where x [0, 2], and for all (2, y) where
y [0, 1/2].
4. (a) The domainDis showninFig. 17.3.4. The rst order partials are f

1
(x, y) = e

1
4
(x+y
2
)
_
1
1
4
(x +y)
_
,
f

2
(x, y) = e

1
4
(x+y
2
)
_
1
1
2
y(x +y)
_
. (b) The point (7/2, 1/2) gives the maximum value 4e
15/16
.
6. (a) f

1
= 9x(x
2
+y
2
)
1/2
4x(x
2
+y
2
)
1/2
, f

2
= 9y(x
2
+y
2
)
1/2
4y(x
2
+y
2
)
1/2
+1. At a stationary
point, f

1
= 0 implies that x = 0 or x
2
+y
2
= 4/9. But x
2
+y
2
= 4/9 implies f

2
= 1, so x = 0 at the
stationary point. Then f

2
= 9y
2
3 if y > 0, whereas f

2
= 9y
2
+ 5 if y < 0. The stationary points
are (0,
1
3

3) and (0,
1
3

5).
(b) S is closed and bounded and f is continuous, so extreme points exist according to the extreme-value
theorem. There are no stationary points in the interior of S, so the extreme points must be on the boundary.
Along x = 0, y [1, 1], f (0, y) = 3y
3
3y if y 0, and f (0, y) = 3y
3
+5y if y 0. Along this
part of the boundary f has its largest value 0 at (0, 1) and at (0, 0), and its smallest value 10

5/9 at
(0,
1
3

5). Along x
2
+y
2
= 1, y [1, 1], f (x, y) = y 1, which has its largest value 0 at (0, 1) and
its smallest value 2 at (0, 1). We conclude that the maximum value is 0 at (0, 1) and at (0, 0), and
that its smallest value is 10

5/9 at (0,
1
3

5).
Knut Sydster and Peter Hammond 2010
44 CHAPTER 17 MUL TI VARI ABL E OPTI MI ZATI ON
y
x
y = 1 x
D
x
y
z
x
y
z
z = ln(1 +x
2
y)
y
1
2
x
1 1
y = 2x
2
y = x
2
y = kx
Figure 17.3.4 Figure 17.4.2(f) Figure 17.4.8
17.4
2. (a) (0, 0) saddle point, (1, 1) local minimum. (b) (4/3, 1/3) local minimum.
(c) (2, 3) local maximum, (a, 0) and (0, b) no decision (a and b arbitrary numbers).
(d) (0, 0) saddle point, (a, 0) and (a, 0) local maxima where a =

3/2.
(e) (0, 0) saddle point, (1/2, 1/4) and (1/2, 1/4) local mininma.
(f) (0, b) no decision (b an arbitrary number). (The graph of this function is shown in Fig. 17.4.2(f). f
is dened for x = 0 and for y > 1/x
2
. f

1
(x, y) = 2xy/(1 + x
2
y) and f

2
(x, y) = x
2
/(1 + x
2
y).
Here f

1
= f

2
= 0 at all points (0, b) with b . Because AC B
2
= 0 when (x, y) = (0, b), the
second-derivative test fails. But by studying the function directly, we see that (0, b) is a local maximum
point if b < 0; a saddle point if b = 0; and a local minimum point if b > 0. See Fig. 17.4.2(f).)
4. (a) The level curves are x = 1 and y = 0. g(x, y) is positive if y > 0 and x = 1.
(b) g

1
= y(x
2
1)e
x+3y
, g

2
= (x 1)
2
(1+3y)e
x+3y
. (1, 1/3) is a local minimum. (1, y
0
) is a local
minimum for y
0
> 0, a local maximum for y
0
< 0, and a saddle point for y
0
= 0. (Use the results in (a).)
(c) g(x, 1) = (x 1)
2
e
x+3
as x , so g has no maximum; g(x, 1) = (x 1)
2
e
x3

as x , so g has no minimum either.


6. We nd easily in all three cases that (0, 0) is a stationary point where z = 0 and AC B
2
= 0. In case
(a), z 0 for all (x, y), so the origin is a maximum point. In case (b), z 0 for all (x, y), so the origin
is a minimum point. In (c), z takes positive and negative values at points arbitrarily close to the origin,
so it is a saddle point.
8. (a) Fig. 17.4.8. The domain where f (x, y) is negative is shaded. (b) (0, 0) is seen to be a stationary
point at which z = 0. As Fig. 17.4.8 shows, this function takes positive and negative values for points
arbitrary close to (0, 0), so the origin is a saddle point. (c) g(t ) = f (t h, t k) = (t kt
2
h
2
)(t k2t
2
h
2
) =
2h
4
t
4
3h
2
kt
3
+ k
2
t
2
, so g

(t ) = 8h
4
t
3
9h
2
kt
2
+ 2k
2
t and g

(t ) = 24h
4
t
2
18h
2
kt + 2k
2
. Then
g

(0) = 0 and g

(0) = 2k
2
, so t = 0 is a minimum point for k = 0. For k = 0, g(t ) = 2t
4
h
4
, which has
a minimum at t = 0.
17.5
2. (a) The set is convex; see Fig. 17.5.2(a). (b) The set is convex; see Fig. 17.5.2(b).
(c) The set is not convex; see Fig. 17.5.2(c). (d) The set is convex; see Fig. 17.5.2(d).
(e) The set is not convex; see Fig. 17.5.2(e). (f) The set is not convex; see Fig. 17.5.2(f). ((4, 0) and
(0, 4) belong to the set, but not
1
2
(4, 0) +
1
2
(0, 4) = (2, 2).)
Knut Sydster and Peter Hammond 2010
CHAPTER 17 MUL TI VARI ABL E OP TI MI ZATI ON 45
y
-3
-2
-1
1
2
3
x
-3 -2 -1 1 2 3
y
x
y
-4
-3
-2
-1
1
2
3
4
x
-4 -3 -2 -1 1 2 3 4
Figure 17.5.2(a) Convex. Figure 17.5.2(b) Convex. Figure 17.5.2(c) Not convex.
y
-1
1
2
3
x
-1 1 2 3
y
-3
-2
-1
1
2
3
x
-3 -2 -1 1 2 3
y
1
2
3
4
x
1 2 3 4
Figure 17.5.2(d) Convex. Figure 17.5.2(e) Not convex. Figure 17.5.2(f) Not convex.
4. To prove that aS + bT is convex, take x aS + bT , y aS + bT , and a number [0, 1]. Because
x aS + bT , there are points x
1
S and y
1
T such that x = ax
1
+ by
1
. Similarly, there are points
x
2
S and y
2
T such that y = ax
2
+by
2
. Then (1)x+y = (1)(ax
1
+by
1
) +(ax
2
+by
2
) =
a[(1 )x
1
+ x
2
] + b[(1 )y
1
+ y
2
]. This belongs to aS + bT because S and T are convex, so
(1 )x
1
+x
2
S and (1 )y
1
+y
2
T . Therefore aS +bT is convex.
17.6
2. If f is linear, then f is both concave and convex according to Example 17.12. On the other hand, if f is
both concave and convex on the convex set S, then [17.15] is satised with equality for any (0, 1)
and for all x, x
0
in S. Thus the graph of f contains any line segment between two points on its graph. So
the graph of f is a hyperplane, and f is linear.
4. We prove the rst statement in [17.16]. The second statement is proved in the same way. Suppose rst
that f is concave. Take any two points (x
1
, y
1
) and (x
2
, y
2
) in M
f
. Then y
1
f (x
1
) and y
2
f (x
2
). Let
[0, 1]. Then (1)(x
1
, y
1
)+(x
2
, y
2
) = ((1)x
1
+x
2
, (1)y
1
+y
2
). Here (1)x
1
+x
2
S,
because S is convex. Moreover, by the concavity of f , f ((1 )x
1
+x
2
) (1 )f (x
1
) +f (x
2
)
(1)y
1
+y
2
. This inequality implies that (1)(x
1
, y
1
) +(x
2
, y
2
) belongs to M
f
, so M
f
is convex.
Suppose on the other hand that M
f
is convex. Take any x
1
, x
2
in S and let [0, 1]. Of course,
f (x
1
) f (x
1
) and f (x
2
) f (x
2
), so (x
1
, f (x
1
)) and (x
2
, f (x
2
)) both belong to M
f
. Because M
f
is
convex, (1 )(x
1
, f (x
1
)) + (x
2
, f (x
2
)) = ((1 )x
1
+ x
2
, (1 )f (x
1
) + f (x
2
)) M
f
. This
implies that (1 )f (x
1
) +f (x
2
) f ((1 )x
1
+x
2
), and so f is concave.
Knut Sydster and Peter Hammond 2010
46 CHAPTER 17 MUL TI VARI ABL E OPTI MI ZATI ON
17.7
2. Part (a) remains valid iff either a or b is strictly positive. This follows immediately from the arguments
in the proof of part (a) of Theorem 17.6. (Of course, if a = b = 0, af + bg is certainly not strictly
concave.) It follows from the proof of part (c) that if F is strictly concave, then the inequality
F
_
f ((1 )x
0
+x)
_
F
_
(1 )f (x
0
) +f (x)
_
becomes strict for x
0
= x, so U is strictly concave.
There is no need to assume that f is strictly concave. For part (e), suppose f and g are strictly concave
over the convex set S. Let (0, 1) and let x = y. Then, because f and g are strictly concave,
f ((1 )x +y) > (1 )f (x) +f (y), and g((1 )x +y) > (1 )g(x) +g(y). It follows that
h((1 )x +y) = min{f
_
(1 )x +(y)
_
, g
_
(1 )x +y
_
}
> min{(1 )f (x) +f (y), (1 )g(x) +g(y)}
But
(1 )f (x) +f (y) (1 ) min{f (x), g(x)} + min{f (y), g(y)}
= (1 )h(x) +h(y)
Similarly, (1)g(x)+g(y) (1)h(x)+h(y). Therefore, h((1)x+y) > (1)h(x)+h(y).
17.8
2. (a) f

11
= 2a, f

12
= 2b, f

22
= 2c, and f

11
f

22
(f

12
)
2
= 2a2c (2b)
2
= 4(ac b
2
). The result follows
from Theorem 17.10. (b) According to Theorem 17.9, f is concave iff a 0, b 0, and ac b
2
0;
f is convex iff a 0, b 0, and ac b
2
0.
4. (a) f

11
(x, y) = e
x
e
x+y
, f

12
(x, y) = e
x+y
, and f

22
(x, y) = e
x+y
, so f

11
(x, y) < 0 and
f

11
f

22
(f

12
)
2
= e
2x+y
> 0. Hence, f is strictly concave.
(b) g is strictly convex. (g

11
= e
x+y
+e
xy
> 0 and g

11
g

22
(g

12
)
2
= 4e
2x
> 0.)
6. f is concave where f

11
= 2 6x 0, f

22
= 2 0, and f

11
f

22
(f

12
)
2
= 2(2 6x) 1 0.
Here 2 6x 0 iff x 1/3 and 2(2 6x) 1 0 iff x 5/12. We conclude that the largest convex
domain on which f is concave is S = {(x, y) : x 5/12}.
8. f

1
= 4x +4, f

2
= 2y +4, f

11
= 4, f

12
= 0, f

22
= 2. Clearly (1, 2) is a stationary point for f
and the conditions in Theorem 17.11(a) are satised, so (1, 2) is the maximum point.
10. (a) The rms choose pand q to maximize px+qy(5+x)(3+2y) = 26p+24q5p
2
6q
2
+8pq69.
This is a strictly concave function. The maximum is given by p = 9, q = 8, x = 16, y = 4. As prot is
123, whereas Bs is 21. (b) FirmAs prot is now
A
(p) = px 5 x = p(29 5p+4q) 5 29 +
5p 4q = 34p 5p
2
+4pq 4q 34, with q xed. This quadratic polynomial is maximized at p =
p
A
(q) =
1
5
(2q+17). Likewise, rmBs prot is now
B
(q) = qy32y = 28q6q
2
+4pq8p35,
with p xed. This quadratic polynomial is maximized at q = q
B
(p) =
1
3
(p +7).
(c) Equilibrium occurs where p = p
A
(q) and q = q
B
(p). So 5p = 2q +17 and 3q = p+7. These give
p = 5, q = 4, x = 20, y = 12. A gets 75, B gets 21.
(d) The process starts at (9, 8) and converges to (5, 4). See Fig. 17.8.10.
Knut Sydster and Peter Hammond 2010
CHAPTER 17 MUL TI VARI ABL E OP TI MI ZATI ON 47
q
2
4
6
8
10
p
2 4 6 8 10 12 14
(5, 4)
q = q
B
(p)
p = p
A
(q)
(9, 8)
Figure 17.8.10
17.9
2. (a) Stationary points must satisfy the three equations: f

1
= 2x +2y +2z = 0, f

2
= 4y +2x = 0, and
f

3
= 6z +2x = 0. The only solution is (0, 0, 0). The Hessian matrix is

11
f

12
f

13
f

21
f

22
f

23
f

31
f

32
f

33

2 2 2
2 4 0
2 0 6

The leading principal minors are 2,

2 2
2 4

= 4,

2 2 2
2 4 0
2 0 6

= 8. So (0, 0, 0) is a local minimum point.


(b) f has two stationary points, (3, 3, 3) and (
1
3
y
2
0
, y
0
, y
0
), where y
0
1.63 is the other real root (apart
from y = 3) of the equation y
4
54y +81 = 0. (There are two complex roots.) The Hessian matrix is
H =

6x 9 9
9 6y 0
9 0 6z

The leading principal minors areD


1
= 6x, D
2
=

6x 9
9 6y

= 36xy 81, D
3
= |H| =
54(4xyz 9y 9z). At (3, 3, 3) the leading principal minors are all positive, so this is a local min-
imumpoint. At (
1
3
y
2
0
, y
0
, y
0
), D
1
> 0, D
2
< 0, and D
3
< 0. According to [17.27], this is a saddle point.
(c) The stationary points are the solutions of the equation system
f

1
= 8x
2
8x
1
= 0, f

2
= 20 +8x
1
12x
2
2
= 0, f

3
= 48 24x
3
= 0, f

4
= 6 2x
4
The rst equation gives x
2
= x
1
, and then the second equation gives 12x
2
1
8x
1
20 = 0 with the two
solutions x
1
= 5/3 and x
1
= 1. The last two equations determine x
3
and x
4
. There are two stationary
points, (5/3, 5/3, 2, 3) and (1, 1, 2, 3). The Hessian matrix is
f

(x
1
, x
2
, x
3
, x
4
) =

8 8 0 0
8 24x
2
0 0
0 0 24 0
0 0 0 2

and the leading principal minors of the Hessian are D


1
= 8, D
2
= 192x
2
64 = 64(3x
2
1), D
3
= 24D
2
,
and D
4
= 48D
2
. At (1, 1, 2, 3) we get D
2
< 0, so this point is a saddle point. The other stationary point,
(5/3, 5/3, 2, 3), we get D
1
< 0, D
2
> 0, D
3
< 0, and D
4
> 0, so this point is a local maximum point.
Knut Sydster and Peter Hammond 2010
48 CHAPTER 17 MUL TI VARI ABL E OPTI MI ZATI ON
17.10
2. If f is increasing, or decreasing, then the upper level sets {x : f (x) a} are all intervals, which are
convex sets, so f is quasi-concave.
4. Let x, x
0
be any pair of vectors in S, and [0, 1]. Let z = (1 )x + x
0
. Now, if f (z)
min{f (x), f (x
0
)}, then f (x) f (x
0
) implies f (z) f (x
0
), as in [17.31]. Hence, Theorem 17.15
implies that f is quasi-concave.
Conversely, suppose f is quasi-concave. If f (x) f (x
0
), then [17.31] implies f (z) f (x
0
). On
the other hand, if f (x) f (x
0
), then interchanging x and x
0
in [17.31] and replacing by 1 implies
that f (z) f (x). In either case, f (z) min{f (x), f (x
0
)}.
6. (a) This is mainly an exercise in manipulating determinants. If you feel that the calculations below look
frightening, try to write them out in full for the case k = 3 (or k = 2). Note that z

ij
= a
i
a
j
z/x
i
x
j
for
i = j, and z

ii
= a
i
(a
i
1)z/x
2
i
. Rule 3 in Theorem 13.1 tells us that a common factor in any column
(or row) in a determinant can be moved outside. Therefore,
D
k
=

11
z

12
. . . z

1k
z

21
z

22
. . . z

2k
.
.
.
.
.
.
.
.
.
.
.
.
z

k1
z

k2
. . . z

kk

a
1
(a
1
1)
x
2
1
z
a
1
a
2
x
1
x
2
z
a
1
a
k
x
1
x
k
z
a
2
a
1
x
2
x
1
z
a
2
(a
2
1)
x
2
2
z
a
2
a
k
x
2
x
k
z
.
.
.
.
.
.
.
.
.
.
.
.
a
k
a
1
x
k
x
1
z
a
k
a
2
x
k
x
2
z
a
k
(a
k
1)
x
2
k
z

(1)
=
a
1
a
2
. . . a
k
x
1
x
2
. . . x
k
z
k

a
1
1
x
1
a
1
x
1

a
1
x
1
a
2
x
2
a
2
1
x
2

a
2
x
2
.
.
.
.
.
.
.
.
.
.
.
.
a
k
x
k
a
k
x
k

a
k
1
x
k

(2)
=
a
1
a
2
. . . a
k
(x
1
x
2
. . . x
k
)
2
z
k

a
1
1 a
1
a
1
a
2
a
2
1 a
2
.
.
.
.
.
.
.
.
.
.
.
.
a
k
a
k
a
k
1

where equality (1) holds because a


j
z/x
j
is a common factor in column j for each j and equality (2) holds
because 1/x
i
is a common factor in row i for each i.
(b) Let s
k
=

k
i=1
a
i
= a
1
+ + a
k
, and P
k
=
a
1
a
2
. . . a
k
(x
1
x
2
. . . x
k
)
2
z
k
. We use the expression for D
k
that
we found in part (a), and add rows 2, 3, . . . , k to the rst row. Then each entry in the rst row becomes
equal to s
k
1. Afterwards we take the common factor s
k
1 in row 1 and move it outside.
D
k
= P
k

s
k
1 s
k
1 s
k
1
a
2
a
2
1 a
2
.
.
.
.
.
.
.
.
.
.
.
.
a
k
a
k
a
k
1

= (s
k
1)P
k

1 1 1
a
2
a
2
1 a
2
.
.
.
.
.
.
.
.
.
.
.
.
a
k
a
k
a
k
1

Nowsubtract column 1 fromall the other columns. Rule 7 in Theorem13.1 says that this does not change
the value of the determinant, so D
k
= (s
k
1)P
k

1 0 0
a
2
1 0
.
.
.
.
.
.
.
.
.
.
.
.
a
k
0 1

= (1)
k1
(s
k
1)P
k
.
Knut Sydster and Peter Hammond 2010
CHAPT ER 18 CONST RAI NED OP TI MI ZATI ON 49
(c) By assumption, a
i
> 0 for all i, so if

n
i=1
a
i
< 1, then s
k
=

k
i=1
a
i
< 1 for all k. Therefore D
k
has
the same sign as (1)
k
. CombiningTheorem15.3 andTheorem17.14, it follows that f is strictly concave.
Chapter 18 Constrained Optimization
18.1
2. (a) When f (x, y) = 3xy, g(x, y) = x
2
+ y
2
, and c = 8, equation [18.3] reduces to 3y/3x = 2x/2y,
and so x
2
= y
2
. Inserted into the constraint this yields x
2
= 4, and so x = 2. The solution candidates
are therefore: (2, 2), (2, 2), (2, 2), (2, 2). Here f (2, 2) = f (2, 2) = 12 and f (2, 2) =
f (2, 2) = 12. So (2, 2) and (2, 2) are the only possible solutions of the maximization problem,
and (2, 2) and (2, 2) are the only possible solutions of the minimization problem. (The extreme-value
theorem ensures that we have found the solutions, because f is continuous and the constraint curve is a
closed bounded set (a circle).)
(b) With L = x + y (x
2
+ 3xy + 3y
2
3), the rst-order conditions are 1 2x 3y = 0
and 1 3x 6y = 0. From these equations we get 2x + 3y = 3x + 6y, or (3y + x) = 0.
Here = 0 is impossible, so x = 3y. Inserted into the constraint we have (3, 1) and (3, 1) as the
only possible solutions of the maximization and minimization problems, respectively. The extreme value
theorem ensures that solutions exist. (The objective function is continuous and the constraint curve is a
closed bounded set (an ellipse).)
18.2
2. (a) x = a/5, y = 2a/5 solve the problem. Solving x + 2y = a for y yields y =
1
2
a
1
2
x, and then
x
2
+y
2
= x
2
+(
1
2
a
1
2
x)
2
=
5
4
x
2

1
2
ax +
1
4
a
2
. This quadratic function certainly has a minimum at
x = a/5. (b) L(x, y) = x
2
+ y
2
(x + 2y a). The necessary conditions are L

1
= 2x = 0,
L

2
= 2y 2 = 0, implying that 2x = y. From the constraint, x = a/5 and then y = 2a/5, = 2a/5.
(c) Findthe point onthe line x+2y = a whichhas the minimal distance fromthe origin. The corresponding
maximization problem has no solution. (See Fig. 18.2.2.)
(d) f

(a) = (a/5)
2
+(2a/5)
2
= a
2
/5, so df

(a)/da = 2a/5, which is also the value of the Lagrangean


multiplier. Equation [18.7] is conrmed.
y
x
(a/5, 2a/5)
a
a/2
Figure 18.2.2
4. L(2, 2) = 2 > L(1, 1) = 1. (In fact, L(x, y) has a saddle point at (1, 1).)
Knut Sydster and Peter Hammond 2010
50 CHAPTER 18 CONS TRAI NED OPTI MI ZATI ON
6. (a) L(x, y) = 100e
x
e
y
(px+qym), so the rst-order conditions are: (1) L

x
= e
x
p = 0
and (2) L

y
= e
y
q = 0. These imply that x = ln(p) = ln +ln p, y = ln +ln q. Inserting
the expressions for x and y into the constraint, and solving for ln , yields
ln = (m+p ln p +q ln q)/(p +q). Therefore x(p, q, m) =
m+q ln q q ln p
p +q
and
y(p, q, m) =
m+p ln p p ln q
p +q
.
(b) x(tp, t q, t m) =
t m+t q ln(t q) t q ln(tp)
tp +t q
=
t m+t q(ln t +ln q) t q(ln t +ln p)
t (p +q)
= x(p, q, m), so x is homogeneous of degree 0. The constraint px +qy = m remains unchanged when
x, y, m are all multiplied by t , so the maximization of U is over the same constraint set in both cases.
Hence, the optimal values of x and y have to be unchanged.
8. (a) g(x, y) =
_
y
0
xt (y t ) dt =
_
y
0
(xyt xt
2
) dt =

y
0
(
1
2
xyt
2

1
3
xt
3
) dt =
1
6
xy
3
. Similar computations
yield f (x, y) =
1
6
xy
5
+
1
12
xy
4
+
1
6
xy
3
. The solution of [] is x = 384
3
M, y = 1/4. (Note that,
because
1
6
xy
3
= M, the problem reduces to that of maximizing M +
1
2
My My
2
for y 0.)
(b) As tends to 0, the cost of extraction tends to innity. Because the sales price increases as t increases,
it is reasonable to expect that the terminal time y tends to innity. (c) The value function is f

(M) =
M+M/16. The Lagrange multiplier must satisfy the equation f

1
g

1
= 0, or
1
6
y
5
+
1
12
y
4
+
1
6
y
3

1
6
y
3
= 0. Inserting the optimal values of x and y and solving for yields = 1 + 1/16. Clearly,
f

(M)/M = .
18.3
2. (a) (i) The constraint implies that x = y
2
/8 0, so the problem reduces to that of minimizing h(x) =
(x 1)
2
+8x for x 0. Because h

(x) = 2x +6 6 for all x 0, the minimum value is 1 at x = 0.


(ii) The problem reduces to minimizing k(y) = (
1
8
y
2
1)
2
+y
2
. Here k

(y) = 2(
1
8
y
2
1)
1
4
y +2y =
y(
1
16
y
2
+
3
2
). Thus k

(y) < 0 for y < 0, and k

(y) > 0 for y > 0. We conclude that k(y) is minimized


at y = 0, which gives the same solution as in (i).
(b) For the LagrangeanL = (x1)
2
+y
2
(y
2
8x), the rst-order conditions are: (1) 2(x1)+8 = 0;
(2) 2y 2y = 0. From (2), either = 1 or y = 0. But inserting = 1 into (1) yields x = 3, and then
no value of y satises the constraint. So y = 0, implying the solution x = y = 0.
(c) The problem is to nd which point on the parabola y
2
= 8x is closest to (1, 0).
18.4
2. In this case, f (x, y) = x
2
+y
2
, g(x, y) = x +2y, so
D(x, y) =

0 g

1
g

2
g

1
f

11
g

11
f

12
g

12
g

2
f

21
g

21
f

22
g

22

0 1 2
1 2 0
2 0 2

= 10
By [18.14], there is a local minimum at the stationary point (a/5, 2a/5).
18.5
2. Here L = x + 4y + 3z (x
2
+ 2y
2
+
1
3
z
2
b). So necessary conditions are:
(1) L

1
= 1 2x = 0; (2) L

2
= 4 4y = 0; (3) L

3
= 3
2
3
z = 0. It follows that = 0,
Knut Sydster and Peter Hammond 2010
CHAPT ER 18 CONST RAI NED OP TI MI ZATI ON 51
and so x = 1/2, y = 1/, z = 9/2. Inserting these values into the constraint yields
2
= 9/b, so
= 3/

b. The value of the criterion function is x +4y +3z = 18/, so = 3/

b minimizes the
criterion function. The minimum point is (a, 2a, 9a), where a =

b/6.
4. With x =
16
15
+ h, y =
1
3
+ k, and z =
11
15
+ , constraints [1] and [2] reduce to h + 2k + l = 0 and
2h k 3 = 0, so k = h and = h. But then easy algebra shows that f (x, y, z) = (16/15)
2
+
(1/3)
2
+(11/15)
2
+3h
2
. Because 3h
2
0 for all h, (16/15)
2
+(1/3)
2
+(11/15)
2
must then be the
maximum value of f (x, y, z) subject to the constraints. The maximum value is obviously attained when
h = k = = 0, or when (x, y, z) = (16/15, 1/3, 11/15).
6. (a) Here L = x + y + z (x
2
+ y
2
+ z
2
1) (x y z 1). Necessary conditions are:
(1) L

1
= 1 2x = 0; (2) L

2
= 1 2y + = 0; (3) L

3
= 1 2z + = 0. Adding (1) and
(2) yields (x + y) = 1. Adding (1) and (3) yields (x + z) = 1. It follows that = 0 and also that
z = y. Substituting z = y in the two constraints yields: (4) x
2
+ 2y
2
= 1; (5) x 2y = 1. From (5),
x = 2y +1, so (4) gives 6y
2
+4y = 0, with solutions y = 0 and y = 2/3. The only possible solutions
to the problem are therefore (1, 0, 0) and (1/3, 2/3, 2/3). In fact, (1, 0, 0) solves the maximization
problem, while (1/3, 2/3, 2/3) solves the minimization problem.
(b) The constraints are represented geometrically by a closed bounded curve which is the intersection
between a sphere (x
2
+y
2
+z
2
= 1) and a plane (x y z = 1). The continuous function f (x, y, z) =
x +y +z must therefore attain maximum and minimum values subject to the constraints, and in (a) we
have found the extreme points.
8. Here L = a
2
1
x
2
1
+ +a
2
n
x
2
n
(x
1
+ +x
n
1). Necessary conditions are that L

j
= 2a
2
j
x
j
= 0,
j = 1, . . . , n, and so x
j
= /2a
2
j
. Inserted into the constraint, this implies that 1 =
1
2
(1/a
2
1
+ +1/a
2
n
).
Thus, for j = 1, . . . , n, we have
x
j
=
1
a
2
j
(1/a
2
1
+ +1/a
2
n
)
=
1
a
2
j

n
i=1
(1/a
2
i
)
(Implicitly, this solution assumes that each a
i
= 0. If at least one a
i
is 0, the minimum value is 0, which
is attained by letting a corresponding x
i
be 1, with the other x
j
all equal to 0.)
10. (a) The Lagrangean is here L = Ax
a
1
1
x
a
n
n
(p
1
x
1
+ + p
n
x
n
m), and so
L

i
= a
i
U/x
i
p
i
= 0, i = 1, . . . , n. In particular, p
1
x
1
= a
1
U/, and so = a
1
U/p
1
x
1
. It
follows that p
i
x
i
= (a
i
/a
1
)p
1
x
1
. Inserted into the budget constraint, we have p
1
x
1
+ (a
2
/a
1
)p
1
x
1
+
+ (a
n
/a
1
)p
1
x
1
= m, which implies that x
1
=
a
1
m
p
1
(a
1
+ +a
n
)
. The demand functions are given
by x
i
=
a
i
m
p
i
(a
1
+ +a
n
)
for i = 1, . . . , n.
(b) The Lagrangean is L = x
a
1
+ +x
a
n
(p
1
x
1
+ +p
n
x
n
m), and so L

i
= ax
a1
i
p
i
= 0.
In particular, = ax
a1
1
/p
1
. Algebraic manipulations yield p
i
x
i
= p
a/(1a)
i
p
1/(1a)
1
x
1
, i = 1, . . . , n.
Using the budget constraint equation gives an expression for x
1
, and subsequently we nd the other
demand quantities x
i
. The consumers demand functions are x
i
=
mp
1/(1a)
i
n

j=1
p
a/(1a)
j
, for i = 1, . . . , n.
Knut Sydster and Peter Hammond 2010
52 CHAPTER 18 CONS TRAI NED OPTI MI ZATI ON
12. For n = 2, the Lagrangean is L = a
11
x
2
1
+2a
12
x
1
x
2
+a
22
x
2
2
(x
2
1
+x
2
2
1). The rst-order conditions
for x
1
= x
0
1
, x
2
= x
0
2
, with =
0
to solve the problem are:
()
2a
11
x
0
1
+2a
12
x
0
2
2
0
x
0
1
= 0
2a
12
x
0
1
+2a
22
x
0
2
2
0
x
0
2
= 0
Cancelling the common factor 2 and writing () as a matrix equation yields
_
a
11
a
12
a
21
a
22
__
x
0
1
x
0
2
_
=
0
_
x
0
1
x
0
2
_
or, in obvious matrix notation, () Ax
0
=
0
x
0
. Thus, if x
0
solves the problem, then x
0
is an eigenvector
of A, with
0
as an eigenvalue. Multiplying () from the left by x

0
= (x
0
1
, x
0
2
) then implies that
x

0
Ax
0
= x

0
(
0
x
0
) =
0
x

0
x
0
=
0
, because x

0
x
0
= 1. We conclude that the maximum value of x

Ax
subject to x

x = 1 must be the largest eigenvalue of A, whereas the minimum value must be the smallest
eigenvalue. Both a maximum and a minimum value do exist because of the extreme-value theorem.
The argument in the general case of n variables is entirely similar. In fact, equating the partials of
the Lagrangean to 0 leads to a system of equations like (), which is then easily seen to reduce to ()
18.6
2. (a) The Lagrangean is L = 4z x
2
y
2
z
2
(z xy). The rst-order conditions are 2x +y = 0,
2y +x = 0, 4 2z = 0, and z = xy. The only triples (x, y, z) that satisfy these conditions are
(0, 0, 0) with = 4, (1, 1, 1) with = 2, and (1, 1, 1) with = 2. (b) f

0.1 = 20.1 = 0.2


18.7
2. If L(x, c) = f (x)

m
j=1

j
(g
j
(x) c
j
), then L/c
j
=
j
, so [18.24] follows from [18.31].
18.8
2. (a) Writing the constraints as g
1
(x, y) = x + e
x
y 0 and g
2
(x, y) = x 0, the Lagrangean is
L =
1
2
x y
1
(x +e
x
y)
2
(x). The KuhnTucker conditions are then:
L
x
=
1
2

1
(1 e
x
) +
2
= 0 (1)
L
y
= 1 +
1
= 0 (2)

1
0 (= 0 if x +e
x
< y) (3)

2
0 (= 0 if x > 0) (4)
From (2) and (3), we conclude that
1
= 1 and so x +e
x
= y.
Either x = 0 or x > 0. In the latter case, (4) implies
2
= 0. Then (1) implies
1
2
(1 e
x
) = 0,
or e
x
=
1
2
. Hence x = ln 2, and so y = x +e
x
= ln 2 +
1
2
.
If x = 0, then (1) implies
2
=
1
2
, which contradicts
2
0.
Knut Sydster and Peter Hammond 2010
CHAPT ER 18 CONST RAI NED OP TI MI ZATI ON 53
We conclude that (x, y) = (ln 2, ln 2 +
1
2
) is the only point satisfying the KuhnTucker conditions.
(By sketching the constraint set and studying the level curves
1
2
x y = c, it is easy to see that the point
we found solves the problem.)
(b) With the Lagrangean L = x
2
+2y
1
(x
2
+y
2
)
2
(y), the KuhnTucker conditions are:
L
x
= 2x 2
1
x = 0 (1)
L
y
= 2 2
1
y +
2
= 0 (2)

1
0 (= 0 if x
2
+y
2
< 5) (3)

2
0 (= 0 if y > 0) (4)
Note that the second constraint cannot be active, because if y = 0, then (2) implies
2
= 2, which
contradicts (4). So y > 0, and from (4),
2
= 0. Then from (2),
1
y = 1. Hence
1
> 0, and so from
(3), x
2
+y
2
= 5. Also, (1) evidently implies that x = 0 or
1
= 1.
If x = 0, then x
2
+ y
2
= 5 implies y =

5. With y > 0, only y =



5 is possible. Then (2)
implies that
1
= 1/

5. Thus (x, y,
1
,
2
) = (0,

5, 1/

5, 0) satises (1) to (4) and is a solution


candidate.
If
1
= 1, then
1
y = 1 implies y = 1 and fromx
2
+y
2
= 5 it follows that x = 2. Thus (2, 1, 1, 0)
and (2, 1, 1, 0) are both solution candidates.
Now, f (0,

5) = 2

5, while f (2, 1) = f (2, 1) = 6. The constraint set is closed and bounded


(as a half-disk), so the extreme-value theorem applies. Thus, (2, 1) and (2, 1) both solve the problem,
with
1
= 1 and
2
= 0.
4. (a) The Lagrangean is L = x + ay (x
2
+ y
2
1) + (x + y). There must exist numbers and
such that: (1) 1 2x + = 0; (2) a 2y + = 0; (3) 0 (= 0 if x
2
+y
2
< 1);
(4) 0 (= 0 if x +y > 0).
(b) For a 1, the point (1/

1 +a
2
, a/

1 +a
2
) solves the problem, with =
1
2

1 +a
2
, = 0.
For a < 1, the point (1/

2, 1/

2) solves the problem, with = (1 a)/2

2, =
1
2
(1 +a).
6. The Lagrangean is L = ln(x
2
+2y)
1
2
x
2
y
1
(2xy) +
2
(x 1) +
3
(y 1). The KuhnTucker
conditions are that there must exist numbers
1
0,
2
0, and
3
0 such that
L

1
(x, y) =
2x
x
2
+2y
x +
1
y +
2
= 0 (1)
L

2
(x, y) =
2
x
2
+2y
1 +
1
x +
3
= 0 (2)

1
(2 xy) = 0 (3)

2
(1 x) = 0 (4)

3
(1 y) = 0 (5)
(We have deliberately formulated the conditions in a slightly different way than was done for Problems
2 and 4. See [18.37] in the main text.)
Knut Sydster and Peter Hammond 2010
54 CHAPTER 18 CONS TRAI NED OPTI MI ZATI ON
Suppose rst that
1
= 0. Then (1) and (2) imply
2x
x
2
+2y
x +
2
= 0,
2
x
2
+2y
1 +
3
= 0
Because x and y are both 1, we get
3
= 1 2/(x
2
+2y) 1
2
3
> 0 and
2
= x 2x/(x
2
+2y) =
x
3
> 0. It follows from (4) and (5) that x = y = 1, which contradicts xy 2. Thus
1
> 0 and so
xy = 2.
Suppose next that
2
> 0. Then x = 1 (because of (4)) and y = 2/x = 2, so
3
= 0. From (2) it
follows that
1
= 3/5. Then (1) implies that
2
= 3/5, contradicting
2
0. Hence,
2
= 0.
Suppose now that
3
> 0. Then (5) implies y = 1, and so x = 2/y = 2. From (1) it follows that

1
= 4/3. Then, from (2),
3
= 1/3 +1 8/3 < 0, which is impossible. Thus
3
= 0.
So far we have proved that
2
=
3
= 0,
1
> 0, and xy = 2. Equations (1) and (2) now yield
2x
x
2
+2y
= x
1
y,
2
x
2
+2y
= 1
1
x
It follows that x
1
y = x(1
1
x). But
1
> 0 and so y = x
2
. Because xy = 2, we must have x
3
= 2.
Hence, x = 2
1/3
and y = 2x
1
= 2
2/3
is the only point which satises the KuhnTucker conditions.
(The corresponding nonnegative multipliers are
1
= 2
1/3

1
3
,
2
=
3
= 0.)
Why is this a maximum? In the feasible set, x
2
+2y 3, and f

1
(x, y) and f

2
(x, y) are both negative.
For a given value of x, f (x, y) decreases as y increases, and for a given value of y, f (x, y) decreases as
x increases. It follows that the constrained maximum lies on the part of the curve xy = 2 between the
two points (1, 2) and (2, 1). (See Fig. 18.8.4.) This is a closed and bounded set, so the extreme-value
theorem implies that f (x, y) has a maximum on this part of the curve. This point must be a maximum
point for f over the whole feasible set.
y
x
(1, 1)
y
1
2
3
4
x
1 2 3 4
Figure 18.8.4
18.9
2. The Lagrangean is L = 9x + 8y 6(x + y)
2

1
(x 5)
2
(y 3)
3
(x + 2y 2) and the
KuhnTucker conditions are:
L

1
= 9 12(x +y)
1
+
3
0 (= 0 if x > 0) (1)
Knut Sydster and Peter Hammond 2010
CHAPT ER 18 CONST RAI NED OP TI MI ZATI ON 55
L

2
= 8 12(x +y)
2
2
3
0 (= 0 if y > 0) (2)

1
0 (= 0 if x < 5) (3)

2
0 (= 0 if y < 3) (4)

3
0 (= 0 if x +2y < 2) (5)
We go through the systematic procedure explained at the end of Section 18.8. (Shortcuts are possible.
For instance, one could start by proving that x = 5 is impossible, then that y cannot be positive, etc.)
x < 5, y < 3, x +2y < 2. Then
1
=
2
=
3
= 0. If x = 0, then from (1), y 3/4 > 0, so from
(2), 812y = 0. Thus y = 2/3, which contradicts y 3/4. So x > 0. Then (1) implies 9 = 12(x +y).
If y > 0, then (2) implies 8 = 12(x +y), a contradiction. Thus y = 0. But then 9 = 12x, so x = 3/4.
So a solution candidate is (x, y) = (3/4, 0), with
1
=
2
=
3
= 0.
x = 5, y < 3, x +2y < 2. Then
2
=
3
= 0 and (1) implies
1
= 912(5+y) < 0, a contradiction.
x < 5, y = 3, x +2y < 2. Then
1
=
3
= 0 and (2) implies
2
= 812(x +3) < 0, a contradiction.
x < 5, y < 3, x +2y = 2. Then
1
=
2
= 0 and 2y = 2 + x > 0, so y > 0 and (2) imply
8 12(x +y) = 2
3
. It follows that
3
= 4 6[x +
1
2
(2 +x)] = 2 9x < 0, a contradiction.
x < 5, y = 3, x +2y = 2. Then x = 4 and
1
= 0. Now (1) gives
3
= 75, and then (2) gives

2
= 226, contradicting
2
0.
x = 5, y < 3, x +2y = 2. Then the rst and second equalities yield y = 3.5, which contradicts y < 3.
x = 5, y = 3, x +2y < 2. Then
3
= 0. Now (1) gives
1
= 87, contradicting
1
0.
x = 5, y = 3, x +2y = 2. Obviously impossible.
We conclude that the only admissible pair satisfying (1) to (5) is (x, y) = (3/4, 0). The extreme-value
theoremimplies that we have found the solution. (Theorem18.4 of the next section also shows optimality.)
4. Using (a) in Theorem 18.3 with x = x implies that f (x
0
)
m

j=1

j
g
j
(x
0
) f ( x)
m

j=1

j
g
j
( x). But,
because x also solves [18.41], f ( x) = f (x
0
). Thus, because
j
0 and g
j
( x) c
j
, j = 1, . . . , m,
and also because of [18.45], we have ()
m

j=1

j
c
j

m

j=1

j
g
j
( x)
m

j=1

j
g
j
(x
0
) =
m

j=1

j
c
j
. Here the
two middle terms, being squeezed between two equal numbers, must themselves be equal. Therefore
f ( x)
m

j=1

j
g
j
( x) = f (x
0
)
m

j=1

j
g
j
(x
0
) f (x)
m

j=1

j
g
j
(x) for all x 0. Also, if
k
> 0 and
g
k
( x) < c
k
for any k, then
m

j=1

j
(g
j
( x) c
j
) < 0, which contradicts (). Thus x satises (a) and (b) in
Theorem 18.3.
6. (a) As suggested by the hint, L(x

) L(x

, ) for all 0 implies that


(1)
m

j=1

j
(g
j
(x

) c
j
)
m

j=1

j
(g
j
(x

) c
j
)
for all 0. If g
k
(x

) > c
k
for any k, then

m
j=1

j
(g
j
(x

) c
j
) can be made arbitrarily large by
choosing
k
large and
j
= 0 for all j = k. Hence, g
j
(x

) c
j
, j = 1, . . . , m. Now, however, because
Knut Sydster and Peter Hammond 2010
56 CHAPTER 18 CONS TRAI NED OPTI MI ZATI ON
one can have
j
= 0, j = 1, . . . , m, (1) implies that

m
j=1

j
(g
j
(x

) c
j
) 0. Yet

j
0 and
g
j
(x

) c
j
, j = 1, . . . , m, so

m
j=1

j
(g
j
(x

) c
j
) = 0.
Finally, whenever g
j
(x) c
j
, j = 1, . . . , m, the inequality L(x,

) L(x

) implies that
f (x) f (x

)
m

j=1

j
[g
j
(x) g
j
(x

)] =
m

j=1

j
[g
j
(x) c
j
] 0
Therefore f (x) f (x

), so x

solves [18.41].
(b) First, part (b) of the question should be corrected to read as follows:
Suppose that there exist x

0 and

0 satisfying both g
j
(x

) c
j
and g
j
(x

) = c
j
whenever

j
> 0 for j = 1, . . . , m, as well as L(x,

) L(x

) whenever x 0. Show that L(x, ) has a


saddle point at (x

) in this case.
Then an answer is as follows: The rst inequality in [] is satised by assumption. As for the second,
under the indicated hypotheses, one has
L(x

) L(x

, ) =
m

j=1
(
j

j
)[g
j
(x

) c
j
]
m

j=1

j
[g
j
(x

) c
j
] = 0
whenever 0. Thus, [] has been veried.
x
P
y
2x y c
2
2x y c
1
Figure 18.10.2
18.10
2. (a) As shown in Fig. 18.10.2, the feasible set is the intersection of the two circular disks with the non-
negative orthant. The solution is at the point P, where the circles intersect. Thus, (x +1)
2
+y
2
= 4 =
x
2
+(y +1)
2
. It follows that x = y and so 2x
2
+2x +1 = 4. The (relevant) positive root is
1
2
(

7 1),
so the solution is
_
1
2
(

7 1),
1
2
(

7 1)
_
. (b) The KuhnTucker conditions are:
2
1
2(x +1)
2
2x 0 (= 0 if x > 0) (1)
1
1
2y
2
2(y +1) 0 (= 0 if y > 0) (2)

1
0 (= 0 if (x +1)
2
+y
2
< 4) (3)

2
0 (= 0 if x
2
+(y +1)
2
< 4) (4)
The objective function is linear and thus concave. The constraint functions are both sums of convex
functions, so they are convex. The conditions (1) to (4) are therefore sufcient for an admissible pair to
Knut Sydster and Peter Hammond 2010
CHAPT ER 18 CONST RAI NED OP TI MI ZATI ON 57
solve the problem. With x = y =
1
2
(

7 1), equations (1) to (4) are satised with


1
= (7 +3

7)/28
and
2
= (3

7 7)/28.
(c) The change in the optimal value of 2x +y is approximately
2
0.1 0.003.
4. (a) If x

= a, f

(x

) 0; if x

(a, b), f

(x

) = 0; if x

= b, f

(x

) 0.
(b) See Figs. 18.10.4.
x
*
a b
x
y
a x
*
b
x
y
x
*
b a
x
y
(a)
Figure 18.10.4
(b) (c)
6. (a) See Fig. 18.10.6. The optimal solution is obviously at (x, y) = (1, 0). At this point the two constraints
g(x, y) = y (1 x)
3
0 and h(x, y) = y 0 are both binding. The gradient vector of g is
(g

1
, g

2
) = (3(1 x)
2
, 1) = (0, 1) at (1, 0), and the gradient of h is (h

1
, h

2
) = (0, 1). Hence, the
two gradients are linearly dependent at (1, 0) (in fact, (0, 1) = (1)(0, 1)), and so the constraint
qualication is violated at this point.
3x y c
x
1
y
1
y (1 x)
3
Figure 18.10.6
(b) The Lagrangean is L = 3x +y
1
(y (1 x)
3
) +
2
x +
3
y, so the KuhnTucker conditions are:
L

1
= 3 3
1
(1 x)
2
+
2
= 0 (1)
L

2
= 1
1
+
3
= 0 (2)

1
0 (= 0 if y < (1 x)
3
) (3)

2
0 (= 0 if x > 0) (4)

3
0 (= 0 if y > 0) (5)
Suppose x > 0. Then from (4),
2
= 0, and (1) implies
1
(1 x)
2
= 1. Thus
1
= 0, so
1
> 0 and
then from (3), y = (1 x)
3
. From
1
(1 x)
2
= 1 it follows also that x = 1, so y = (1 x)
3
= 0, and
Knut Sydster and Peter Hammond 2010
58 CHAPTER 19 L I NE AR P ROGRAMMI NG
hence y > 0. So from (5),
3
= 0. Then (2) yields
1
= 1. Therefore (1) implies that (1 x)
2
= 1. So
x = 2, because x > 0. But for x = 2, one has y = (1 x)
3
= 1, a contradiction.
Suppose x = 0. Then (1) implies 3 3
1
+
2
= 0, so 3
1
= 3 +
2
> 0. Thus, from (3),
y = (1 x)
3
= 1, and so by (5),
3
= 0. Then from (2),
1
= 1. So all the conditions (1)(5) are
satised by x = 0, y = 1, with
1
= 1 and
2
=
3
= 0.
So the KuhnTucker conditions have a unique solution, which is not optimal. Finally, the two
constraints g(x, y) = y (1 x)
3
0 and k(x, y) = x 0 are both binding at (0, 1). But (g

1
, g

2
) =
(3(1 x)
2
, 1) = (3, 1) and (k

1
, k

2
) = (1, 0) are linearly independent. So the constraint qualication
is satised at (0, 1), where the KuhnTucker conditions are met. The KuhnTucker conditions can be
violated at the optimum (1, 0), because the constraint qualication is not satised at that point.
Chapter 19 Linear Programming
19.1
2. (a) No maximum exists. Consider Fig. 19.1.2. By increasing c, the dashed level curve x
1
+x
2
= c for the
criterion function moves to the north-east and so this function can obviously take arbitrary large values.
(b) Yes, the maximum is at P = (1, 0). The level curves are the same as in (a), but the direction of
increase is reversed.
x
2
x
1 1 6 5 2 3 4
1
4
2
3
x
1
x
2
c
x
1
x
2
1
x
1
3x
2
3
700x 1000y c
1000 2000
1000
y
P
x
Figure 19.1.2 Figure 19.1.4
4. The LP problem is:
() max 700x +1000y subject to

3x + 5y 3900
x + 3y 2100
2x + 2y 2200
x 0 , y 0
The problem is represented geometrically in Fig. 19.1.4. The solution is obviously at P, where the two lines
3x + 5y = 3900 and 2x + 2y = 2200 intersect. Solving these equations yields x = 800 and y = 300. So
the rm should produce 800 TV sets of type A and 300 sets of type B.
19.2
2. (a) min 6u
1
+4u
2
subject to
_
3u
1
+ u
2
3
2u
1
+ 4u
2
4
u
1
0, u
2
0
(b) max 11x
1
+20x
2
subject to
_
x
1
+ 2x
2
10
3x
1
+ 5x
2
27
x
1
0, x
2
0
Knut Sydster and Peter Hammond 2010
CHAP TER 19 L I NE AR P ROGRAMMI NG 59
19.3
2. max 300x
1
+500x
2
subject to
_
10x
1
+ 25x
2
10 000
20x
1
+ 25x
2
8 000
x
1
0 , x
2
0
The solution can be found graphically. It is x

1
= 0, x

2
= 320, and the value of the criterion function is
160, 000, the same value found in Example 19.2 for the optimal value of the primal criterion function.
19.5
2. (a) Figure 19.5.2 shows a portion of the feasible set and two dashed level curves for the criterion function
Z = y
1
+2y
2
. We see that the minimum is attained at the point (y

1
, y

2
) = (3, 2).
(b) The dual is:
max 15x
1
+5x
2
5x
3
20x
4
subject to
x
1
+x
2
x
3
+ x
4
1
6x
1
+x
2
+x
3
2x
4
2
x
1
0, x
2
0, x
3
0, x
4
0
Because y

1
and y

2
are both positive, the rst two constraints are satised with equality. Furthermore,
the two last constraints in the primal problem are satised with strict inequality at the optimum. Hence
x

3
= x

4
= 0. Thus we have x

1
+x

2
= 1, 6x

1
+x

2
= 2, so x

1
= 1/5 and x

2
= 4/5. The maximum is
thus at (x

1
, x

2
, x

3
, x

4
) = (1/5, 4/5, 0, 0).
(c) If the constraint is changed to y
1
+6y
2
15.1, the solution to the primal (P) is still at the intersection
of the lines (1) and (2) in Fig. 19.5.2, but with (1) shifted up slightly. The solution to the dual is completely
unchanged. The optimal values in (P) and (D) both increase by (15.1 15) x

1
= 0.02.
y
2
5
10
y
1
5 10 15
(3)
y
1
+2y
2
= Z
0
(1)
(3, 2)
(2)
(4)
Figure 19.5.2
4. The dual to problem [1] is
[2] min q
1
V
1
+q
2
V
2
+r
1
x
1
+ +r
N
x
N
s.t.

1
1
q
1
+
1
2
q
2
+r
1
1

2
1
q
1
+
2
2
q
2
+r
2
1
.
.
.
.
.
.

N
1
q
1
+
N
2
q
2
+r
N
1
where the variables are all 0. The implications [i] to [vi] are all special cases of the complementary slackness
conditions [19.13] and [19.14] in Theorem 19.4.
Knut Sydster and Peter Hammond 2010
60 CHAPTER 20 DI F F E RENCE E QUATI ONS
Chapter 20 Difference Equations
20.1
2. (a) Monotone convergence to x

from below. (b) Damped oscillations around x

.
(c) Monotonically increasing towards . (d) Explosive oscillations around x

.
(e) x
t
= x

for all t . (f) Oscillations around x

with constant amplitude.


(g) Monotonically (linearly) increasing towards . (h) Monotonically (linearly) decreasing towards .
(i) x
t
= x
0
for all t.
4. (a) Because the parameters are positive, y
k+1
is positive provided that y
k
> 0. Because y
0
is positive, by
induction, y
t
is positive for all t . (b) Substituting y
t
= 1/x
t
gives the equation x
t +1
= (a/c)x
t
+b/c.
When a = 2, b = 3, and c = 4, this equation reduces to x
t +1
= (1/2)x
t
+ 3/4. When x
0
= 1/y
0
= 2,
the solution is x
t
= (1/2)
t +1
+3/2, and so y
t
=
_
(1/2)
t +1
+3/2
_
1
. Clearly, y
t
2/3 as t .
6. (a) From [1],
W
t +2
W
t +1
=
P
t +1
P
t
=
+W
t +1
+W
t
, and [2] follows immediately.(b) According to [2], the
fraction W
t +1
/( +W
t
) is the same for all t . For t = 0 it is equal to W
1
/P
0
= c, and thus [3] follows.
From [20.4], the solution is
W
t
=
_
W
0

c
1 c
_
(c)
t
+
c
1 c
(1 = c)
(c) The equation is stable iff |c| < 1. In this case, W
t

c
1 c
as t .
20.2
2. According to [4] in Example 20.5, the yearly repayment is
z =
0.07 100, 000
1 (1.07)
30
8058.64
In the rst year the interest payment is 0.07B = 7000, and so the principal repayment is 8058.64
7000 = 1058.64. In the last year, the interest payment is 0.07b
29
8058.64[1 (1.07)
1
] 527.20,
and so the principal repayment is 8058.64 527.20 = 7531.44.
20.4
2. x
t
= A + B t is a solution: x
t +2
2x
t +1
+ x
t
= A + B (t + 2) 2[A + B (t + 1)] + A + B t =
A +Bt +2B 2A 2Bt 2B +A +B t = 0. Substituting t = 0 and t = 1 in x
t
= A +B t yields
A = x
0
and A+B = x
1
, with solution A = x
0
and B = x
1
x
0
. So x
t
= A+B t is the general solution
of the given equation.
4. x
t
= (A +B t )2
t
+1 is a solution:
x
t +2
4x
t +1
+4x
t
= [A +B (t +2)]2
t +2
+1 4{[A +B (t +1)]2
t +1
+1} +4[(A +B t )2
t
+1] =
4A2
t
+4Bt 2
t
+8B2
t
+1 8A2
t
8Bt 2
t
8B2
t
4 +4A2
t
+4Bt 2
t
+4 = 1. Substituting t = 0
and t = 1 in x
t
= A2
t
+B t 2
t
+1 yields A+1 = x
0
and 2A+2B +1 = x
1
, with solution A = x
0
1
and B =
1
2
x
1
x
0
+
1
2
. So x
t
= A2
t
+B t 2
t
+1 is the general solution of the given equation.
Knut Sydster and Peter Hammond 2010
CHAPTE R 20 DI F F E RENCE EQUAT I ONS 61
20.5
2. (a) The characteristic equation m
2
+ 2m + 1 = (m + 1)
2
= 0 has the double root m = 1, so the
general solution of the homogeneous equation is x
t
= (C
1
+C
2
t )(1)
t
. We nd a particular solution by
inserting u

t
= A2
t
. This yields A = 1, and so the general solution of the inhomogeneous equation is
x
t
= (C
1
+C
2
t )(1)
t
+2
t
.
(b) By using the method of undetermined coefcients to determine the constants A, B, and C in the
particular solution u

t
= A5
t
+ B cos

2
t + C sin

2
t , we obtain A =
1
4
, B =
3
10
, and C =
1
10
. So the
general solution to the given equation is x
t
= C
1
+C
2
2
t
+
1
4
5
t
+
3
10
cos

2
t +
1
10
sin

2
t .
4. The characteristic equationis m
2
4(ab+1)m+4a
2
b
2
= 0, withsolutions m
1,2
= 2(ab+1

1 +2ab ).
The general solution is therefore D
n
= C
1
m
n
1
+C
2
m
n
2
.
6. Inserting x
t
= u
t
(a/2)
t
into [20.21], assuming that b =
1
4
a
2
, we obtain the equation
x
t +2
+ax
t +1
+
1
4
a
2
x
t
= u
t +2
(a/2)
t +2
+au
t +1
(a/2)
t +1
+
1
4
a
2
u
t
(a/2)
t
=
1
4
a
2
(a/2)
t
(u
t +2
2u
t +1
+ u
t
), which is 0 if u
t +2
2u
t +1
+ u
t
= 0. The general solution of this
equation is u
t
= A +B t , so x
t
= u
t
(a/2)
t
= (A +B t )(a/2)
t
, which is the result claimed for case
2 in the frame on page 751.
8. (a) The rst two equations state that consumption and capital are proportional to the net national product
in the previous period. The third equation states that net national product, Y
t
, is divided between con-
sumption, C
t
, and net investment, K
t
K
t 1
.
(b) To derive a second-order difference equation, rst replace t by t +2 in the last displayed equation in
the problem to obtain Y
t +2
= C
t +2
+K
t +2
K
t +1
. But C
t +2
= cY
t +1
, K
t +2
= Y
t +1
, and K
t +1
= Y
t
,
so we obtain Y
t +2
(c + )Y
t +1
+ Y
t
= 0. Explosive oscillations occur when (c + )
2
< 4 and
> 1.
10. From the quadratic formula, m
2
+ am + b has real zeros iff b
1
4
a
2
. Combining [20.27] and [20.28],
we must prove that f (m) = am
2
+bm+c = 0 has both roots in the interval (1, 1) iff |a| < 1 +b and
b < 1.
For these roots to be in the interval (1, 1), it is necessary that f (1) > 0, f (1) > 0, f

(1) < 0,
and f

(1) > 0. Thus 1 a +b > 0, 1 +a +b > 0, 2 +a < 0, and a +2 > 0, which is equivalent to
|a| < 1 +b and |a| < 2. But then, because the roots are real, b
1
4
a
2
< 1.
Conversely, if |a| < 1 + b and b < 1, then f (1) > 0, f (1) > 0, and |a| < 2. So f

(1) =
2 +a < 0 and f

(1) = 2 +a > 0.
12. (a) Because u
(1)
t
and u
(2)
t
are two linearly independent solutions of the homogeneous (!) equation, Au
(1)
t
+
Bu
(2)
t
is the general solution of the homogeneous equation. So, to prove that [] is the general solution of
[], it sufces to prove that U

t
= u
(1)
t
t 1

i=0
c
i
u
(2)
i+1
D
i+1
+u
(2)
t
t 1

i=0
c
i
u
(1)
i+1
D
i+1
is a particular solution of []. Insertion
in [], followed by tedious algebraic computation, eventually yields the result.
(b) p
t
= Ar
t
1
+Br
t
2
+
t 1

i=0
r
t i1
2
r
t i1
1
r
2
r
1
u
i+2
, t = 0, 1, . . .
(First write the equation in the form p
t +2
+
1
p
t +1
+
2
p
t
= u
t +2
, t = 0, 1, 2, . . .. Assuming that r
1
, r
2
are real(!) and different roots of the characteristic polynomial, two linearly independent solutions of the
homogeneous equation are u
(1)
t
= r
t
1
and u
(2)
t
= r
t
2
. So D
i+1
= r
i+1
1
r
i+2
2
r
i+2
1
r
i+1
2
= r
i+1
1
r
i+1
2
(r
2
r
1
).
Knut Sydster and Peter Hammond 2010
62 CHAPTER 21 DI F F E RENTI AL EQUATI ONS
Then
r
t
1
t 1

i=0
u
i+2
r
i+1
2
r
i+1
1
r
i+1
2
(r
2
r
1
)
+r
t
2
t 1

i=0
u
i+2
r
i+1
1
r
i+1
1
r
i+1
2
(r
2
r
1
)
=
t 1

i=0
r
t i1
2
r
t i1
1
r
2
r
1
u
i+2
is the particular solution in [] to the nonhomogeneous equation.)
Chapter 21 Differential Equations
21.1
2. (a) If x = Ct
2
, then x = 2Ct , and so t x = 2Ct
2
= 2x. (b) x = Ct
2
passes through (1, 2) if 2 = C 1
2
,
so C = 2. Hence, x = 2t
2
is the desired solution.
4. (a) Replace the problem by: If x = x(t ) satises x
2
= 2
_
1 t
2
, then x =
t
x

1 t
2
.
(The original problem makes no sense. The differential equation only makes sense if t (1, 1),
and then x
2
+ 2
_
1 t
2
= 0 is only satised for x = 0.) Differentiating each side of x
2
= 2

1 t
2
w.r.t. t yields 2x x = 2t /
_
1 t
2
, and so x = t /x
_
1 t
2
.
(b) Differentiating each side yields
1
2
e
t
2
2t e
x
x(x +1) +e
x
x = 0, and the result follows.
(c) Differentiating each side yields x
2
+ 2(1 t )x x = 3t
2
. Simplifying and using
(1 t )x
2
= t
3
yields the given equation.
6. The point of this problem is that you do not have to solve the differential equation (which would involve
the inverse tangent function, anyway). From x = (1+x
2
)t , we see at once that x < 0 for t < 0 and x > 0
for t > 0. Thus t = 0 is a global minimum point for x(t ), and because x(0) = 0, one has x(t ) 0 for all
t . Differentiating the given equation w.r.t. t yields x = 2x xt +(1 +x
2
) = 2xt (1 +x
2
)t +(1 +x
2
) =
(1 +x
2
)(2xt
2
+1). Clearly x > 0 for all t , so x = x(t ) is convex. (In fact, the equation does not have a
solution on all of , so x(t ) only has a minimum on an open interval around 0.)
21.2
2. See Fig. 21.2.2. (The solutions are the circles t
2
+ x
2
= C, t = 0, with C an arbitrary nonnegative
constant. This can be shown by direct differentiation, or by solving the separable equation x = t /x
using the method set out in the next section.)
x
1
t
1
t
2
+x
2
= 4
Figure 21.2.2
Knut Sydster and Peter Hammond 2010
CHAP TER 21 DI F F ERENT I AL EQUATI ONS 63
21.3
2. (a) Direct integration yields x =
1
4
t
4

1
2
t
2
+C. (b) x = t e
t
e
t

1
2
t
2
+C (c) e
x
(dx/dt ) = t +1, or
e
x
dx = (t +1) dt , so that
_
e
x
dx =
_
(t +1) dt and e
x
=
1
2
t
2
+t +C. The solution is x = ln(
1
2
t
2
+t +C).
4. In both cases

N depends on both N and t . (In the rst gure, for instance, N(t
1
) = N(t
2
), but

N(t
1
) =

N(t
2
).)
6. (a) x = Ct
a
(b) x = Ct
b
e
at
(c) x = Cbt
b
/(1 aCt
b
)
8. If z = x/t , then x = zt , so x = zt + z, and the equation x = g(x/t ) becomes the separable equation
t z = g(z)z. The special equation is x =
x
3
+t
3
3t x
2
=
1
3
x
t
+
1
3
_
x
t
_
2
. The suggested substitution leads
to t z =
1
3
z
2

2
3
z. Separating the variables and integrating yields
_
dz
z
2
2z
=
1
3
_
dt
t
. The integral
on the LHS is
_
z
2
dz
1 2z
3
. It can be found by introducing the new variable u = 1 2z
3
, implying that
du = 6z
2
dz. The nal answer is x =
3
_
1
2
t
3
+Ct .
21.4
2. (a) Because g() = 1/, f (s) = s, x
0
= 1, and t
0
=

2, formula [21.3] yields
x
_
1
d =
t
_

2
s ds,
or

x
1
1
2

2
=

2
1
2
s
2
, or
1
2
x
2

1
2
=
1
2
t
2
1. Thus, the required solution is x =

t
2
1. (Actually,
it is probably easier to nd the general solution x
2
t
2
= C rst, and then determine the appropriate
constant.)
(b) Formula [21.3] gives
_
x
0
( + 1)
2
d =
_
t
0
e
2s
ds, so

x
0
1/( + 1) =
1
2

t
0
e
2s
, implying that
1 1/(x +1) =
1
2
(1 e
2t
). Hence, x =
1 e
2t
1 +e
2t
.
4. Using the given identity, the separable differential equation [1] becomes
_ _
1
y
+
y
1
1 y

_
dy =
_
dx
x
Integration yields ln y (1/) ln |1 y

| = ln x + C
1
. Multiplying both sides by yields
ln y

ln |1 y

| = ln x

+C
1
, or
ln
y

|1 y

|
= ln e
C
1

, hence
y

1 y

= Cx

with C = e
C
1

Putting = 1/C and solving for y yields [2].


21.5
2. x = Ce
t /2
+
1
2
. The equilibrium state x

= 1/2 is stable. See Fig. 21.5.2.


4. dx/dt = b ax, so
_
dx/(b ax) =
_
dt +A for some constant A. This implies that (1/a) ln |b
ax| = t + A, so ln |b ax| = at aA, and |b ax| = e
at aA
= e
at
e
aA
. It follows that ()
b ax = e
at aA
= e
aA
e
at
= Ce
at
for C = e
aA
. The conclusion follows by solving ()
for x.
Knut Sydster and Peter Hammond 2010
64 CHAPTER 21 DI F F E RENTI AL EQUATI ONS
x
1
1
t
x =
1
2
e
t /2
+
1
2
(C =
1
2
)
x = e
t /2
+
1
2
(C = 1)
Figure 21.5.2
6. From x = X/N, by logarithmic differentiation, x/x =

X/X

N/N. Moreover, [2] implies that

X/X = a

N/N, so x/x = (a 1)

N/N = (a 1)[ (1/x)]. It follows that the differential equation
for x is x = (a 1)x (a 1). The solution of this equation is x(t ) = [x(0) /]e
(a1)t
+/.
Then [2] and x = X/N together imply that N(t ) = [x(t )/A]
1/(a1)
, X(t ) = A[N(t )]
a
. For 0 < a < 1,
x(t ) /, N(t ) (/A)
1/(a1)
, and X(t ) A(/A)
a/(a1)
as t .
21.6
2. Formula [21.12] yields
x =
_
t
0
s(1 +s
2
)e
_
t
s
2 d
ds =
_
t
0
s(1 +s
2
)e
t
2
s
2
ds
By substituting u = t
2
, then integrating by parts, we eventually derive the solution x(t ) = e
t
2
1
1
2
t
2
.
It follows that lim
t
x(t ) does not exist because x(t ) .
4. Multiply the given equation by x
n
to obtain x
n
x = Q(t )x
1n
+ R(t ). From z = x
1n
we nd
z = (1 n)x
n
x, so the new equation is
1
1n
z = Q(t )z + R(t ), which is a linear differential equation
in the unknown function z.
6. Using the technique suggested by Problem 4 yields the solution
K =
_
Ce
(1b)t
+An
a
0
(1 b)e
(av+)t
/(av + +(1 b))
_
1/(1b)
21.7
2. (a) x = (x 1)(x + 1)
2
. Here x = 1 is unstable, whereas x = 1 is neither stable nor unstable. (It
is stable on the right, but unstable on the left.) See Fig. 21.7.2(a). (b) No equilibrium states. See Fig.
21.7.2(b). (c) The only equilibrium state x = 0 is unstable. See Fig. 21.7.2(c).
2
1
1
2
x
2 2
x
2
1
1
2
x
1 1
x
2
1
1
2
x
1 3 2
x
Figure 21.7.2(a) Figure 21.7.2(b) Figure 21.7.2(c)
Knut Sydster and Peter Hammond 2010
CHAP TER 21 DI F F ERENT I AL EQUATI ONS 65
4. (a) x(t ) = (1 +Ae
t
)/(1 Ae
t
), where A = (x
0
1)/(x
0
+1). Another solution is x(t ) = 1 for all t .
For x
0
< 1, x(t ) 1 as t . For x
0
> 1, which occurs when 0 < A < 1, one has x(t ) as
t (ln A)

, and x(t ) as t (ln A)


+
. For x
0
= 1, x(t ) 1. See Fig. 21.7.4(a) for some
integral curves. (b) x = 1 is stable; x = 1 is unstable. See Fig. 21.7.4(b).
x
t
x 1
x 1
x
1
2
x
2 1 1 2
Figure 21.7.4(a) Figure 21.7.4(b)
21.8
2. (a) It is easy to verify by direct differentiation that u
1
= e
t
and u
2
= t e
t
both satisfy x 2 x +x = 0. If
t e
t
= ke
t
for all t , thent = k for all t , whichis absurd. The general solutionis therefore x(t ) = Ae
t
+Bt e
t
,
where A and B are arbitrary constants. (b) A particular solution of the equation is obviously u

(t ) = 3,
so the general solution of the nonhomogeneous equation is x(t ) = Ae
t
+Bt e
t
+3.
4. By direct differentiation, u
1
= e
at
and u
2
= e
at /(1)
are easily seen to be solutions. They are not
proportional, so the general solution is x(t ) = Ae
at
+Be
at /(1)
, where Aand B are arbitrary constants.
21.9
2. (a) x = C
1
e
t
+ C
2
e
t

1
2
sin t . Not stable. (b) x = C
1
e
t
+ C
2
e
t

1
2
t e
t
. Not stable.
(c) x = Ae
5t
+Bt e
5t
+
2
75
t +
3
125
. Not stable.
4. x = Ae
2t
+Be
7t /4
+
1
14
t +
15
14
2
+
1
65
(2 sin t +3 cos t )
6. (a) If x = tf (1/t ), then x = f (1/t ) (1/t )f

(1/t ) and x = (1/t


3
)f

(1/t ). But then x + t


n2
x =
(1/t
3
)f

(1/t ) +t
n2
tf (1/t ) = (1/t
3
)[f

(1/t ) +t
n+2
f (1/t )] = 0, because when x = f (t ) satises
x +t
n2
x = 0, then f

(u) +u
n2
f (u) = 0 at u = 1/t in particular, and so f

(1/t ) +t
n+2
f (1/t ) = 0.
(b) Let n = 2. Then x + t
n2
x = x + x = 0, with general solution x = Asin t + B cos t . From (a), it
follows that x+t
n2
x = x+t
4
x = 0, or t
4
x+x = 0, has the solution x = t [Asin(1/t )+B cos(1/t )].
8. For = (a ) > 0 the solution is p(t ) = C
1
e
rt
+ C
2
e
rt
k/, where r =

; for = 0 the
solution is p(t ) = C
1
t +C
2
+
1
2
kt
2
; for < 0 the solution is p(t ) = C
1
cos st +C
2
sin st k/, where
s =

. In no case is the solution ever stable.


10. If x = ue
rt
, then x = e
rt
( u + ru) and x = e
rt
[ u + 2r u + r
2
u]. Hence, x + a x + bx = e
rt
[ u + (2r +
a) u +(r
2
+ar +b)] = e
rt
u because r = a/2 and r satises the characteristic equation. We conclude
that x = ue
rt
satises the given equation provided u = 0. Then u = At +B, and the conclusion follows.
Knut Sydster and Peter Hammond 2010
66 APPENDI X B S UMS , PRODUCTS , AND I NDUCTI ON
Appendix B Sums, Products, and Induction
B.1
2. (a) 2

0 +2

1 +2

2 +2

3 +2

4 = 2(1 +

2 +

3 +2) = 2(3 +

2 +

3)
(b) (x +0)
2
+(x +2)
2
+(x +4)
2
+(x +6)
2
= 4(x
2
+6x +14)
(c) a
1i
b
2
+a
2i
b
3
+a
3i
b
4
+ +a
ni
b
n+1
(d) f (x
0
)x
0
+f (x
1
)x
1
+f (x
2
)x
2
+ +f (x
m
)x
m
4.
2 3 +3 5 +4 7
1 3 +2 5 +3 7
100 =
6 +15 +28
3 +10 +21
100 =
49
34
100 144.12
6. (a) The total number of people moving fromregion i. (b) The total number of people moving to region j.
B.2
2. (a +b)
6
= a
6
+6a
5
b +15a
4
b
2
+20a
3
b
3
+15a
2
b
4
+6ab
5
+b
5
. (The coefcients are those in the sixth
row of Pascals triangle in the text.)
4. (a)
_
8
3
_
=
8 7 6
1 2 3
= 56. Also,
_
8
8 3
_
=
_
8
5
_
= 56;
_
8
3
_
+
_
8
3 +1
_
= 56 +
8 7 6 5
1 2 3 4
= 126 and
_
8 +1
3 +1
_
=
_
9
4
_
=
9 8 7 6
1 2 3 4
= 126.
(b)
_
m
k
_
=
m(m1) (mk +1)
k!
=
m!
(mk)!k!
=
_
m
mk
_
and
_
m
k
_
+
_
m
k +1
_
=
m!
(mk)!k!
+
m!
(mk 1)!(k +1)!
=
m!(k +1 +mk)
(mk)!(k +1)!
=
(m+1)!
(mk)!(k +1)!
=
_
m+1
k +1
_
6.
n1

i=0
(a +id) = a +(a +d) + +[a +(n 2)d] +[a +(n 1)d]
= [a + (n 1)d] + [a + (n 2)d] + + (a + d) + a =
1
2
[2a + (n 1)d]n = na +
n(n1)d
2
.
An alternative proof is this:
n1

i=0
(a +id) =
n1

i=0
a +d
n1

i=0
i = na +
1
2
[1 +(n 1)](n 1) = na +
n(n 1)d
2
.
B.3
2.

i
j=1
a
ij
is the sum of all the i numbers in the ith row, so in the rst double sum we sum all the sums in
the m rows.

m
i=j
a
ij
is the sum of all the mj +1 numbers in the jth column, so in the second double
sum we sum all the sums in the m columns.
B.4
2. (a)
n

k=1
2k
2k 1
2k
2k +1
=
n

k=1
4k
2
4k
2
1
=
4 1
2
4 1
2
1

4 2
2
4 2
2
1

4n
2
4n
2
1
(b)
n

i=1
a
i
b
i
=
a
1
b
1
a
2
b
2

a
n
b
n
(c)
n1

i=1
_
n

s=i+1
a
s
_
b
i
=
n1

i=1
_
a
i+1
a
i+2
a
n
_
b
i
=
(a
2
a
3
a
n
)b
1
+(a
3
a
4
a
n
)b
2
+ +a
n
b
n1
Knut Sydster and Peter Hammond 2010
APPE NDI X C TRI GONOME TRI C F UNCTI ONS 67
B.5
2. We prove only [B.7]; the proof of [B.6] is very similar, but slightly easier. For n = 1 the LHS and the
RHS of [B.7] are both equal to 1. Suppose [B.7] is true for n = k, so that
k

i=1
i
3
= 1
3
+2
3
+3
3
+ +k
3
=
_
k(k +1)
2
_
2
Then
k+1

i=1
i
3
= 1
3
+2
3
+3
3
+ +k
3
+(k +1)
3
=
_
k(k +1)
2
_
2
+(k +1)
3
= (k +1)
2
(
1
4
k
2
+k +1)
But this last expression is equal to
(k +1)
2
(k
2
+4k +4)
4
=
_
(k +1)(k +2)
2
_
2
, which proves that [B.7]
is true for n = k +1. By induction, we have proved [B.7].
4. The claim is true for n = 1. Suppose k
3
+(k +1)
3
+(k +2)
3
is divisible by 9. Then (k +1)
3
+(k +
2)
3
+(k +3)
3
= (k +1)
3
+(k +2)
3
+k
3
+9k
2
+27k +27 = k
3
+(k +1)
3
+(k +2)
3
+9(k
2
+3k +3)
is divisible by 9, because the rst three terms are divisible by 9 by the induction hypothesis, whereas the
last term is also obviously divisible by 9.
6. s
1
= 41, s
2
= 43, s
3
= 47, s
4
= 53, s
5
= 61 are all prime. For n = 41, we have n
2
n +41 = (41)
2
,
which is not prime!
Appendix C Trigonometric Functions
C.1
2. Look at Fig. C.1 on page 866 of the text. If x is changed to x, the point P
x
will have coordinates (u, v),
so sin(x) = sin x, and cos(x) = cos x. Then by denition [C.2], tan(x) = sin(x)/ cos(x) =
sin x/ cos x = tan x.
4. cos(y /2) = sin y follows directly from [C.8]. Then, from the hints in the question, as well as [C.8]
and the result of Problem 2, it follows that
sin(x +y) = cos[x +(y /2)] = cos x cos[(y /2)] +sin x sin[(y /2)]
= sin x cos y +cos x sin y
Also, using the result of Problem 2 again, this last equation implies that
sin(x y) = sin x cos(y) +cos x sin(y) = sin x cos y cos x sin y
6. (a) 1/2. (Either draw a gure similar to Figure C.3 on page 868, or use the formula for sin(x y) in
Problem 4.) (b)

3/2 (c)

2/2 (d)

2/2 (e)

3/3 (f) sin /12 = sin(/3 /4) =
sin /3 cos /4 cos /3 sin /4 =
1
4
(

2).
8. Note that x + y = A and x y = B imply x =
1
2
(A + B) and y =
1
2
(A B). The claimed formula
then follows easily from the hint.
Knut Sydster and Peter Hammond 2010
68 APPENDI X C T RI GONOME TRI C F UNCTI ONS
10. (a) See Fig. C.1.10(a). Period , amplitude 1. (b) See Fig. C.1.10(b). Period 4, amplitude 3.
(c) See Fig. C.1.10(c). Period 2/3, amplitude 2.
y
1
1
x

y
3
2
1
1
2
3
x
2 4
y
2
4
x 2
Figure C.1.10(a) Figure C.1.10(b) Figure C.1.10(c)
12. (a) y = 2 sin
1
4
x (b) y = 2 +cos x (b) y = 2e
x/
cos x
14. The given equality is (cos x cos y)
2
+(sin x sin y)
2
= 1+12 cos(x y). By expanding and using
[C.7], we easily nd formula [C.8].
C.2
2. y = tan x =
sin x
cos x
gives y

=
cos x cos x sin x(sin x)
cos
2
x
=
cos
2
x +sin
2
x
cos
2
x
= 1 + tan
2
x. Since
cos
2
x +sin
2
x = 1, we get y

= 1/ cos
2
x as an alternative answer.
4. (a) a sin ax (b) a sin bt +abt cos bt (c) a cos(at +b) sin[sin(at +b)] cos{cos[sin(at +b)]}
6. f

(x) = 3(sin x x 1)
2
(cos x 1). In the open interval I = (0, 3/2), f

(x) = 0 only at cos x = 1,


when x = /2. (It is easy to see that sin x < x + 1 for all x 0 because the function dened by
g(x) = x + 1 sin x for all x 0 satises g(0) = 1 and g

(x) = 1 cos x 0 for all x 0.) Thus,


the only possible extreme points are 0, /2, and 3/2. Comparing the function values at these points,
we nd that f (x) has its maximum 1 at x = 0, and its minimum (2 + 3/2)
3
at x = 3/2. (The
extreme-value theorem ensures that extreme points do exist.)
8. Implicit differentiation yields 1 cos y +x(sin y)y

sin x y cos x = 0, so y

=
cos y y cos x
sin x +x sin y
.
At (, /2), y

= 1/2, so the equation for the tangent is y = x/2.


10. (a) cos x + C (b)
_
/2
0
cos x dx =

/2
0
sin x = sin(/2) 0 = 1 (c) Integrating by parts yields
I =
_
sin
2
x dx = sin x(cos x)
_
cos x(cos x) dx = sin x cos x+
_
cos
2
x dx = sin x cos x+
_
(1sin
2
x) dx. Hence, I = sin x cos x+xI +C. Solving for I gives I =
1
2
(sin x cos xx)+C
1
.
(d)
_

0
x cos x dx =

0
x sin x
_

0
sin x dx = 0 +

0
cos x = cos cos 0 = 2.
12. (a) /4 (b) /2 (c) /6 (d) /3. (You can read all these values off from Table C.1.)
14. Make use of [C.20] and the chain rule to derive y

=
1
1 +
_
e
x
e
x
2
_
2

e
x
+e
x
2
. Then simplify.
C.3
2. See Fig. C.3.2.
Knut Sydster and Peter Hammond 2010
69
4. (a) 2

3(cos /3 +i sin /3) (b) cos +i sin (c) 4(cos 4/3 +i sin 4/3)
(d)

2(cos 7/4 +i sin 7/4)


Imaginary axis
2i
i
i
2i
3i
4i
Real axis
1 1 2 3 4
w = 1 +3i
z +w = 3 +i
z = 2 2i
Figure C.3.2
Knut Sydster and Peter Hammond 2010
70
TEST I (Elementary Algebra)
A certain familiarity with elementary algebra is an essential prerequisite for reading the textbook (and for
understanding most modern economics texts). This test is designed for students and instructors to discover
whether the students have the proper background. (In a number of countries, many beginning economics
students background in elementary algebra appears to have become much weaker during the last few years.
In fact, lecturers using this test (or similar ones) have been shocked by the results, and have had to readjust
their courses.)
At the head of each problem, immediately after the number, the relevant sections of the introductory
chapters in the book are given in parentheses, followed in square brackets by the number of points for a
correct answer to each separate part of the problem. In a 2030 minute test, any student who scores less than
50 (out of 100) has serious problems with elementary algebra. Such students denitely need to review the
relevant section of Chapters 1 and 2, or consult other elementary material.
The correct answers are given on a separate page following the test.
1 (1.2) [Points: (a) 2; (b) 2; (c) 3; (d) 3] Calculate/simplify:
(a)
7
3
7
2
7
4
(b) (5.5 3.5)
3
(c)
_
2
5
__
2
5
__
2
5
_
(d)
2
19
2
17
2
19
+2
17
2 (1.21.4) [Points: (a) 2; (b) 2; (c) 2; (d) 4]
(a) If 2x
2
y = 5, then 4x
4
y
2
=? (b) 11 % of 3500 is? (c)

13
2
12
2
=?
(d) Rationalize the denominator of

3 +

2
(i.e. nd a new fraction that is equal but has no square
root in the denominator).
3 (1.3) [Points: (a) 2; (b) 2; (c) 2] Expand:
(a) (x +2y)
2
(b) (2x 3y)
2
(c) (a +b)(a b)
4 (1.3) [Points: (a) 2; (b) 3; (c) 3; (d) 4] Expand and simplify:
(a) 5a (3a +2b) 2(a 3b) (b) (x +2)
2
+(x 2)
2
2(x +2)(x 2)
(c) (1 x)
2
(1 +x)
2
(d) (2 a)
3
5 (1.2) [Points: 4] If the GNP of a certain country in 2000 was 8 billion dollars, write down an expression
for the GNP 6 years later if it increases by 5% each year.
6 (1.3) [Points: (a) 3; (b) 3; (c) 4] Factorize:
(a) 5a
2
b +15ab
2
(b) 9 z
2
(c) p
3
q 4p
2
q
2
+4pq
3
Knut Sydster and Peter Hammond 2010
71
7 (1.4) [Points: (a) 2; (b) 2; (c) 2] Expand and simplify to a single fraction:
(a)
1
2

1
3
(b)
6a
5

a
10
+
3a
20
(c)
1
2

1
3
1
4

1
6
8 (1.5) [Points: (a) 2; (b) 2; (c) 2; (d) 2] Calculate/simplify:
(a) 25
1/2
(b) (x
1/2
y
1/4
)
4
(c)
3

27a
6
(d) p
1/5
(p
4/5
p
1/5
)
9 (2.1) [Points: (a) 2; (b) 2; (c) 2] Solve the following equations for the unknown x:
(a)
3
5
x = 6 (b)
1
x 1
=
3
2x +3
(c)

3 x = 2
10 (1.6) [Points: (a) 2; (b) 3; (c) 3] Solve the following inequalities:
(a) 3x +2 < 5 (b)
x 1
x +3
0 (c) x
3
< x
11 (2.3) [Points: (a) 3; (b) 3; (c) 3] Solve the following equations:
(a) 3x 9x
2
= 0 (b) x
2
2x 15 = 0 (c) 2P
2
= 2 3P
12 (2.4) [Points: (a) 3; (b) 4; (c) 4] Solve the following systems of equations:
(a)
2x y = 5
x +2y = 5
(b)
1.5p 0.5q = 14
2.5p +1.5q = 28
(c)
3
p
+
3
q
= 3
3
p

1
q
= 7
Knut Sydster and Peter Hammond 2010
72
TEST II (Elementary Mathematics)
Students who now enter university or college courses in economics tend to have a wide range of math-
ematical backgrounds and aptitudes. At the low end, they may have no more than a shaky command of
elementary algebra. Or, at the high end, they may already have a ready facility with calculus, though
often it is some years since economics students took their last formal mathematics course. Experience
suggests therefore that right from the start of the course, it is very important that the instructor, as well
as each individual student, should get some impression of what the student knows well, what is vaguely
familiar, and what seems to be more or less forgotten or perhaps never learned at all.
The present test is meant to test the students actual knowledge of some elementary mathematical
topics of interest to economists. The level is nevertheless more advanced than for Test I and the topics
covered are discussed in the earlier chapters of the main text. The maximum total score is 100.
1 [Points: (a) 2; (b) 2; (c) 2; (d) 2]
(a) 2
5
+2
5
= 2
x
x = (b) 3
15
+3
15
+3
15
= 3
y
y =
(c)
_
13
2
5
2
= (d)
2
26
2
23
2
26
+2
23
=
z
9
z =
2 [Points: (a) 1+1; (b) 2+2]
(a) Find the slopes of the following straight lines:
(i) y =
3
2
x +4 (ii) 6x 3y = 5
(b) Find the equation of the straight line that:
(i) passes through (2, 3) and has slope 2.
(ii) passes through both (a, 0) and (0, b).
3 [Points: 5] Fill in the following table and sketch the graph of y = x
2
+2x +4.
x 2 1 0 1 2 3 4
y = x
2
+2x +4
4 [Points: 4+4] Determine the maximum/minimum points for:
(a) y = x
2
4x +8 (b) y = 2x
2
+16x 14
5 [Points: 4+4] Perform the following polynomial divisions:
(a) (2x
3
3x +10) (x +2) (b) (x
4
+x) (x
2
1)
6 [Points: 5] f (x) =
4
3
x
3

1
5
x
5
. For what values of x is f

(x) = 0?
7 [Points: 3+3+3+3] Sketch the graph of a function f in each of the following cases:
(a) f

(x) > 0 and f

(x) > 0, (b) f

(x) > 0 and f

(x) < 0
(c) f

(x) < 0 and f

(x) > 0, (d) f

(x) < 0 and f

(x) < 0
Knut Sydster and Peter Hammond 2010
73
8 [Points: 3+3]
(a) The cost in dollars of extracting T tons of a mineral ore is given by C = f (T ). Give an economic
interpretation of the statement that f

(1000) = 50.
(b) A consumer wants to buy a certain item at the lowest possible price. Let P(t ) denote the lowest price
found after searching the market for t hours. What are the likely signs of P

(t ) and P

(t )?
9 [Points: 2+2+2+2] Write down the general rules for differentiating the following:
(a) y = f (x) +g(x) y

= (b) y = f (x)g(x) y

=
(c) y = f (x)/g(x) y

= (d) y = f (g(x)) y

=
10 [Points: 2+2+2+2+2+2+2+2] Differentiate the following functions:
(a) y = x
2
(b) y = x
5
/5 (c) y =
x
x +1
(d) y = (x
2
+5)
6
(e) y = e
x
(f) y = ln x (g) y = 2
x
(h) y = x
x
11 [Points: 2+2+2+2+2] Which of the following statements are correct?
(a) The rule which converts a temperature measured in degrees Fahrenheit into the same temperature
measured in degrees Celsius is an invertible function.
(b) A concave function always has a maximum.
(c) A differentiable function can only have an interior maximum at a stationary point for the function.
(d) If f

(a) = 0, then a is either a local maximum point or a local minimum point.


(e) The conditions f

(a) = 0 and f

(a) < 0 are necessary and sufcient for a to be a local maximum


point for f .
12 [Points: 2+2+2+2] In each of the following cases, decide whether the given formula is correct or not:
(a)
_
x
2
dx =
1
3
x
3
+C
(b)
_
[f (x) +g(x)] dx =
_
f (x) dx +
_
g(x) dx
(c)
_
f (x)g(x) dx =
_
f (x) dx
_
g(x) dx
(d)
_
b
a
x dx = b a
Knut Sydster and Peter Hammond 2010
74
Answers to TEST I
1. (a)
7
3
7
2
7
4
=
7
3+2
7
4
=
7
5
7
4
= 7
54
= 7
1
= 7 (b) (5.5 3.5)
3
= 2
3
= 8
(c)
_
2
5
__
2
5
__
2
5
_
=
8
125
(= 0.064) (d)
2
19
2
17
2
19
+2
17
=
2
17
(2
2
1)
2
17
(2
2
+1)
=
3
5
2. (a) 4x
4
y
2
= (2x
2
y)
2
= 25 (b) 11 % of 3500 is 3500 11/100 = 3500 0.11 = 385
(c)

13
2
12
2
=

(13 +12)(13 12) =

25 = 5 (or

13
2
12
2
=

169 144, etc.)
(

13
2
12
2
=

13
2

12
2
= 13 12 = 1 is a SERIOUS mistake.)
(d)

3 +

2
=
(

3 +

2)(

3 +

2)
(

2)(

3 +

2)
=
3 +2

2 +2
3 2
= 5 +2

6
3. (a) (x +2y)
2
= x
2
+4xy +4y
2
(b) (2x 3y)
2
= 4x
2
12xy +9y
2
(c) (a +b)(a b) = a
2
b
2
4. (a) 5a (3a +2b) 2(a 3b) = 5a 3a 2b 2a +6b = 4b
(b) (x +2)
2
+(x 2)
2
2(x +2)(x 2) = [(x +2) (x 2)]
2
= 4
2
= 16
(c) (1 x)
2
(1 +x)
2
= [(1 x)(1 +x)]
2
= (1 x
2
)
2
= 1 2x
2
+x
4
(d) (2 a)
3
= (2 a)
2
(2 a) = (4 4a + a
2
)(2 a) = 8 4a 8a + 4a
2
+ 2a
2
a
3
=
a
3
+6a
2
12a +8
5. 8(1.05)
6
billion dollars.
6. (a) 5a
2
b +15ab
2
= 5ab(a +3b) (b) 9 z
2
= (3 z)(3 +z)
(c) p
3
q 4p
2
q
2
+4pq
3
= pq(p
2
4pq +4q
2
) = pq(p2q)
2
. (One point for the rst equality.)
7. (a)
1
2

1
3
=
3
2 3

2
3 2
=
3
6

2
6
=
1
6
(b) 20 is the common denominator, so
6a
5

a
10
+
3a
20
=
4 6a
20

2a
20
+
3a
20
=
24a 2a +3a
20
=
25a
20
=
5a
4
(c)
1
2

1
3
1
4

1
6
=
6
12

4
12
3
12

2
12
=
2
12
1
12
= 2 (or
1
2

1
3
= 2(
1
4

1
6
), so the ratio is 2).
8. (a) 25
1/2
= 5 (b) (x
1/2
y
1/4
)
4
= x
(1/2)4
y
(1/4)4
= x
2
y
1
(c)
3

27a
6
=
3

27
3

a
6
= 3a
2
(d) p
1/5
(p
4/5
p
1/5
) = p
1/5
p
4/5
p
1/5
p
1/5
= p
1/5+4/5
p
1/51/5
= p 1
9. (a) 3x = 30, so x = 10
(b) 2x +3 = 3(x 1), so x = 6. (Neither denominator is 0 when x = 6.)
(c) If

3 x = 2, then 3 x = 4, so x = 1. This is indeed a solution, as is easily checked.


10. (a) 3x + 2 < 5 or 3x < 3, so that x > 1. (Recall that an inequality is reversed if multiplied
by a negative number.)
(b) 3 < x 1. (Use a sign diagram. Note that the fraction is undened if x = 3.)
(c) x < 1 or 0 < x < 1. (x
3
< x, or x
3
x < 0, and so x(x
2
1) < 0, or x(x 1)(x +1) < 0.
Then use a sign diagram.)
11. (a) 3x(1 3x) = 0, so x = 0 or x = 1/3 (b) x = 3, x = 5 (c) P = 2 or P = 1/2
12. (a) x = 3, y = 1 (b) p = 10, q = 2 (c) Put x = 1/p, y = 1/q. Then 3x + 3y = 3 and
3x y = 7. The solution to this system is x = 2 and y = 1. But then p = 1/2 and q = 1.
Knut Sydster and Peter Hammond 2010
75
Answers to TEST II
1. (a) 2
5
+2
5
= 2 2
5
= 2
6
, so x = 6.
(b) 3
15
+3
15
+3
15
= 3 3
15
= 3
14
, so y = 14.
(c)

13
2
5
2
=

169 25 =

144 = 12.
(d)
2
26
2
23
2
26
+2
23
=
2
23
(2
3
1)
2
23
(2
3
+1)
=
2
3
1
2
3
+1
=
7
9
, so z = 7.
2. (a) (i) 3/2 (ii) 2 (b) (i) y = 2x 1 (ii) y = (b/a)x +b
3.
x 2 1 0 1 2 3 4
y = x
2
+2x +4 4 1 4 5 4 1 4
The graph of the function is shown in the gure below.
y
x
y
x
y
4
3
2
1
1
2
3
4
5
x
4 3 2 1 1 2 3 4
y = x
2
+2x +4
4. (a) Minimum 4 for x = 2. (Follows from x
2
4x +8 = x
2
4x +2
2
+8 2
2
= (x 2)
2
+4,
or by using calculus.)
(b) Maximum 18 for x = 4. (Follows from 2x
2
+16x 14 = 2(x
2
8x +7) =
2(x
2
8x +4
2
+7 4
2
) = 2[(x 4)
2
9] = 2(x 4)
2
+18, or by using calculus.)
5. (a)
2x
3
3x +10
x +2
= 2x
2
4x +5
(b)
x
4
+x
x
2
1
= x
2
+1 +
x +1
x
2
1
= x
2
+1 +
1
x 1
6. f

(x) = 4x
2
x
4
= x
2
(4 x
2
) = x
2
(2 x)(2 +x) = 0 for x = 0 and for x = 2.
7. y
x
y = f (x)
(a)
y
x
y = f (x)
(b)
y
x
y = f (x)
(c)
y
x
y = f (x)
(d)
Knut Sydster and Peter Hammond 2010
76
8. (a) The marginal cost is 50 when output is 1000 tons. (Or: The cost of extracting one ton more than
1000 tons is approximately 50 dollars.)
(b) P

(t ) 0, because more search never leads to a higher best price, and generally leads to a lower
one. However, P

(t ) is likely to be positive, because the longer you search the smaller gain in price
you will obtain. (P(t ) is decreasing and convex.)
9. (a) y = f (x) +g(x) y

= f

(x) +g

(x)
(b) y = f (x)g(x) y

= f

(x)g(x) +f (x)g

(x)
(c) y = f (x)/g(x) y

=
f

(x)g(x) f (x)g

(x)
(g(x))
2
(d) y = f (g(x)) y

= f

(g(x))g

(x)
10. (a) y = x
2
y

= 2x (b) y =
1
5
x
5
y

= x
4
(c) y =
x
x +1
y

=
1
(x +1)
2
(d) y = (x
2
+5)
6
y

= 6(x
2
+5)
5
2x = 12x(x
2
+5)
5
(e) y = e
x
y

= e
x
(f) y = ln x y

= 1/x (g) y = 2
x
y

= 2
x
ln 2 (h) y = x
x
y

= x
x
(ln x +1)
11. (a) Correct. (b) Wrong. (Consider y = e
x
.) (c) Correct. (d) Wrong. (a could be an inec-
tion point.) (e) Wrong. (The conditions are sufcient, but not necessary. For example, f (x) =
x
4
has a maximum at x = 0 and yet f

(0) = f

(0) = 0.)
12. (a) Correct. (b) Correct. (c) Wrong, because
d
dx
__
f (x) dx
_
g(x) dx
_
= f (x)
_
g(x) dx +g(x)
_
f (x) dx = f (x)g(x), except for very special functions f and g.
(d) Wrong, because
_
b
a
x dx =

b
a
1
2
x
2
=
1
2
(b
2
a
2
), which equals b a only when a +b = 2 or
a = b.
Knut Sydster and Peter Hammond 2010

Você também pode gostar